Contents

advertisement
Contents
Contents
iv
List of Tables
v
List of Figures
vi
1
Mathematics Proof Method
Metode Pembuktian Matematis
1.1
1
Direct Proof
Bukti Langsung . . . . . . . . . . . . . . . . . . . . . . . . . . . . .
1.2
Indirect Proof
Bukti Tak Langsung . . . . . . . . . . . . . . . . . . . . . . . . . . .
2
Number Theory
Teori Bilangan
2.1
6
11
Divisibility
Keterbagian . . . . . . . . . . . . . . . . . . . . . . . . . . . . . . .
2.2
3
11
Special Number
Bilangan Khusus . . . . . . . . . . . . . . . . . . . . . . . . . . . .
2.2.1
A Prime and Composite Number
Bilangan Prima dan Komposit . . . . . . . . . . . . . . . .
2.2.2
14
14
Perfect Square
Bilangan Kuadrat Sempurna . . . . . . . . . . . . . . . . .
i
15
CONTENTS
ii
2.3
GCD dan Algoritma Euclid . . . . . . . . . . . . . . . . . . . . . .
2.4
Modular Arithmetic
Modulo Aritmatik . . . . . . . . . . . . . . . . . . . . . . . . . . . .
2.5
Algebra Functions
Fungsi Aljabar
3.1
23
28
Polynomials Inequality
Pertidaksamaan Polinomial . . . . . . . . . . . . . . . . . . . . . .
3.1.1
3.1.2
28
Inverse Function
Fungsi Invers . . . . . . . . . . . . . . . . . . . . . . . . . .
32
Arithmetic and Geometric Sequence
Barisan Aritmatik dan Geometrik . . . . . . . . . . . . . .
3.2
19
Linear Diophantine Equations
Persamaan Linier Diophantin . . . . . . . . . . . . . . . . . . . . .
3
17
34
Arithmetic, Geometric, Harmonic, and Quadratic Means
Rataan Aritmatik, Geometrik, Harmonik dan
Kuadratik . . . . . . . . . . . . . . . . . . . . . . . . . . . . . . . .
3.3
43
The Polynomials and Remainder Theorem
Suku Banyak dan Teorema Sisa . . . . . . . . . . . . . . . . . . . .
3.3.1
Polynomials Division
Pembagian Suku Banyak . . . . . . . . . . . . . . . . . . .
3.3.2
47
Remainder Theorem
Teorema Sisa . . . . . . . . . . . . . . . . . . . . . . . . . .
3.3.3
47
49
Factor Theorem
Teorema faktor . . . . . . . . . . . . . . . . . . . . . . . . .
49
CONTENTS
3.3.4
iii
Properties of Polynomial Roots
Sifat-Sifat Akar-Akar Suku Banyak . . . . . . . . . . . . . .
4
Trigonometry
Trigonometri
4.1
4.1.1
4.1.2
63
Formulas of Sum and Difference of Angles
Rumus-rumus Jumlah dan Selisih Sudut . . . . . . . . . .
4.1.3
58
Sine and Cosine Rule
Aturan Sinus dan Cosinus . . . . . . . . . . . . . . . . . . .
65
Trigonometric Equation
Persamaan Trigonometri . . . . . . . . . . . . . . . . . . . .
69
Limit Fungsi . . . . . . . . . . . . . . . . . . . . . . . . . . . . . . .
71
4.2.1
Solution Techniques
Metode Penyelesaian . . . . . . . . . . . . . . . . . . . . . .
4.2.2
4.2.3
75
Limit of Algebraic Function
Limit Fungsi Aljabar . . . . . . . . . . . . . . . . . . . . . .
76
Limit of Trigonometric Function
Limit Fungsi Trigonometri . . . . . . . . . . . . . . . . . . .
5
57
Trigonometric Function
Fungsi Trigonometri . . . . . . . . . . . . . . . . . . . . . . . . . .
4.2
50
79
Geometri
86
5.1
Segitiga . . . . . . . . . . . . . . . . . . . . . . . . . . . . . . . . . .
86
5.2
Segitiga . . . . . . . . . . . . . . . . . . . . . . . . . . . . . . . . . .
87
5.3
Lingkaran . . . . . . . . . . . . . . . . . . . . . . . . . . . . . . . .
96
CONTENTS
6
Kombinatorika
iv
125
6.1
Permutasi dan Kombinasi . . . . . . . . . . . . . . . . . . . . . . . 125
6.2
Prinsip Inklusi-Ekslusi dan Peluang . . . . . . . . . . . . . . . . . 126
6.3
Koefisien Binomial . . . . . . . . . . . . . . . . . . . . . . . . . . . 127
6.4
Prinsip Sarang Merpati . . . . . . . . . . . . . . . . . . . . . . . . . 131
6.5
Paritas . . . . . . . . . . . . . . . . . . . . . . . . . . . . . . . . . . 132
6.6
Relasi Rekurensi . . . . . . . . . . . . . . . . . . . . . . . . . . . . . 133
6.7
Soal-soal dan Pembahasan . . . . . . . . . . . . . . . . . . . . . . . 133
List of Tables
4.1
Trigonometric quadrant system . . . . . . . . . . . . . . . . . . . .
59
4.2
The value of trigonometric functions for special angles . . . . . .
60
4.3
The value of trigonometric functions for any angle (X o − α) . . .
61
4.4
The value of trigonometric functions for negative angles . . . . .
62
4.5
The value of trigonometric functions for any angle (X o + α) . . .
62
v
List of Figures
3.1
Squares in the circles. . . . . . . . . . . . . . . . . . . . . . . . . . .
37
4.1
The right triangle trigonometric system . . . . . . . . . . . . . . .
58
4.2
Triangle and circle of radius R . . . . . . . . . . . . . . . . . . . . .
64
4.3
Sum and Difference of Angles . . . . . . . . . . . . . . . . . . . . .
66
5.1
. . . . . . . . . . . . . . . . . . . . . . . . . . . . . . . . . . . . . .
86
vi
¥¥¥¥¥¥
CHAPTER 1
Mathematics Proof Method
Metode Pembuktian Matematis
In mathematics, a proof is a convincing demonstration (within the accepted standards of a field study) to show that some mathematical statement is necessarily
true. Proofs are obtained from deductive reasoning, rather than from inductive
or empirical arguments. That is, a proof must demonstrate that a statement is
true in all cases, without a single exception.
Dalam matematika, bukti adalah suatu demonstarasi meyakinkan (mengikuti
beberapa standar yang diterima dari suatu bidang kajian tertentu) untuk menunjukkan bahwa pernyataan matematika itu betul-betul benar. Pembuktian
lebih diperoleh dari penarikan kesimpulan secara deduktif dibandingkan dengan penarikan kesimpulan yang secara induktif atau empiris. Dengan demikian,
bukti harus menunjukkan bahwa sebuah pernyataan itu adalah benar disegala
hal tanpa suatu perkecualian sedikitpun.
The statement that is proved is often called a theorem. Once a theorem is proved,
it can be used as the basis to prove further statements. A theorem may also be
referred to as a lemma, that is a sub theorem, especially if it is intended for use
as a stepping stone in the proof of another theorem. An implication of theorems
or lemmas is known as a corollary. An unproved proposition that is believed to
be true is known as a conjecture.
Sebuah pernyataan yang sudah terbuktikan disebut dengan teorema. Sekali teorema itu terbuktikan maka hal ini dapat digunakan sebagai dasar untuk membuktikan pernyataan-pernyataan selanjutnya. Kadangkala teorema disebut juga
1
Chapter 1. Mathematics Proof Method
2
dengan lemma, bagian kecil dari teorema, khusunya jika hal ini digunakan sebagai batu loncatan untuk membuktikan teorema-teorema lainnya. Sebuah akibat
dari beberapa teorema atau lemma disebut dengan korolary. Sebuah pernyataan
yang tidak terbuktikan namun diyakini kebenarannya dikenal dengan istilah
konjektur.
Proofs employ logic but usually include some amount of natural language which
usually admits some ambiguity. In fact, the vast majority of proofs in written mathematics can be considered as applications of rigorous informal logic.
Purely formal proofs, written in symbolic language instead of natural language,
are considered in proof theory. The distinction between formal and informal
proofs has led to much examination of current and historical mathematical practice, quasi-empiricism in mathematics, and so-called folk mathematics (in both
senses of that term). Therefore, the philosophy of mathematics is concerned with
the role of language and logic in proofs, and mathematics as a language.
Beberapa pembuktian matematika menggunakan logika, namun biasanya juga
melibatkan beberapa bahasa biasa yang kadangkala memunculkan dua arti. Namun demikian fakta menunjukkan bahwa hampir semua pembuktian dalam
pernyataan matematika dapat dikatakan sebagai suatu aplikasi dari logika matematika informal. Dalam pembuktian formal yang asli, penulisan dengan simbolsimbol matematika dibandingkan dengan penulisan dengan bahasa biasa lebih
dipilih dalam teori pembuktian. Perbedaan antara pembuktian formal dan informal telah mendasari beberapa evaluasi matematika dan sejarah latihan matematika akhir-akhir ini, termasuk juga matematika semi empiris, sehingga kita
mempunyai istilah yang dikenal dengan matematika untuk umum (terhadap
kedua istilah itu). Dengan demikian, filosofi matematika sesungguhnya adalah
terkait dengan bagaimana perananan bahasa dan logika dalam matematika itu
sendiri, sehingga matematika menjadi suatu bahasa.
Chapter 1. Mathematics Proof Method
3
Mathematics statement can be either true or false. A statement which is always
true is called a tautology, a statement which is always false is called a contradiction. To prove a truth of mathematics statement, we need a proof technique. Basically, there are two types of proof technique, namely direct proof and indirect
proof. In the following, we describe how the two techniques are implemented
in proving the truth of a statement.
Pernyataan matematika dapat bernilai benar atau salah. Suatu pernyataan yang
selalu bernilai benar disebut tautologi, sedangkan pernyataan yang selalu bernilai salah disebut kontradiksi. Untuk membuktikan kebenaran suatu pernyataan
matematika dibutuhkan suatu metode pembuktian. Pada prinsipnya terdapat
dua metode pembuktian, yaitu bukti langsung dan bukti tak langsung. Berikut
ini akan dijelaskan bagaimana kedua metode itu diterapkan untuk membuktikan kebenaran suatu pernyataan.
1.1
Direct Proof
Bukti Langsung
In this case, to prove a truth of mathematics statement is utilized a direct way
with a particular technique in direction of having a conclusion. In general, there
are three direct proofs, namely one way proof (implication), two ways proof
(biimplication/equivalence) and mathematics induction. Some examples of the
use of those methods are presented in the following.
Dalam hal ini, pembuktian dalam kebenaran matematika dibuktikan dengan
cara langsung dengan teknik-teknik tertentu sampai mencapai kesimpulan. Secara umum, terdapat tiga pembuktian langsung yaitu pembuktian satu arah
(implikasi), pembuktian dua arah (biimplikasi/ekuivalensi) dan induksi matematika.
Beberapa contoh penggunaan metode tersebut adalah sebagai berikut.
4
Chapter 1. Mathematics Proof Method
Lemma 1.1.1 If n is a natural number then 1 + 2 + 3 + · · · + n = n2 (n + 1) for any n.
Lema 1.1.1 Jika n suatu bilangan asli maka 1 + 2 + 3 + · · · + n =
setiap n.
n
(n
2
+ 1) untuk
Proof. Implication Proof: Suppose Un = n2 =⇒ Un+1 = (n + 1)2 = n2 + 2n +
1 =⇒ Un+1 − Un = 2n + 1. Then we have
Bukti. Pembuktian Implikasi: Misal Un = n2 =⇒ Un+1 = (n + 1)2 = n2 + 2n +
1 =⇒ Un+1 − Un = 2n + 1. Maka didapat
Un+1 − Un = 2n + 1
Un − Un−1 = 2(n − 1) + 1
..
.
U3 − U2 = 2 · 2 + 1
U2 − U1 = 2 · 1 + 1.
Sum up the n equations above, we get
Jumlahkan seluruh n persamaan di atas maka akan didapat
Un+1 − U1 = 2(1 + 2 + · · · + n) + (1 + 1 + · · · + 1)
n2 + n
1 + 2 + ··· + n =
2
Therefore
Dengan demikian 1 + 2 + · · · + n = n2 (n + 1).
2
Lemma 1.1.2 Let a, b be two integer numbers and n be a positif integer. For any integer
n, n|a and n|b will give the same remainder if and only if n|(a − b).
Lema 1.1.2 Diberikan dua bilangan bulat a, b dan bilangan bulat positip n. Untuk
sebarang bilangan bulat n, na dan nb akan mempunyai sisa yang sama jika dan hanya
jika n|(a − b).
Chapter 1. Mathematics Proof Method
5
Proof. Equivalence Proof:
Bukti. Pembuktian Ekuivalensi:
(=⇒)
Let s be a remainder of a and b divided by n, we have a = kn + s and b = jn + s
for 0 ≤ s ≤ n dan k, j ∈ I.
Misal sisa pembagian bilangan a dan b oleh n adalah s maka a = kn + s dan
b = jn + s, dengan 0 ≤ s ≤ n dan k, j ∈ I.
a − b = (kn + s) − (jn + s)
= (kn − jn)
= (k − j)n.
Since k, j ∈ I, k − j = p where p is also integer, and a − b = pn, it shows that
n|(a − b).
Karena k, j ∈ I maka k−j = p dimana p juga bilangan bulat, sehingga a−b = pn,
yang artinya n|(a − b).
(⇐=)
Suppose that n|(a − b). We will prove that a and b will give the same remainder
when they are divided by n. Let a = kn + s1 and b = jn + s2 for 0 ≤ s1 ≤ n and
0 ≤ s2 ≤ n, we will show that s1 = s2 .
Misal n|(a − b). Akan dibuktikan bahwa a dan b akan mempunyai sisa yang
sama bila dibagi n. Misal a = kn + s1 dan b = jn + s2 untuk 0 ≤ s1 ≤ n dan
0 ≤ s2 ≤ n, maka akan ditunjukkan s1 = s2 .
a − b = pn
a = b + pn
= (jn + s2 ) + pn
= (j + p)n + s2
= kn + s2 .
Chapter 1. Mathematics Proof Method
6
Since the remainder of n|a is single, we have s2 = s1
Karena sisa dari n|a adalah tunggal maka s2 = s1 .
2
Lemma 1.1.3 Prove that 3|(22n − 1) for n ≥ 1.
Lema 1.1.3 Buktikan bahwa 3|(22n − 1) untuk n ≥ 1.
Proof. Mathematics Induction. For n = 1 −→ 3|(22 −1) −→ 3|3 (true). Suppose
it is true for n = k, we have 3|(22k − 1). Thus, is that true for n = k + 1?
Bukti. Induksi Matematika. Untuk n = 1 −→ 3|(22 − 1) −→ 3|3 (benar). Misal
benar untuk n = k maka 3|(22k − 1). Selanjutnya apakah benar untuk n = k + 1?
3|(22(k+1) − 1) ?
3|(22k+2 − 1)
3|(22k · 22 − 1)
3|(22k · 22 − 22 + 3)
3|(22 (22k − 1) + 3).
Since 3|22 (22k − 1) and 3|3. It follows that 3|(22 (22k − 1) + 3), hence it is also true
for n = k + 1.
Karena 3|22 (22k − 1) dan 3|3 maka 3|(22 (22k − 1) + 3). Sehingga hal ini juga benar
untuk n = k + 1.
1.2
2
Indirect Proof
Bukti Tak Langsung
In this proof technique, we do not start involving the existing facts in direction of having a conclusion. We start the prof even from the opposite facts. In
Chapter 1. Mathematics Proof Method
7
general, there are two indirect proofs, namely contradictive proof and contrapositive proof. Contradictive proof is also called reductio ad absurdum proof.
For instance, we will proof that A is true, we start the proof by assuming that A
is not true. Contrapositive proof is obtained by finding a contraposition of an
implication statement in logic math. For instance, we will proof that p −→ q is
true, we start the proof by determining the contraposition of p −→ (q ∨ r), i.e.
∼ (q ∨ r) −→∼ p. As we know, in logic math p −→ (q ∨ r) =∼ (q ∨ r) −→∼ p. In
the following, we present how to use these methods.
Dalam teknik pembuktian ini, fakta-fakta yang ada tidak digunakan secara langsung untuk menuju pada kesimpulan. Pembuktia dimulai justru dari hal sebaliknya. Secara umum terdapat dua pembuktian tak langsung, yakni pembuktian kontradiksi dan pembuktian kontraposisi. Pembuktian kontradiksi disebut juga
pembuktian kemustahilan. Misal yang akan dibuktikan adalah benarnya pernyataan A, maka pembuktian dimulai dengan mengandaikan bahwa A adalah
salah. Pembuktian kontraposisi diperoleh dari menentukan kontraposisi dari
sebuah pernyataan implikasi dalam logika matematika. Misal akan dibuktikan
bahwa pernyataan p =⇒ (q∨r) benar, maka pembuktian diawali dengan menentukan kontraposisi dari p =⇒ (q ∨ r), yaitu ∼ (q ∨ r) =⇒∼ p. Seperti yang kita
ketahui, dalam logika matematika p =⇒ (q ∨ r) =∼ (q ∨ r) =⇒∼ p. Berikut ini
kita jelaskan bagaimana menggunakan metode pembuktian ini.
Lemma 1.2.1 Prove that
√
2 is an irrational number.
Lema 1.2.1 Buktikan bahwa
√
2 adalah suatu bilangan irasional.
√
Proof. Contradictive Proof. Suppose that 2 is a rational number. We will
√
have 2 = ab , where ab is a simplified form. By squaring the two sides, we
obtain a2 = 2b2 . It follows that a2 is even which implies that a is even. Suppose
a = 2k −→, we have (2k)2 = 2b2 −→ b2 = 2k 2 which implies that b is also even.
8
Chapter 1. Mathematics Proof Method
Since both a and b are even,
a
b
is not a simplified form any more, which is a
contradiction.
Bukti. Pembuktian kontradiksi. Misal
a
b
(dimana
a
adalah
b
2
2
√
2 adalah bilangan rasional maka
√
2=
bentuk yang paling sederhana). Kuadratkan kedua ruasnya
diperoleh a = 2b , sehingga a2 adalah bilangan genap dan a pasti genap. Misal
a = 2k −→ (2k)2 = 2b2 −→ b2 = 2k 2 yang berakibat b juga genap. Bila a dan b
adalah sama-sama genap maka
a
b
bukan bentuk yang paling sederhana lagi, ini
jelas kontradiktif.
2
Lemma 1.2.2 Prove that if m + n ≥ 73 then m ≥ 37 or n ≥ 37, for m, n ∈ I.
Lema 1.2.2 Buktikan bahwa jika m + n ≥ 73 maka m ≥ 37 atau n ≥ 37, untuk
m, n ∈ I.
Proof. Contrapositive Proof. Consider the statement as a logic expression p =⇒
(q ∨ r), where p ≡ m + n ≥ 73, q ≡ m ≥ 37 and r ≡ n ≥ 37. The contraposition
of the statement is ∼ (q ∨ r) =⇒∼ p or (∼ q∧ ∼ r) =⇒∼ p. Therefore, to prove
the truth of the statement above, we can start proving that if m < 37 and n < 37
then m + n < 73. Suppose the two any numbers are m ≤ 36 dan n ≤ 36, we have
m + n ≤ 36 + 36 −→ m + n ≤ 72 −→ m + n < 73. It completes the prove that if
m + n ≥ 73 then m ≥ 37 or n ≥ 37, for m, n ∈ I.
Bukti. Pembuktian kontraposisi. Misal pernyataan tersebut disajikan dalam
p =⇒ (q ∨ r), dimana p ≡ m + n ≥ 73, q ≡ m ≥ 37 dan r ≡ n ≥ 37. Kontraposisi
pernyataan tersebut adalah ∼ (q ∨ r) =⇒∼ p atau (∼ q∧ ∼ r) =⇒∼ p. Sehingga
untuk membuktikan kebenaran pernyataan di atas dapat dimulai dengan membuktikan bahwa jika m < 37 dan n < 37 maka m + n < 73. Misal sebarang dua
bilangan itu adalah m ≤ 36 dan n ≤ 36, maka m + n ≤ 36 + 36 −→ m + n ≤
72 −→ m + n < 73, terbukti. Sehingga terbukti pulalah kebenaran pernyataan
semula.
2
9
Chapter 1. Mathematics Proof Method
P ROBLEMS
AND
S OLUTIONS
S OAL - SOAL DAN P EMBAHASAN
1. Prove that (an + b)m = bm mod n.
Buktikan bahwa (an + b)m = bm mod n.
Solution. Direct Proof. To prove (an + b)m = bm mod n, we need to show
that there exists an integer k such that (an + b)m − bm = kn.
Solusi. Bukti Langsung. Untuk membuktikan (an + b)m = bm mod n
perlu ditunjukkan bahwa terdapat bilangan bulat k sedemikian hingga
(an + b)m − bm = kn.
µ
m
m
(an + b) − b
=
¶
m
m−1
(an) + m(an)
m−1
b + ... + m(an)b
m
+b
− bn
= (an)m + m(an)m−1 b + ... + m(an)bm−1 + bm − bm
= (an)m + m(an)m−1 b + ... + m(an)bm−1
µ
¶
m m−1
m−1 m−1
m−1 m−1
=
(a) n
+ m(a)
n
b + ... + m(a)b
b
n
µ
Let k =
m m−1
(a) n
m−1 m−1
+ m(a)
n
m−1 m−1
b + ... + m(a)b
b
¶
, hence we get
(an + b)m − bm = kn. Therefore (an + b)m = bm mod n.
µ
¶
m m−1
m−1 m−1
m−1 m−1
Bila k = (a) n
+ m(a)
n
b + ... + m(a)b
b
, maka didapat
(an + b)m − bm = kn. Sehingga (an + b)m = bm mod n.
2. If p is a prime number and p|a1 a2 ...an then p|ai for any 1 ≤ i ≤ n.
Jika p adalah bilangan prima dan p|a1 a2 ...an maka p|ai untuk sebarang 1 ≤
i ≤ n.
Solution. Mathematics Induction. Let P (k) be a representation statement. Step I: If p|a1 , then it is obvious that P |ai for 1 ≤ i ≤ 1. Step II:
Chapter 1. Mathematics Proof Method
10
Assume it is true for P (k), that is if p|a1 a2 ...ak then p|ai for any 1 ≤ i ≤ k.
We will show that P (k+1) is also true. Since p|a1 a2 ...ak ak+1 and p is a prime
number, it holds p|a1 a2 ...ak or p|a(k+1) . From the two possibilities, we have
p|ai for 1 ≤ i ≤ k + 1 as P (k) is true. It concludes that P (k + 1) is also true.
Solusi. Induksi Matematika. Misal P (k) adalah representasi dari pernyataan tersebut. Langkah I : Jika p|a1 , jelas bahwa P |ai untuk suatu 1 ≤
i ≤ 1. Langkah II : Misalkan p(k) benar, artinya jika p|a1 a2 ...ak maka p|ai
untuk suatu 1 ≤ i ≤ k. Akan dibuktikan bahwa P (k + 1) benar. Diketahui p|a1 a2 ...ak ak+1 , maka karena p bilangan prima berlaku p|a1 a2 ...ak atau
P |a(k+1) . Dari kedua kemungkinan ini dikombinasikan, karena P (k) bernilai benar, maka didapatkan untuk suatu 1 ≤ i ≤ k + 1. Jadi terbukti bahwa
P (k + 1) bernilai benar.
3. Prove that if xm is divisible by a prime p, then x is also divisible by p.
Bukatikan bahwa jika xm habis dibagi bilangan prima p, maka x habis
dibagi p.
Solution. Since p divides xm , we have p|xm or p|x.xm−1 . Since p is a prime,
we have p|x or p|xm−1 . Suppose p does not divide x, it follows p|xm−1 or
p|x.xm−2 . Repeat the process, we will be able to show that p|x.
Solusi. Karena p membagi habis xm , didapat p|xm atau p|x.xm−1 . Karena
p adalah bilangan prima maka p|x atau p|xm−1 . Andaikan p tidak habis
membagi x, maka p|xm−1 atau p|x.xm−2 . Jika proses diteruskan maka akan
didapatkan bahwa p|x.
¥¥¥¥¥¥
CHAPTER 2
Number Theory
Teori Bilangan
2.1
Divisibility
Keterbagian
When 13 is divided by 5, it will give quotient 2 and remainder 3, denoted by
13
5
= 2+
3
5
or 13 = 2 × 5 + 3. In general, for any positive integers a and b there
exists a unique pair (q, r) of nonnegative integers such that b = q × a + r and
0 < r < a. We say that q is the quotient and r is the remainder when b is divided
by a. If r = 0 then we say that b is divisible by a or a divides b, denoted by a| b.
If b is not divisible by a then we denote as a - b.
Jika 13 dibagi 5 maka hasil baginya 2 dan sisanya 3 dan ditulis:
13
5
= 2+
3
5
atau
13 = 2×5+3. Secara umum, apabila a bilangan bulat dan b bilangan bulat positif,
maka ada tepat satu bilangan bulat q dan r sedemikian hingga b = q × a + r dan
0 < r < a. Dalam hal ini, q disebut hasil bagi dan r adalah sisa pembagian bila b
dibagi a. Jika r = 0 maka dikatakan b habis dibagi a dan ditulis a| b. Bila b tidak
habis dibagi a maka ditulis a - b.
Lemma 2.1.1 (1) If a|b then a|bc for any c ∈ I; (2) If a| b and b| c then a| c; (3) If ab| c
then a| c and b| c; (4) If a|b and b|a then a = ±b; (5) If a| b and a| c then a| (bx + cy) for
any integers x and y.
11
Chapter 2. Number Theory
12
Lema 2.1.1 (1) Jika a|b maka a|bc untuk sebarang c ∈ I; (2) jika a| b dan b| c maka a| c;
(3) jika ab| c maka a| c dan b| c; (4) jika a|b dan b|a maka a = ±b; (5) jika a| b dan a| c
maka a| (bx + cy) untuk sebarang bilangan bulat x dan y.
Proof. Property (1): If a| b then b = ka, and if b| c then c = lb = l(ka) = (kl)a.
It implies that a| c. Property (3): If a| b then b = ka −→ bx = kxa, if a| c then
c = la −→ cy = kya. We have bx + cy = (kx + ly)a, therefore a| (bx + cy).
Bukti. Sifat (1): a| b maka b = ka, dan b| c maka c = lb = l(ka) = (kl)a maka
a| c. Sifat (3): a| b maka b = ka −→ bx = kxa, dan a| c maka c = la −→ cy = kya.
Kemudian bx + cy = (kx + ly)a maka a| (bx + cy).
2
Lemma 2.1.2 A number a is divisible by 2n if the last n digit of the number is divisible
by 2n .
Lema 2.1.2 Suatu bilangan a habis dibagi 2n jika n angka terkhir dari bilangan tersebut
habis dibagi 2n .
Proof. Suppose n = 1, thus 2 divides a if the last digit of the number is divisible by 2. Let a = . . . a3 a2 a1 a0 , we have a = 10(. . . a3 a2 a1 ) + a0 . Since
2| 10(. . . a3 a2 a1 ), a0 must be divisible by 2 to have 2|a.
Bukti. Misal n = 1, berarti a habis dibagi 2 jika angka terakhir dari bilangan
tersebut habis dibadi 2. Misal a = . . . a3 , a2 a1 a0 maka a = 10(. . . a3 , a2 a1 ) + a0 .
Karena 2| 10(. . . a3 , a2 a1 ) a0 harus habis dibagi 2 untuk memperoleh 2|a.
Example. Is 173332 divisible by 8? Since 23 | 332, we have 8| 173332.
Contoh. Apakah 173332 habis dibagi oleh 8? Karena 23 | 332 maka 8| 173332.
2
Chapter 2. Number Theory
13
Example. Is 13 + 23 + · · · + 1003 divisible by 7? False, as we have 13 + 23 + · · · +
1003 = (1 + 2 + 3 + · · · + 100)2 = (5050)2 = 25502500 and 7 - 25502500.
Contoh. Apakah 13 + 23 + · · · + 1003 habis dibagi 7? Tidak, karena 13 + 23 + · · · +
1003 = (1 + 2 + 3 + · · · + 100)2 = (5050)2 = 25502500 dan 7 - 25502500.
Lemma 2.1.3 A number a = an an−1 . . . a1 a0 is respectively divisible by 3,9 and 11 if
the sum of its digits satisfies respectively the following: (an +an−1 +an−2 +· · ·+a1 +a0 )
is divisible by 3; (an + an−1 + an−2 + · · · + a1 + a0 ) is divisible by 9; and (an − an−1 +
an−2 − an−3 + . . . ) is divisible by 11.
Lema 2.1.3 Suatu bilangan a = an an−1 . . . a1 a0 berturut-turut habis dibagi 3,9 dan
11 jika jumlah angka-angkanya memenuhi masing-masing sifat berikut: (an + an−1 +
an−2 + · · · + a1 + a0 ) habis dibagi 3; (an + an−1 + an−2 + · · · + a1 + a0 ) habis dibagi 9;
dan (an − an−1 + an−2 − an−3 + . . . ) habis dibagi 11.
Proof.
Bukti.
a = an an−1 . . . a1 a0
= an × 10n + an−1 × 10n−1 + . . . a1 × 101 + a0 × 100
= an × (9 + 1)n + an−1 × (9 + 1)n−1 + . . . a1 × (9 + 1)1 + a0 × (9 + 1)0
= an [9n + n · 9n−1 + · · · + 9n] + an + an−1 [9n−1 + (n − 1) · 9n−2 +
· · · + 9(n − 1)] + an−1 + · · · + 9a1 + a1 + a0
= an [9n + n · 9n−1 + · · · + 9n] + an−1 [9n−1 + (n − 1) · 9n−2 + · · · +
9(n − 1)] + · · · + 9a1 + an + an−1 + · · · + a1 + a0
= K(a) + Q(a)
Since (3 ∧ 9)| K(a), it must be (3 ∧ 9)| Q(a) to have (3 ∧ 9)| a .
Karena (3 ∧ 9)| K(a) maka haruslah (3 ∧ 9)| Q(a) supaya (3 ∧ 9)| a .
2
Chapter 2. Number Theory
2.2
14
Special Number
Bilangan Khusus
2.2.1
A Prime and Composite Number
Bilangan Prima dan Komposit
The integer p > 1 is called a prime number if there is no integer d with d > 1
and d 6= p such that d|p. Any integer n > 1 has at least one prime divisor. If n is
a prime, then that prime divisor is n itself. An integer n > 1 that is not a prime
is called composite.
Bilangan bulat p > 1 disebut sebuah bilangan prima jika tidak ada bilangan
bulat d dimana d > 1 dan d 6= p sedemikian hingga d|p. Setiap bilangan bulat
n > 1 mempunyai paling sedikit satu pembagi prima. Jika n adalah bilangan
prima, maka pembagi primanya adalah bilngan n itu sendiri. Sebuah bilangan
bulat n > 1 yang bukan bilngan prima disebut bilangan komposit.
Theorem 2.2.1 Eratosthenes. For any composite n, there exists a prime p such that
√
p| n and p ≤ n. In other words ”If there does not exist a prime p which divides n,
√
where p ≤ n, then n is a prime”.
Teorema 2.2.1 Eratosthenes. Untuk setiap bilangan komposit n ada bilangan prima
√
p sehingga p| n dan p ≤ n. Dengan kata lain ”Jika tidak ada bilangan prima p yang
√
dapat membagi n dengan p ≤ n maka n adalah bilangan prima”.
Example. Are 157 and 221 prime numbers?. The primes which are less than
√
157 are 2, 3, 5, 7, 11. Since none of them divides 157, then 157 is a prime. The
√
primes which are less than 221 are 2, 3, 5, 7, 11, 13. Since 13| 221 then 221 is a
composite number.
Chapter 2. Number Theory
15
Contoh. Apakah bilangan 157 dan 221 bilangan prima?. Bilangan-bilangan
√
prima yang lebih kecil dari 157 adalah 2, 3, 5, 7, 11. Karena tidak ada satupun
dari bilangan- bilangan prima itu yang dapat membagi 157, maka 157 merupakan bilangan prima. Kemudian bilangan - bilangan prima yang lebih kecil
√
dari 221 adalah 2, 3, 5, 7, 11, 13. Karena 13| 221 maka 221 merupakan bilangan
komposit.
2.2.2
Perfect Square
Bilangan Kuadrat Sempurna
Any perfect square satisfies the following three properties:
• The possible ones of perfect square number is either 0, 1, 4, 5, 6, dan 9.
• If 4 divides a perfect square then the remainder is either 0 or 1.
• If p is a prime and p| x2 then p| z, where z = x2 /p.
Sebarang bilangan kuadrat sempurna memenuhi tiga sifat berikut ini:
• Angka satuan yang mungkin untuk bilangan kuadrat sempurna adalah 0,
1, 4, 5, 6, atau 9.
• Jika 4 membagi bilangan kuadrat sempurna maka sisanya 0 atau 1.
• Jika p bilangan prima dan p| x2 maka p| z, dimana z = x2 /p.
Example. Obtain a perfect square whose digits are k, k + 1, k + 2, 3k, k + 3.
Contoh. Carilah suatu bilangan kuadrat sempurna yang angka-angkanya berturutturut adalah k, k + 1, k + 2, 3k, k + 3.
16
Chapter 2. Number Theory
Solution. The ones of the number is k +3, it follows that k can be either 1, 2, 3 or
6. Whilst the tens is 3k, it follows that k can be either 0, 1, 2 or 3. They imply that
the possible k is either 1, 2 or 3 which give perfect square numbers 12334, 23465
or 34596. Since the remainder of 12334 divided by 4 is 2, it gives that 12334 is
not a perfect square. The remainder of 23465 divided by 4 is 1 and 5|23465, but
5 - 4693, so that 23465 is not a perfect square. Now, 4|34596, and we have the
following
Solusi. Angka satuan bilangan kuadrat ini adalah k + 3 sehingga k yang mungkin adalah 1, 2, 3, 6. Sedangkan angka puluhannya adalah 3k maka k yang mungkin adalah 0, 1, 2, 3. Dari kedua kemungkinan ini diperoleh k yang mungkin
adalah 1, 2, 3, dengan demikian bilangan kuadrat yang mungkin adalah 12334,
23465, 34596. Karena 12334 dibagi 4 bersisa 2 maka 12334 bukan bilangan kuadrat.
Bilangan 23465 dibagi 4 bersisa 1 dan 5|23465, akan tetapi 5 - 4693 maka 23465
bukan bilangan kuadrat. Sekarang, bilangan 4|34596, dan berikut ini berlaku
2 | 34596
2 | 17298
3 | 8649
3 | 2883
31 | 961
31 | 31
Therefore, 34596 = 22 · 32 · 312 = 1862 which is a perfect square.
Sehingga 34596 = 22 · 32 · 312 = 1862 yang merupakan bilangan kuadrat.
17
Chapter 2. Number Theory
2.3
GCD dan Algoritma Euclid
Let a, b be any integers. An integer d satisfying d| a and d| b is called a common
divisor of a and b. The biggest (respectively, smallest) value of d is called Greater
Common Divisor/GCD, denoted by GCD(a, b)) (respectively, Least Common Multiple/LCM, denoted by LCM (a, b))). Several techniques have been developed to
obtain GCD or LCM, one of them is Euclidean algorithm.
Misal a dan b sembarang bilangan bulat. Bilangan bulat d yang memenuhi sifat
d| a dan d| b disebut faktor persekutuan dari a dan b. Nilai terbesar dari d disebut
faktor persekutuan terbesar atau FPB dan ditulis dengan F P B(a, b) sedangkan
nilai terkecil dari d disebut kelipatan persekutuan terkecil atau KPK dan ditulis
KP K(a, b). Beberapa tekni telah dikembangkan untuk menentukan FPB dan
KPK, salah satunya adalah algoritma Euclid.
Algorithm 2.3.1 Euclidean Algorithm. Given two integer numbers a and b with a >
b > 0, GCD(a, b) can be obtained by repeating the following algorithm:
a = q1 b + r1 ;
0 < r1 < b
b = q2 r1 + r2 ;
0 < r 2 < r1
r1 = q3 r2 + r3 ;
..
.
0 < r 3 < r2
rn−2 = qn rn−1 + rn ;
0 < rn < rn−1
rn−1 = qn+1 rn + 0
If rn is the last divisor of the division process which gives a remainder 0 then rn is
GCD(a, b).
Algoritma 2.3.1 Algoritma Euclide. Diberikan dua bilangan bulat a dan b dengan
18
Chapter 2. Number Theory
a > b > 0, maka GCD(a, b) bisa dicari dengan mengulang algoritma berikut:
a = q1 b + r1 ;
0 < r1 < b
b = q2 r1 + r2 ;
0 < r 2 < r1
r1 = q3 r2 + r3 ;
..
.
0 < r 3 < r2
rn−2 = qn rn−1 + rn ;
0 < rn < rn−1
rn−1 = qn+1 rn + 0
Jika rn merupakan pembagi terakhir dari pembagian di atas yang memberikan sisa 0
maka rn merupakan GCD(a, b).
Example. Determine GCD(4840, 1512). Using Euclidean Algorithm, the solution is the following:
Contoh. Tentukan GCD(4840, 1512). Dengan menggunakan Algoritma Euclid
maka solusinya adalah sebagai berikut:
4840 = 3 × 1512 + 304
1512 = 4 × 304 + 296
304 = 1 × 296 + 8
296 = 37 × 8 + 0
Therefore
Jadi GCD(4840, 1512) = 8.
Lemma 2.3.1 Any a|c and b|c imply ab|c if and only if GCD(a, b) = 1.
Lema 2.3.1 Sebarang a|c dan b|c berakibat ab|c jika dan hanya jika GCD(a, b) = 1.
Chapter 2. Number Theory
19
Example. 3|30 and 5|30 imply 3 × 5|30 as GCD(3, 5) = 1, however 2|30 and 6|30
imply 2 × 6 - 30 as GCD(2, 6) 6= 1.
Contoh. 3|30 dan 5|30 maka 3 × 5|30 karena GCD(3, 5) = 1, akan tetapi 2|30 dan
6|30 maka 2 × 6 - 30 karena GCD(2, 6) 6= 1.
2.4
Modular Arithmetic
Modulo Aritmatik
Let a, b, and m be integers, with m > 1. We say that a and b are congruent
modulo m, denoted by a ≡ b (mod m), if the remainder of a divided by m and b
divided by m are the same.
Diberikan bilangan bulat a, b dan m dimana m > 1. Bilangan a dikatakan kongruen dengan b modulo m, dituliskan dengan a ≡ b (mod m), jika sisa pembagian a oleh m dan b oleh m memberikan sisa yang sama.
Lemma 2.4.1 If a and b are congruent modulo m then m| (a − b).
Lema 2.4.1 Jika a dan b kongruen modulo m maka m| (a − b).
Proof. a ≡ b (mod m) =⇒ a = q1 m + r and b = q2 m + r. Hence a − b = (q1 − q2 )m,
it follows m| (a − b).
Bukti. a ≡ b (mod m) =⇒ a = q1 m + r dan b = q2 m + r. Kemudian a − b =
(q1 − q2 )m sehingga m| (a − b).
2
Chapter 2. Number Theory
20
Lemma 2.4.2 (1) a ≡ b (mod m), b ≡ a (mod m) are equivalent with a−b ≡ 0 (mod m);
(2) if a ≡ b (mod m) and b ≡ c (mod m) then a ≡ c (mod m); (3) if a ≡ b (mod m)
and d|m then a ≡ b (mod d); (4) if a ≡ b (mod m) and c ≡ d (mod m) then ax + cy ≡
bx + dy (mod m) and ac ≡ bd (mod m), for any x, y ∈ I.
Lema 2.4.2 (1) a ≡ b (mod m), b ≡ a (mod m) adalah setara dengan a−b ≡ 0 (mod m)
; (2) jika a ≡ b (mod m) dan b ≡ c (mod m) maka a ≡ c (mod m); (3) jika a ≡
b (mod m) dan d|m maka a ≡ b (mod d); (4) jika a ≡ b (mod m) dan c ≡ d (mod m)
maka ax + cy ≡ bx + dy (mod m) dan ac ≡ bd (mod m), untuk sebarang x, y ∈ I.
Proof. a ≡ b (mod m) −→ m|(a − b) and c ≡ d (mod m) −→ m|(c − d). From
Lemma 2.1.1, we have m|((a−b)x+(c−d)y) or m|((ax+cy)−(bx+dy)), therefore
ax + cy ≡ bx + dy (mod m).
Bukti. a ≡ b (mod m) −→ m|(a − b) dan c ≡ d (mod m) −→ m|(c − d). Sesuai
Lema 2.1.1 didapat m|((a − b)x + (c − d)y) atau m|((ax + cy) − (bx + dy)), sehingga
ax + cy ≡ bx + dy (mod m).
2
It follows from Lemma 2.4.2, if f (x) is a polynomial of integer coefficients and
a ≡ b (mod m) then f (a) ≡ f (b) (mod m).
Akibat dari Lemma 2.4.2, jika f (x) adalah suatu fungsi polinom dengan koefisienkoefisien bilangan bulat dan a ≡ b (mod m) maka f (a) ≡ f (b) (mod m).
Example. Prove that for any natural n, S = 2903n −803n +261n −464n is divisible
by 7 and 271. Furthermore, prove that 1897|S.
Contoh. Buktikan bahwa untuk setiap bilangan asli n maka S = 2903n − 803n +
261n − 464n habis dibagi 7 dan 271. Buktikan juga bahwa 1897|S.
Solution. Since 2903 ≡ 803 (mod 7) and 464 ≡ 261 (mod 7), as well as 2903 ≡
464 (mod 271) and 803 ≡ 261 (mod 271), from Lemma 2.4.2(4) we must have 7|S
Chapter 2. Number Theory
21
and 271|S. Furthermore, since 1897 = 7 × 271 and GCD(7, 271) = 1, it follows
from Lemma 2.3.1 that 1897|S.
Solusi. Karena 2903 ≡ 803 (mod 7) dan 464 ≡ 261 (mod 7), demikian juga 2903 ≡
464 (mod 271) dan 803 ≡ 261 (mod 271) maka sesuai dengan Lema 2.4.2(4) dipastikan bahwa 7|S dan 271|S. Selanjutnya karena 1897 = 7×271 dan GCD(7, 271) =
1, maka sesuai Lema 2.3.1 terbukti 1897|S.
Lemma 2.4.3 (am + b)n ≡ bn (mod m)
Lema 2.4.3 (am + b)n ≡ bn (mod m)
Proof. The proof is the same with showing that there exists an integer k such
that (am + b)n − bn = km.
Bukti. Pembuktian ini sama artinya dengan membuktikan ada bilangan bulat
k sehingga (am + b)n − bn = km.
(am + b)n − bn = (am)n + n(am)n−1 · b + · · · + n(am)bn−1 + bn − bn
= {a(am)n−1 + an(am)n−2 + · · · + an (b)n−1 }m
= km.
2
Example. Determine the ones of 19971991 .
Contoh. Tentukan angka satuan bilangan 19971991 .
Solution. From Lemma 2.4.3, the solution is as follows:
22
Chapter 2. Number Theory
Solusi. Dengan menggunakan Lema 2.4.3 maka solusinya adalah sebagai berikut:
The ones (Angka satuan) 19971991 = The remainder (Sisa bagi) 19971991 oleh 10
= (199 × 10 + 7)1991 (mod 10)
= 71991 (mod 10)
= 74×497+3 (mod 10)
= (74 )497 × 73 (mod 10)
= (2401)497 × 343 (mod 10)
= (240 × 10 + 1)497 × (34 × 10 + 3) (mod 10)
= 1 × 3 (mod 10)
= 3 (mod 10)
Thereforre, the ones of 19971991 is 3.
Sehingga, angka satuan bilangan 19971991 adalah 3.
It is easy to see that powering numbers 0, 1, 2, . . . , 9 give the following ones/units:
0 => 0 → 1 type, 1 => 1 → 1 type, 2 => 2, 4, 8, 6 → 4 types, 3 => 3, 9, 7, 1 → 4
types, 4 => 4, 6 → 2 types, 5 => 5 → 1 type, 6 => 6 → 1 type, 7 => 7, 9, 3, 1 →
4 types, 8 => 8, 4, 2, 6 → 4 types, 9 => 9, 1 → 2 types.
Mudah dicermati bahwa perpangkatan bilangan 0, 1, 2, . . . , 9 akan menghasilkan
angka satuan berikut: 0 => 0 → 1 jenis, 1 => 1 → 1 jenis, 2 => 2, 4, 8, 6 → 4
jenis, 3 = 3, 9, 7, 1 → 4 jenis, 4 => 4, 6 → 2 jenis, 5 => 5 → 1 jenis, 6 => 6 → 1
jenis, 7 => 7, 9, 3, 1 → 4 jenis, 8 => 8, 4, 2, 6 → 4 jenis, 9 => 9, 1 → 2 jenis.
Therefore, finding the units of exponential number can be obtained by the following ways: Consider the example above. Determine the ones of 19971991 . Since
the ones of the base is 7, the ones of the exponential number contains 4 types,
namely 7,9,3,1. Thus 1991/4 = 4 × 497 + 3, it implies that 19971991 = 19974×497+3
which follows the ones of 73 → 3. Therefore the ones of 19971991 is 3.
Chapter 2. Number Theory
23
Dengan demikian pencarian angka satuan dari perpangkatan bilangan dapat
dilakukan dengan cara berikut: Misal pada contoh soal di atas. Berapa angka
satuan bilangan 19971991 . Karena angka satuan bilangan dasar adalah angka 7
maka angka satuan perpangkatan bilangan ini akan meliputi 4 jenis yaitu 7,9,3,1.
Selanjutnya 1991/4 = 4 × 497 + 3 sehingga 19971991 = 19974×497+3 yang berakibat
angka satuan dari 73 → 3. Dengan demikian angka satuan bilangan 19971991
adalah 3.
Example. Determine the ones of 345678292784383951 .
Contoh. Tentukan angka satuan bilangan 345678292784383951 .
Solution. Since the ones of the base is 4, the ones of the exponential number
contains 2 types, namely 4,6. Thus 383951/2 = 2 × 191975 + 1, it implies that
345678292784383951 = 3456782927842×191975+1 which follows the ones of 41 → 4.
Therefore the ones of 345678292784383951 is 4.
Solusi. Karena angka satuan bilangan dasarnya adalah 4 maka angka satuan
perpangkatan bilangan ini akan meliputi 2 jenis yaitu 4,6. Selanjutnya tentukan
2|383951 = 2 × 191975 + 1 sehingga 345678292784383951 = 3456782927842×191975+1
yang berakibat angka satuan dari 41 → 4. Dengan demikian angka satuan bilangan 345678292784383951 adalah 4.
2.5
Linear Diophantine Equations
Persamaan Linier Diophantin
An equation of the form ax + by = c where a, b, c are fixed integers and a, b are
all different from zero is called a linear Diophantine equation if the solutions x, y
respect to integers.
24
Chapter 2. Number Theory
Persamaan ax + by = c dengan a, b, c bilangan-bilangan bulat dan a, b duaduanya bukan nol disebut persamaan linier Diophantine jika penyelesaiannya
dicari untuk bilangan-bilangan bulat.
Theorem 2.5.1 Diophantine equation ax+by = c is solvable if and only if GCD(a, b)|c.
Teorema 2.5.1 Persamaan Diophantine ax + by = c mempunyai penyelesaian jika dan
hanya jika GCD(a, b)|c.
Proof. Let d = GCD(a, b) and d| c. We have d| c ⇐⇒ c = kd for any integers k.
Whilst d| GCD(a, b) ⇐⇒ am + bn = d for any m and n such that:
k(am + bn) = kd
a(km) + b(kn) = c,
it implies that x = mk dan y = nk
Bukti. Misal d = GCD(a, b) dan d| c, maka d| c ⇐⇒ c = kd untuk sebarang
bilangan bulat k. Sedangkan d| GCD(a, b) ⇐⇒ am + bn = d untuk sebarang
bilangan bulat m dan n sehingga:
k(am + bn) = kd
a(km) + b(kn) = c,
berarti x = mk dan y = nk
Theorem 2.5.2 If d = GCD(a, b) and x0 , y0 are the solutions of Diophantine equation
ax + by = c, then the general solutions are
b
a
x = x0 + k and y = y0 − k; where k is an integer parameter.
d
d
25
Chapter 2. Number Theory
Teorema 2.5.2 Jika d = GCD(a, b) dan x0 , y0 merupakan penyelesaian persamaan
Diophantine ax + by = c, maka penyelelesaian umum persamaan tersebut adalah :
b
a
x = x0 + k dan y = y0 − k; dengan k parameter bilangan bulat.
d
d
Example. Find the general solutions of Diophantine equation 738x + 621y = 45.
Contoh. Tentukan solusi umum dari persamaan Diophantine 738x + 621y = 45.
Solution. Finding GCD(738, 621) with Euclidean Algorithm
Solusi. Mencari GCD(738, 621) dengan Alogaritma Euclide
738 = 1 × 621 + 117
621 = 5 × 117 + 36
117 = 3 × 36 + 9
36 = 4 × 9 + 0.
we get GCD(738, 621) = 9. Since 9| 45, the equation is solvable. Consider the 9
as a linear combination of 738 and 621.
diperoleh GCD(738, 621) = 9. Karena 9| 45 maka persamaan di atas mempunyai
penyelesaian. Jadikan 9 sebagai kombinasi linear dari 738 dan 621.
9 = 117 − 3 · 36
= 117 − 3(621 − 5 × 117) = −3 × 621 + 16 × 117
= −3 × 621 + 16(738 − 621)
9 = 16 × 738 − 19 × 621
multiplying the two sides by 5, we get 45 = 80 × 738 − 45 × 621. It implies that
x0 = 80, y0 = −95. Therefore the general solutions are:
Chapter 2. Number Theory
26
Kalikan kedua ruas dengan 5, diperoleh 45 = 80 × 738 − 45 × 621, sehingga
didapat x0 = 80, y0 = −95. Dengan demikian penyelesaian umumnya adalah:
621
k = 80 + 69k
9
738
y = −95 −
k = −95 − 82k
9
x = 80 +
P ROBLEMS
AND
S OLUTIONS
S OAL - SOAL DAN P EMBAHASAN
1. A number A is the smallest natural number which is a product of the smallest three of prime number. Determine two numbers between 200 and 300
which gives the same prime factor with A.
Bilangan A adalah bilangan asli terkecil yang merupakan hasil kali dari 3
bilangan prima terkecil. Sebutkan dua buah bilangan di antara 200 dan
300 yang mempunyai faktor prima yang serupa dengan bilangan A.
Solution. A = 2.3.5 = 30, the desired numbers are 24 .3.5 = 240 and
2.32 .5 = 270
Solusi. A = 2.3.5 = 30, jadi bilangan yang dicari adalah 24 .3.5 = 240 dan
2.32 .5 = 270
2. P (n) is a multiplication of the digits of number n, and S(n) is summation
of its digits. Determine the ones of n satisfying P (n) + S(n) = n if n is a
number consisting two digits.
P (n) didefinisikan sebagai perkalian antara angka-angka bilangan n dan
S(n) adalah penjumlahan antara angka-angka bilangan n. Tentukan angka
satuan n yang memenuhi P (n) + S(n) = n jika n adalah bilangan yang
terdiri dari dua angka.
27
Chapter 2. Number Theory
Solution. Assume the number n is ab, we have a, b as a tens and ones.
Solution. Asumsikan bilangan tersebut adalah ab, maka kita mengartikan
a sebagai puluhan dan b sebagai satuan.
P (n) + S(n) = n
a.b + (a + b) = ab
a.b + (a + b) = 10a + b
ab = 9a = a9
Therefore, the ones of the number is 9
Dari sini didapatkan bahwa satuan bilangan n adalah 9
3. What is the remainder of 13 + 23 + 33 + ... + 1003 divided by 7?
Berapakah sisa pembagian 13 + 23 + 33 + ... + 1003 oleh 7?
Proof.
Bukti.
13 + 23 + 33 + ... + 1003 = (1 + 2 + 3 + ... + 100)3
= (5050)2
= (101x50)2 (mod 7)
= (101)2 x(50)2 (mod 7)
= (2x72 + 3)2 x(72 + 1)2 (mod 7)
= 32 x1 (mod 7)
= 9 (mod 7)
= 2 (mod 7)
¥¥¥¥¥¥
CHAPTER 3
Algebra Functions
Fungsi Aljabar
3.1
Polynomials Inequality
Pertidaksamaan Polinomial
In this section, we will show how so solve inequalities. Solving an inequality
means finding all of its solutions. A solution of an inequality is a number which
when substituted for the variable makes the inequality a true statement.
Dalam bagian ini, akan dipelajari bagaimana menyelesaikan pertidaksamaan.
Menyelesaikan pertidaksamaan berarti mencari semua himpunnan penyelesaian. Penyelesaian dari pertidakasamaan adalah sebuah bilangan dimana pada
saat bilangan itu disubstitusikan maka pertidaksamaan menjadi pernyataan yang
benar.
Some properties related to inequality are as follows:
• If a > b then a = b + k, for any k.
• If a > b then a + c > b + c and a.c > b.c where c > 0.
• If a > b and b > c then a > c.
• If a > b and c > d then a + c > b + d.
• If a > b > 0 then 1/a > 1/b, and if a/b > 0 then a.b > 0.
28
Chapter 3. Algebra Functions
29
Beberapa sifat-sifat terkait dengan pertidaksamaan adalah sebagai berikut:
• Jika a > b maka a = b + k, untuk sebarang k.
• Jika a > b maka a + c > b + c dan a.c > b.c dimana c > 0.
• Jika a > b dan b > c maka a > c.
• Jika a > b dan c > d maka a + c > b + d.
• Jika a > b > 0 maka 1/a > 1/b, dan jika a/b > 0 maka a.b > 0.
In general, we present inequality polynomials as:
Secara umum pertidaksamaan fungsi dinyatakan sebagai
a0 xn + a1 xn−1 + · · · + an−1 x + an ≥ · ∨ · ≤ 0
The following steps shows how to solve inequality:.
• factorizing the polynomials, if it is not factorizible then consider whether
the function is definite or not.
• Draw a number line and show the roots of polynomial equality on it.
• Put sign + or − on each interval of the number line respecting to the following rules:
– Consider the coefficient of the highest order of the inequality.
– If the coefficient of the the highest order is positive (respectively, or
negative) then the right side of line number interval is + (respectively,
or −). Hence, place an alternate sign on the remaining interval.
– Consider the sign of the inequality and confirm with the signs on the
interval to conclude the solution set.
Chapter 3. Algebra Functions
30
Lagkah-langkah berikut menunjukkan bagaimana cara menyelesaikan persamaan
ini.
• Lakukan memfaktorkan terhadap fungsi polinomial, bila tidak dapat difaktorkan pertimbangkan apakah fungsi tersebut definit.
• Gambar sebuah garis bilangan dan letakkan semua akar-akar persamaan
polinomial dalam garis bilangan.
• Berikan tanda + atau − pada setiap interval dengan prinsip berikut:
– Lihat koefisien pangkat tinggi pertidaksamaan tersebut.
– Jika koefisien pangkat tertingginya positif maka interval paling kanan
adalah +, bila negatif maka tulis − dan kemudian interval selanjutnya
secara bergantian berlawanan tanda.
– Lihat tanda pertidaksamaannya dan cocokkan dengan tanda tanda
dalam interval untuk menentukan himpunan penyelesaiannya.
Example. Determine x satisfying inequality x2 − 2x − 80 ≤ 0 and
Contoh. Tentukan nilai-nilai x yang memenuhi kedua pertidaksamaan x2 −2x−
80 ≤ 0 dan
(2x − 5)501 (x2 − 3x − 4)(−x2 + 7x − 80)(x3 + 4x2 − x + 1)2000
≥0
(x + 1)(x + 14)51
Solution. To solve this inequality we should consider discriminant D = b2 −4ac
of quadratic function ax2 + bx + c = 0. If D < 0 and a > 0 then the quadratic
function is positive definite, If D < 0 and a < 0 the quadratic function is negative
definite. Furthermore, all number of even power must be positive.
Jika
31
Chapter 3. Algebra Functions
Solusi. Untuk menyelesaikan pertidaksamaan ini maka kita harus ingat konsep Deskriminan D = b2 − 4ac untuk fungsi kuadratik ax2 + bx + c = 0. Bila
D < 0 dan a > 0 maka fungsi kuadratik adalah definit positif, Bila D < 0 dan
a < 0 maka fungsi kuadratik adalah definit negatif. Kemudian setiap bilangan
yang berpangkat genap pasti bernilai positif.
x2 − 2x − 80 ≤ 0
(x − 10)(x + 8) ≤ 0 ⇐⇒ Hp1 = {x| − 8 ≤ x ≤ 10}
Whilst (Sedangkan)
(2x − 5)501 (x2 − 3x − 4)(−x2 + 7x − 80)(x3 + 4x2 − x + 1)2000
≥ 0
(x + 1)(x + 14)51
(2x − 5)501 (x − 4) ª ⊕
≥ 0
(x + 14)51
For (Untuk) x 6= −14, (2x − 5)501 (x − 4)(x + 14)51 ª ⊕ ≥ 0
For (Untuk) x 6= −14, (2x − 5)501 (x − 4)(x + 14)51 ª ≥ 0
|
{z
}
ª
501
(x − 4)(x + 14)51 ≤ 0
5
Therefore (Sehingga) Hp2 = {x| − 14 < x ≤ · ∨ · x ≥ 4}
2
It follows (Berarti) (2x − 5)
Since the problem shows a conjunction relation then both solution sets Hp1 and
Hp2 must be joined. It implies that Hp = {x| − 8 ≤ x ≤
5
2
· ∨ · 4 ≤ x ≤ 10}.
Karena masalah di atas menunjukkan relasi konjungsi, maka kedua himpunan
penyelesaian Hp1 dan Hp2 harus digabung. Sehingga diperoleh Hp = {x| − 8 ≤
x≤
5
2
· ∨ · 4 ≤ x ≤ 10}.
Example. The value of x satisfying inequality (x − 2)4 (−x2 − x − 10)|x2 − x −
1|(−x + 1)2 ≤ 1 is . . .
Contoh. Harga x yang memenuhi pertidaksamaan (x − 2)4 (−x2 − x − 10)|x2 −
x − 1|(−x + 1)2 ≤ 1 adalah . . .
Chapter 3. Algebra Functions
32
(A) x ≤ −1 ∨ 0 ≤ x ≤ 1 ∨ x ≥ 2
(B) − 12 ≤ x ≤ 0 ∨
3
2
≤x≤2
(C) −1 ≤ x ≤ 0 ∨ 1 ≤ x ≤ 2
(D) − 12 < x ≤
3
2
(E) All x (Semua harga x)
3.1.1
Inverse Function
Fungsi Invers
If f is a function from A to B then an inverse function for f is a function in the
opposite direction, from B to A, with the property that a round trip (a composition) from A to B to A (or from B to A to B) returns each element of the initial
set to itself. Thus, if an input x into the function produces an output y, then
inputting y into the inverse function f −1 (read f inverse, not to be confused with
exponentiation) produces the output x. Not every function has an inverse; those
that are called invertible.
Jika f adalah fungsi dari A ke B maka fungsi invers untuk f adalah suatu fungsi
balikan yang berlawanana arah, yaitu dari B ke A, dengan sifat rute balikan
itu (komposisi fungsi) adalah dari A ke B ke A (atau dari B ke A ke B) akan
mengembalikan masing-masing element dari himpunan asal ke elemen-element
itu sendiri. Dengan kata lain, jika dimasukkan x ke dalam fungsi memberikan
hasil y, maka dengan memasukkan nilai y ke dalam fungsi invers f −1 (baca f invers, biar tidak dibingungkan dengan pengertian eksponensial) akan diperoleh
hasil x. Tidak setiap fungsi memiliki balikan, fungsi yang tidak mempunyai
fungsi invers ini disebut invertibel.
33
Chapter 3. Algebra Functions
Theorem 3.1.1 If f (x) is function with x ∈ < then inverse function f −1 (x) satisfies
the following:
Teorema 3.1.1 Jika f (x) suatu fungsi dalam x ∈ < maka fungsi invers f −1 (x) memenuhi
hal berikut:
1. f â—¦ f −1 (x) = x · ∧ · (f â—¦ g)−1 (x) = g −1 â—¦ f −1 (x)
2. f (x) = ax + b =⇒ f −1 (x) =
3. f (x) =
ax+b
cx+d
=⇒ f −1 (x) =
2
4. f (x) = ax + bx + c =⇒
5. f (x) =
√
x−b
a
−dx+b
cx−a
−1
f12
(x)
ax + b =⇒ f −1 (x) =
=
b
− 2a
±
q ¡
1
a
D
4a
¢
x2 −b
a
6. f (x) = ax + b · ∧ · f â—¦ g = px + q =⇒ g(x) =
7. f (x) = abx =⇒ f −1 (x) =
x+
px+q−b
a
a log x
b
Bukti. For no. 4. From (1), we know that f â—¦ f −1 (x) = x, so that for f (x) =
ax2 + bx + c we get
Proof. Untuk nomor 4. Dari (1) dipahami bahwa f â—¦ f −1 (x) = x sehingga untuk
f (x) = ax2 + bx + c diperoleh
f (f −1 ) = x
a(f −1 )2 + b(f −1 ) + c = x
a(f −1 )2 + b(f −1 ) + c − x = 0
−1
f12
b
= − ±
2a
s µ
¶
1
b2 − 4ac
x+
a
4a
2
Chapter 3. Algebra Functions
3.1.2
34
Arithmetic and Geometric Sequence
Barisan Aritmatik dan Geometrik
In mathematics, an arithmetic sequence or arithmetic progression is a sequence
of numbers such that the difference of any two successive members of the sequence is a constant. For instance, the sequence 3, 5, 7, 9, 11, 13, . . . is an arithmetic progression with common difference 2. The sum of the terms of a arithmetic progression is known as a arithmetic series. Thus, the general form of a
arithmetic sequence is a, a + b, a + 2b, . . . , a + (n − 1)b; and that of a geometric
series is (a) + (a + b) + (a + 2b) + · · · + (a + (n − 1)b).
Dalam matematika, suatu barisan aritmatika atau urutan aritmatika didefinisikan sebagai barisan bilangan sedemikian hingga beda dari dua bilangan yang
berurutan dari barisan itu adalah konstan. Sebagai contoh, barisan bilangan
3, 5, 7, 9, 11, 13, . . . adalah barisan aritmatika dengan beda 2. Jumlah suku-suku
dari barisan arimatika disebut dengan deret aritimatika. Bentuk umum dari
barisan aritmatika adalah a, a + b, a + 2b, . . . , a + (n − 1)b; sedangkan bentuk
umum dari deret aritmatika adalah (a) + (a + b) + (a + 2b) + · · · + (a + (n − 1)b).
Meanwhile, A geometric sequence is a sequence of numbers where each term
after the first is found by multiplying the previous one by a fixed non-zero number called the common ratio. For example, the sequence 2, 6, 18, 54,. . . is a
geometric progression with common ratio 3. Similarly 10, 5, 2.5, 1.25, . . . is a
geometric sequence with common ratio 1/2. The sum of the terms of a geometric progression is known as a geometric series. Thus, the general form of
a geometric sequence is a, ar, ar2 , ar3 , ar4 , . . .; and that of a geometric series is
a + ar + ar2 + ar3 + ar4 + . . .
Sementara, suatu barisan geometrik adalah barisan bilangan dimana masingmasing suku setelah bilangan pertama didapat dari mengalikan bilangan se-
Chapter 3. Algebra Functions
35
belumnya dengan bilangan konstan yang tidak nol, yang disebut dengan rasio dan rasio ini selalu sama. Sebagai contoh, barisan bilangan 2, 6, 18, 54,. . .
adalah barisan geometrik dengan rasio 3. Sama hanya dengan barisan 10, 5,
2.5, 1.25, . . . adalah barisan geometrik dengan rasio 1/2. Jumlah suku-suku
dari barisan geometrik ini disebut dengan deret geometrik. Bentuk umum dari
barisan geometri adalah a, ar, ar2 , ar3 , ar4 , . . .; sedangkan bentuk umum dari
deret geometri adalah a + ar + ar2 + ar3 + ar4 + . . .
Lemma 3.1.1 Given an arithmetic sequence a+(a+b)+(a+2b)+· · ·+(a+(n−1)b),
where a is an initial term an b is a deference of any two successive terms. Then, we have
the followings:
Lema 3.1.1 Diberikan suatu deret aritmatika a+(a+b)+(a+2b)+· · ·+(a+(n−1)b),
dimana a adalah nilai suku awal dan b adalah beda suku-suku yang berurutan, maka
beberapa hal berikut berlaku:
Un = a + (n − 1)b
n
Sn =
[U1 + Un ]
2
n
Sn =
[2a + (n − 1)b]
2
1
Ut = a + (n − 1)b
2
(3.1)
(3.2)
(3.3)
(3.4)
Lemma 3.1.2 If we insert some k numbers on between any two successive numbers of
arithmetic sequence and they form a new arithmetic sequence, then
36
Chapter 3. Algebra Functions
Lema 3.1.2 Jika diantara dua buah bilangan yang berurutan dari suatu deret aritmatika disisipi k buah bilangan dan membentuk deret aritmatika baru maka
b
k+1
= n + (n − 1)k
b0 =
(3.5)
n0
(3.6)
Lemma 3.1.3 Let a + ar + ar2 + · · · + arn−1 be a geometric sequence, where a, r are
respectively an initial value and ratio. The followings hold for this squence
Lema 3.1.3 Diberikan suatu deret geometrik a + ar + ar2 + · · · + arn−1 , dimana a
adalah nilai suku awal dan r adalah rasio suku-suku yang berurutan. Beberapa hal
berikut berlaku untuk deret ini.
Un = arn−1
a(rn − 1)
Sn =
, r>1
r−1
a(1 − rn )
Sn =
, r<1
√1 − r
Ut = a rn−1
(3.7)
(3.8)
(3.9)
(3.10)
Lemma 3.1.4 If we insert some k numbers on between any two successive numbers of
geometric sequence and they form a new geometric sequence, then
Lema 3.1.4 Jika diantara dua buah bilangan yang berurutan dari suatu deret geometrik
disisipi k buah bilangan dan membentuk deret geometrik baru maka
√
r0 = k+1 r
(3.11)
n0 = n + (n − 1)k
(3.12)
37
Chapter 3. Algebra Functions
Lemma 3.1.5 For arithmetic and geometric series satisfy Un = Sn − S(n − 1).
Lema 3.1.5 Untuk deret aritmatik maupun geometrik berlaku Un = Sn − S(n − 1).
Lemma 3.1.6 An infinite convergence geometric series, where |r| < 1, satisfies S∞ =
a
.
1−r
Lema 3.1.6 Untuk deret geometrik tak hingga konvergen, dimana |r| < 1, berlaku
a
S∞ = 1−r
.
Example. Determine the sum of all radius of circles whose numbers are infinity.
Example. Tentukan jumlah seluruh jari-jari lingkaran berikut ini sampai pada
banyaknya lingkaran tak higga.
∞
Figure 3.1: Squares in the circles.
38
Chapter 3. Algebra Functions
Furthermore, how to find the n-term of sequence 1, 4, 11, 22, 37, . . . . Both formulas Un can not be used to answer this problems as the sequence does not have
a common difference at first layer, namely 3,7,11,15. Thus then, this sequence
admits an arithmetic sequence of common difference 4. The solution can be obtained by considering polynomial f (x) = ak xk + ak−1 xk−1 + ak−2 xk−2 + · · · + a0 x0 .
Selanjutnya, bagaimana menentukan suku ke−n dari barisan bilangan: 1, 4, 11,
22, 37, . . . . Kedua rumus Un di atas tidak dapat dipakai untuk menjawab pertanyaan ini, sebab beda pada layer pertama tidak sama, yaitu 3,7,11,15, baru
kemudian barisan beda ini merupakan barisan aritmatika dengan beda 4. Solusinya diperoleh dari mempertimbangkan fungsi polinomial f (x) = ak xk +
ak−1 xk−1 + ak−2 xk−2 + · · · + a0 x0 .
Lemma 3.1.7 Let f (x) = a1 x + a0 ; x = 1, 2, . . . , n be a linear function series of the
polynomial, it has common difference b = a1 at firs layer.
Lema 3.1.7 Misal deret fungsi linear dari polinomial di atas adalah f (x) = a1 x +
a0 ; x = 1, 2, . . . , n, maka deret ini mempunyai beda b = a1 pada layer pertama.
Proof. An expansion of the linear function for the n-terms is as follows:
Bukti. Ekspansi fungsi linear sampai suku ke−n adalah sebagai berikut:
a1 + a0 , 2a1 + a0 , . . . , a1 (n − 3) + a0 , a1 (n − 2) + a0 , a1 (n − 1) + a0 , a1 n + a0
hence the common difference of the successive terms is b = f (n) − f (n − 1) =
f (n − 1) − f (n − 2) = · · · = f (2) − f (1) = a1 , namely
sehingga beda dari suku-suku yang berurutan adalah b = f (n) − f (n − 1) =
f (n − 1) − f (n − 2) = · · · = f (2) − f (1) = a1 , yaitu
a1 + a0 , 2a1 + a0 , . . . , a1 (n − 3) + a0 , a1 (n − 2) + a0 , a1 (n − 1) + a0 , a1 n + a0
|
{z
}
|
{z
} |
{z
}
a1
a1
a1
Chapter 3. Algebra Functions
39
Lema 3.1.8 Let f (x) = a2 x2 + a1 x + a0 ; x = 1, 2, . . . , n be a quadratic function series
of the polynomial, it has common difference b = 2a2 at second layer.
Lema 3.1.9 Misal deret fungsi kuadrat dari polinomial di atas adalah f (x) = a2 x2 +
a1 x + a0 ; x = 1, 2, . . . , n, maka deret ini mempunyai beda b = 2a2 pada layer kedua.
Proof. An expansion of the quadratic function for the n-terms is as follows:
Bukti. Ekspansi fungsi quadratic untuk suku ke−n adalah sebagai berikut:
a2 + a1 + a0 , 4a2 + 2a1 + a0 , . . . , a2 (n − 1)2 + a1 (n − 1) + a0 , a2 n2 + a1 n + a0
hence the differences of the successive terms are as follows:
sehingga beda layer pertama dari suku-suku yang berurutan adalah sebagai
berikut:
bn = f (n) − f (n − 1) = 2a2 n − a2 + a1 = (2n − 1)a2 + a1
bn−1 = f (n − 1) − f (n − 2) = 2a2 n − 3a2 + a1 = (2n − 3)a2 + a1
bn−2 = f (n − 2) − f (n − 3) = 2a2 n − 5a2 + a1 = (2n − 5)a2 + a1
..
.
b3 = f (3) − f (2) = 5a2 + a1
b2 = f (2) − f (1) = 3a2 + a1 .
Therefore, the common difference of the successive terms of the difference sequence of the first layer is
Dengan demikian, beda dari suku-suku yang berurutan pada barisan beda untuk layer pertama adalah
b = bn − bn−1 = bn − bn−1 = · · · = b3 − b2 = 2a2
40
Chapter 3. Algebra Functions
or
atau
a + a1 + a0 , 4a2 + 2a1 + a0 , 9a2 + 3a1 + a0 , 16a2 + 4a1 + a0 , 25a2 + 5a1 + a0
|2
{z
}|
{z
}|
{z
}|
{z
}
3a2 +a1
|
{z
2a2
5a2 +a1
}|
{z
2a2
7a2 +a1
}|
{z
9a2 +a1 →Layer I
}
2a2 →Layer II
2
Lema 3.1.10 Let f (x) = a3 x3 + a2 x2 + a1 x + a0 ; x = 1, 2, . . . , n, be a cubic function
series of the polynomial, it has common difference b = 6a3 at third layer.
Lema 3.1.11 Misal deret fungsi kubik dari polinomial di atas adalah f (x) = a3 x3 +
a2 x2 + a1 x + a0 ; x = 1, 2, . . . , n, maka deret ini mempunyai beda b = 6a3 pada layer
ketiga.
Proof. As exercise.
Bukti. Sebagai latihan.
2
We conclude inductively in the following theorem:
Secara induktif dapat disimpulkan dalam teorema berikut:
Theorem 3.1.2 The series of polynomial function f (x) = ak xk +ak−1 xk−1 +ak−2 xk−2 +
· · · + a0 x0 ; x = 1, 2, . . . , n has a common difference b = k!ak at layer k.
Teorema 3.1.2 Deret fungsi polinomial f (x) = ak xk + ak−1 xk−1 + ak−2 xk−2 + · · · +
a0 x0 ; x = 1, 2, . . . , n, mempunyai beda yang sama b = k!ak pada layer ke−k.
41
Chapter 3. Algebra Functions
Proof. As exercise.
Bukti. Sebagai latihan.
2
Example. Given a series 15 + 32 + 63 + 108 + 167 + . . . . Obtain the n−term and
the n−sum.
Example. Diberikan suatu deret 15 + 32 + 63 + 108 + 167 + . . . . Tentukan suku
ke−n dan jumlah ke−n.
Solution. The series above forms the following
Solusi. Deret di atas akan mengikuti pola berikut
15
+ 32} |+{z63} +
108} +
167} + . . .
| {z
| {z
| {z
17
31
45
59
| {z } | {z } | {z }
14
14
14
Since the series has a common difference at second layer, we have k = 2 and
the function f (x) = a2 x2 + a1 x + a0 . We need to find the coefficients a2 , a1 , a0 .
Theorem 3.1.2 implies b = k!ak = 2!a2 = 14 → a2 = 7, it follows f (x) = 7x2 +
a1 x + a0 . Let Un = f (n) and Un = 7n2 + a1 n + a0 . We have the following:
Karena deret fungsi mempunyai beda yang sama pada layer kedua maka k = 2
dan rumus f (x) = a2 x2 + a1 x + a0 . Selanjutnya perlu dicari koefisien a2 , a1 , a0 .
Dari Theorema 3.1.2 diperoleh b = k!ak = 2!a2 = 14 → a2 = 7. Sehingga
f (x) = 7x2 + a1 x + a0 . Misal Un = f (n) dan Un = 7n2 + a1 n + a0 , maka akan
didapat:
n = 1 → 7 + a1 + a0 = 15 → a1 + a0 = 8
n = 2 → 28 + 2a1 + a0 = 32 → 2a1 + a0 = 4
From both equations, we have a1 = −4 dan a0 = 12 which implies that the
n−term is Un = 7n2 − 4n + 12.
42
Chapter 3. Algebra Functions
Dari kedua persamaan diperoleh nilai a1 = −4 dan a0 = 12. Sehingga suku
ke−n adalah Un = 7n2 − 4n + 12.
To find the n−sum of the above series, we consider the following series
Untuk menentukan jumlah ke−n dari deret di atas, maka kita lihat deret berikut
Un → 15, 32, 63, 108, 167, . . .
Sn → 15, 47, 110, 218, 385, . . .
15, 47 , 110 , 218 , 385 + . . .
| {z } | {z } | {z } | {z }
32
63
108
167
| {z } | {z } | {z }
31
45
59
| {z } | {z }
14
14
Since the sequence of function has a common difference on the third layer, we
have k = 3 and function f (x) = a3 x3 + a2 x2 + a1 x + a0 . We need to find the
coefficients a3 , a2 , a1 , a0 . Theorem 3.1.2 implies b = k!ak = 3!a3 = 14 → a3 = 73 .
Hence f (x) = 37 x3 + a2 x2 + a1 x + a0 . Let Sn = f (n) and Sn = 37 n3 + a2 n2 + a1 n + a0 .
We have the following:
Karena deret fungsi mempunyai beda yang sama pada layer ketiga maka k = 3
dan rumus f (x) = a3 x3 + a2 x2 + a1 x + a0 . Selanjutnya perlu dicari koefisien
a3 , a2 , a1 , a0 . Dari Theorema 3.1.2 diperoleh b = k!ak = 3!a3 = 14 → a3 =
7
.
3
Sehingga f (x) = 37 x3 + a2 x2 + a1 x + a0 . Misal Sn = f (n) maka Sn = 73 n3 + a2 n2 +
a1 n + a0 , maka akan didapat:
7
38
+ a2 + a1 + a0 = 15 → a2 + a1 + a0 =
3
3
56
85
n=2 →
+ 4a2 + 2a1 + a0 = 47 → 4a2 + 2a1 + a0 =
3
3
189
141
n=3 →
+ 9a2 + 3a1 + a0 = 110 → 9a2 + 3a1 + a0 =
3
3
n=1 →
Solving the three equations, we have a2 = 23 , a1 =
the n−sum is Sn = 37 n3 + 32 n2 +
67
n.
6
67
6
and a0 = 0 which implies
Chapter 3. Algebra Functions
43
Dengan menyelesaikan ketiga persamaan di atas diperoleh nilai a2 = 32 , a1 =
67
6
dan a0 = 0. Sehingga jumlah suku ke−n adalah Sn = 73 n3 + 32 n2 +
2
3.2
67
n.
6
Arithmetic, Geometric, Harmonic, and Quadratic
Means
Rataan Aritmatik, Geometrik, Harmonik dan
Kuadratik
In mathematics or statistics, an arithmetic mean of a list of numbers is the sum
of all of the list divided by the number of items in the list. The arithmetic mean
is the most commonly-used type of average and is often referred to simply as
the average. A geometric mean is a type of mean or average, which indicates
a central tendency of numbers. It is similar to arithmetic mean, which is what
most people think of with the word ”average,” except that instead of adding the
set of numbers and then dividing the sum by the count of numbers in the set
n, the numbers are multiplied and then the nth root of the resulting product is
taken. A harmonic mean (formerly sometimes called the subcontrary mean) is
one of several kinds of average. Typically, it is appropriate for situations when
the average of rates is desired. A quadratic mean is a type of average which is
calculated as the square root of the mean of the squares.
Dalam matematika atau statistika, suatu rataan aritmatik dari daftar bilangan
adalah jumlah bilangan itu dibagi dengan banyaknya bilangan dalam daftar.
Rataan aritmatika adalah sesuatu yang paling sering dipakai dan secara umum
juga disebut dengan rata-rata. Rataan geometrik adalah jenis rataan atau ratarata yang merepresentasikan suatu tendensi sentral dari sekumpulan bilangan.
Hampir sama dengan rataan aritmatik dimana banyak orang berpikir bahwa ini
44
Chapter 3. Algebra Functions
hanyalah rata-rata, namun dalam rata-rata seluruh bilangan dalam daftar dijumlahkan kemudian dibagi dengan banyaknya bilangan dalam daftar, untuk
rataan geometrik semua bilangan dalam daftar dikalikan kemudian ditarik akar
pangkat n. Rataan harmonik (dulu disebut sebagai kebalikan rataan) adalah
jenis lain dari rata-rata. Biasanya ini sesuai untuk menentukan rata-rata yang
mempertimbangkan tingkatan. Rataan kuadratik adalah tipe dari rata-rata yang
diperoleh dari menarik akar kuadrat dari rata-rata kuadratnya.
Let AM =Arithmetic Mean, GM =Geometric Mean, HM =Harmonic Mean and
QM =Quadratic Mean. For any positive real numbers a, b, it satisfies
Misal AM =Rataan Aritmatik, GM =Rataan Geometrik, HM =Rataan Harmonik
and QM =Rataan Kuadratik. Untuk sebarang a, b bilangan real positif, berlaku
(a − b)2 ≥ 0 ⇐⇒ a2 + b2 ≥ 2ab
(3.13)
√
√
a and b → b then, from (3.13), it follows a + b ≥ 2 ab or
√
√
√
Jika a → a dan b → b maka (3.13) menjadi a + b ≥ 2 ab atau
If a →
√
a+b √
≥ ab
2
(3.14)
In general we can present (3.14) as follows
Secara umum kita dapat menulis (3.14) sebagai
√
x1 + x2 + · · · + xn
≥ n x1 x2 . . . x n
{z
}
n
{z
} |
|
GM
AM
If a →
1
a
Jika a →
and b →
1
a
1
b
dan b →
(3.15)
then (3.14) shows
1
b
maka (3.14) menjadi
√
ab ≥
1
a
2
+
1
b
(3.16)
45
Chapter 3. Algebra Functions
In general, we can present (3.16) as
Secara umum kita dapat menulis (3.16) sebagai
√
n
x1 x2 . . . x n ≥
1
x1
|
+
1
x2
n
+ · · · + x1n
{z
}
(3.17)
HM
From (3.13), we get 2(a2 + b2 ) ≥ a2 + 2ab + b2 ⇐⇒ 2(a2 + b2 ) ≥ (a + b)2 . If the two
¡ ¢2
2
2
sides are divided by 4 then we have a +b
≥ a+b
, or
2
2
Dari (3.13) didapat 2(a2 + b2 ) ≥ a2 + 2ab + b2 ⇐⇒ 2(a2 + b2 ) ≥ (a + b)2 . Bila kedua
¡ a+b ¢2
2
2
ruas dibagi 4, diperoleh a +b
≥
, atau
2
2
r
a2 + b2
a+b
≥
2
2
(3.18)
In general (3.18) can be written as
Secara umum (3.18) ditulis sebagai
r
x21 + x22 + · · · + x2n
x1 + x2 + · · · + xn
≥
n
{zn
}
|
(3.19)
QM
Therefore, from(3.15), (3.17) and (3.19) we can conclude that for any positive real
numbers x1 , x2 , . . . , xn satisfy:
Dengan demikian, berdasarkan (3.15), (3.17) dan (3.19) dapat disimpulkan bahwa
untuk setiap bilangan real positif x1 , x2 , . . . , xn berlaku:
QM ≥ AM ≥ GM ≥ HM
where
46
Chapter 3. Algebra Functions
dimana
r
x21 + x22 + · · · + x2n
n
x1 + x2 + · · · + xn
AM =
n
√
GM = n x1 x2 . . . xn
n
HM = 1
1
+ x2 + · · · + x1n
x1
QM =
Example. Let x, y, z be any real positive numbers such that x + y + z = 1. Prove
that xy(x + y)2 + yz(y + z)2 + xz(x + z)2 ≥ 4xyz.
Contoh. Diketahui x, y, z adalah bilangan real positif sehingga x + y + z = 1.
Buktikan bahwa xy(x + y)2 + yz(y + z)2 + xz(x + z)2 ≥ 4xyz.
Solution. Since x + y + z = 1, we have
Solusi. Karena x + y + z = 1 maka didapat
xy(1 − z)2 + yz(1 − x)2 + xz(1 − y)2 ≥ 4xyz
xy + yz + xz − 6xyz + xyz 2 + x2 yz + xy 2 z ≥ 4xyz
xy + yz + xz + xyz 2 + x2 yz + xy 2 z
1 1 1
+ + +x+y+z
x y z
1 1 1
+ +
x y z
1
3
x+y+z
∴
3 }
| {z
AM
≥ 10xyz
≥ 10
≥ 9
≥
≥
3
1
x
+
1
y
+
1
z
+
1
z
3
1
x
|
+
1
y
{z
HM
}
Chapter 3. Algebra Functions
3.3
47
The Polynomials and Remainder Theorem
Suku Banyak dan Teorema Sisa
A polynomial f (x) of degree n can be presented as:
Suatu suku banyak f (x) yang berderajad n dinyatakan dengan:
f (x) = a0 xn + a1 xn−1 + · · · + an−1 x + an
where a0 , a1 , . . . , an are constant, a0 6= 0 and n is a cardinal number.
dimana a0 , a1 , . . . , an adalah konstanta, a0 6= 0 dan n adalah bilangan cacah.
3.3.1
Polynomials Division
Pembagian Suku Banyak
The division of polynomial is similar to the division of numbers. For instance in
the number: Since 3 × 4 = 12, it follows 12 : 4 = 3 or 12 : 3 = 4. In the case of
12 : 4 = 3, numbers 4, 3 are respectively called divisor and quotient.
Pembagian suku banyak menyerupai pembagian bilangan. Sebagai contoh pada
bilangan: Karena 3×4 = 12 maka 12 : 4 = 3 atau 12 : 3 = 4. Pada kasus 12 : 4 = 3
maka bilangan 4, 3 masing-masing disebut pembagi dan hasil bagi.
A polynomial f (x) divided by a divisor P (x) will give a quotient H(x) and a
remainder S(x). Mathematically, we can write as:
f (x) = P (x)H(x) + S(x)
where: f (x) = is a polynomial of degree n; P (x) = is a polynomial of degree k;
H(x) = is a polynomial of degree n − k; S(x) = is a polynomial of degree k − 1.
Chapter 3. Algebra Functions
48
Suatu suku banyak f (x) yang dibagi dengan pembagi P (x) akan menghasilkan
hasil bagi H(x) dan sisanya S(x). Secara matematis dapat ditulis sebagai:
f (x) = P (x)H(x) + S(x)
dimana: f (x) = suku banyak berderajad n; P (x) = suku banyak berderajad k;
H(x) = suku banyak berderajad n − k; S(x) = suku banyak berderajad k − 1.
When dividing polynomials, we need to consider the followings:
1. If the divisor is a linear term then the quotient and remainder can be obtained by Horner technique.
2. If the divisor is not linear and not be able to be factorized into product of
linear terms then the quotient and remainder can be obtained by identity
technique.
Dalam melakukan pembagian terhadap suku banyak perlu diperhatikan hal-hal
berikut:
1. Jika pembaginya linier, maka hasil bagi dan sisanya dapat dicari dengan
menggunakan cara Horner.
2. Jika pembaginya bukan linier dan tidak dapat diuraikan menjadi bentuk
perpangkatan linier maka hasil bagi dan sisanya dapat dicari dengan pergunakan metoda Identitas.
49
Chapter 3. Algebra Functions
3.3.2
Remainder Theorem
Teorema Sisa
1. If polynomial f (x) is divided by (x ± a) then the remainder is f (∓a).
2. If polynomial f (x) is divided by (ax ± b) then the remainder is f (∓ ab ).
3. If (x − a)|f (x) then f (a) = 0.
1. Jika suatu suku banyak f (x) dibagi dengan (x ± a) maka sisanya f (∓a).
2. Jika suatu suku banyak f (x) dibagi dengan (ax ± b) maka sisanya f (∓ ab ).
3. Jika (x − a)|f (x) maka f (a) = 0.
3.3.3
Factor Theorem
Teorema faktor
1. If (x − a) is a factor of f (x) then the root of f (x) = 0 is x = a.
2. If polynomial f (x) satisfies f (a) = 0, f (b) = 0 and f (c) = 0 then f (x) is
divisible by (x − a)(x − b)(x − c).
3. If f (x) is divided by (x − a)(x − b) then the remainder is S(x) =
(x−a)
f (b) +
(b−a)
(x−b)
f (a).
(a−b)
4. If f (x) is divided by (x − a)(x − b)(x − c) then the remainder is S(x) =
(x−a)(x−b)
f (c)
(c−a)(c−b)
+
(x−a)(x−c)
f (b)
(b−a)(b−c)
+
(x−b)(x−c)
f (a).
(a−b)(a−c)
50
Chapter 3. Algebra Functions
1. Jika (x − a) adalah faktor dari f (x) maka akar dari f (x) = 0 adalah x = a.
2. Jika pada suku banyak f (x) berlaku f (a) = 0, f (b) = 0 dan f (c) = 0 maka
f (x) habis dibagi (x − a)(x − b)(x − c).
3. Jika f (x) dibagi dengan (x − a)(x − b) maka sisanya S(x) =
(x−a)
f (b)
(b−a)
+
(x−b)
f (a).
(a−b)
4. Jika f (x) dibagi dengan (x−a)(x−b)(x−c) maka sisanya S(x) =
(x−a)(x−c)
f (b)
(b−a)(b−c)
3.3.4
+
(x−a)(x−b)
f (c)+
(c−a)(c−b)
(x−b)(x−c)
f (a).
(a−b)(a−c)
Properties of Polynomial Roots
Sifat-Sifat Akar-Akar Suku Banyak
In this section, we consider Vieta’s Formula. Let si be the sum of the products of
distinct polynomial roots rj of the polynomial equation of degree n
Dalam hal ini akan disajikan penggunaan rumus Vieta. Misal si adalah jumlah dari hasil kali akar-akar polinomial yang berbeda rj dari sebuah polinomial
berderajad n
an xn + an−1 xn−1 + ... + a1 x + a0 = 0
where the roots are taken i at a time (i.e., si is defined as the symmetric polynomial Πi (r1 , ..., rn ) for i = 1, ..., n). For example, the first few values of si are
dimana akar-akar itu dihitung sebanyak i dalam suatu proses (atau si didefinisikan sebagai polinomial simetrik Πi (r1 , ..., rn ) untuk i = 1, ..., n). Sebagai contoh, beberapa nilai si yang pertama adalah
s1 = r1 + r2 + r3 + r4 + . . .
s2 = r1 r2 + r1 r3 + r1 r4 + r2 r3 + . . .
s3 = r1 r2 r3 + r1 r2 r4 + r2 r3 r4 + . . .
..
.
51
Chapter 3. Algebra Functions
and so on. Then Vieta’s formulas states that
dan seterusnya. Maka rumus Vieta’s dinyatakan sebagai
si = (−1)i
an−i
an
The followings are some example of Vieta’s formula.
Berikut ini adalah beberapa contoh dari rumusan Vieta.
1. For the polynomial ax3 + bx2 + cx + d = 0, we have:
Pada suku banyak ax3 + bx2 + cx + d = 0 berlaku:
1) x1 + x2 + x3 = −b/a
2) x1 x2 + x1 x3 + x2 x3 = c/a
3) x1 · x2 · x3 = −d/a
2. For the polynomial ax4 + bx3 + cx2 + dx + e = 0, we have:
Pada suku banyak ax4 + bx3 + cx2 + dx + e = 0 berlaku:
1) x1 + x2 + x3 + x4 = −b/a
2) x1 x2 + x1 x3 + x1 x4 + x2 x3 + x2 x4 + x3 x4 = c/a
2) x1 x2 x3 + x1 x2 x4 + x1 x3 x4 + x2 x3 x4 = −d/a
3) x1 · x2 · x3 · x4 = e/a
We note that if the degree of polynomial is even then the the value of s is positive
otherwise it is negative.
Dapat dicatat bahwa bila pangkat tertinggi dari polinomial adalah genap maka
nilai s adalah positif jika ganjil maka nilai s adalah negatif.
To find some rational roots of polynomials can be used the following steps:
Untuk menentukan beberapa akar rasional dari suku banyak dapat digunakan
langkah-langkah berikut:
Chapter 3. Algebra Functions
52
1. If the sum of all polynomial coefficients is equal to 0 then x = 1 is one of
the root.
Jika jumlah seluruh koefisien suku banyak sama dengan 0, maka x = 1
merupakan salah satu akarnya.
2. If the sum of the coefficients of odd order and even order are the same then
x = −1 is one of the root.
Jika jumlah koefisien pangkat ganjil dan genap adalah sama, maka x = −1
merupakan salah satu akarnya.
3. If (1) and (2) are not applicable then consider a trial and error technique by
finding factors of the coefficient of the lowest order and substituting into
f (x). Observe whether f (x) = 0 or not.
Jika langkah (1) dan (2) tidak memenuhi, maka gunakan cara coba-coba
yaitu dengan menentukan faktor dari koefisien pangkat terendahnya dan
masukkan ke dalam f (x). Amati apakah f (x) = 0 atau tidak.
P ROBLEMS
AND
S OLUTIONS
S OAL - SOAL DAN P EMBAHASAN
1. A geometric series is presented as :
1
1
+ + 2 + 16 + 128 + 1024
32 4
When we insert two numbers in between any two successive numbers
such that it forms a new geometric series, determine the new ratio and
the number of terms of the new geometric series.
Suatu deret geometri diketahui sebagai berikut:
1
1
+ + 2 + 16 + 128 + 1024
32 4
53
Chapter 3. Algebra Functions
Jika disisipkan dua buah bilangan kedalam dua suku yang berurutan pada
deret geometri ini sedemikian hingga deret itu membentuk deret geometri
baru, tentukan ratio dan banyaknya suku deret geometri baru tersebut.
Solution. We have the following:
Solusi. Diketahui berikut:
1
1
+ + 2 + 16 + 128 + 1024
32 4
So (Sehingga)
Un
Un − 1
r = 8
r =
Let k, n be number of inserted numbers and terms. We have k = 2 and
n = 6. The new ratio is:
Misal k, n masing-masing adalah banyaknya bilangan yang disisipkan dan
banyaknya suku sebelum disisikpan, maka diperoleh k = 2 dan n = 6.
Ratio yang baru adalah:
r0 =
r0 =
k+1
√
3
√
r
8
r0 = 2
Number of the new terms is
Banyaknya suku yang baru adalah
n0 = n + (n − 1)k
n0 = 6 + (6 − 1).2
n0 = 16
54
Chapter 3. Algebra Functions
The desired geometric series is:
Deret geometri yang dicari adalah:
1
1
1 1 1
+
+ + + + 1 +2 + .... + 265
+ 512} +1024
| {z
32 |16{z 8} 4 |2 {z }
|
{z
}
the inserted numbers (bilangan yang disisipkan)
2. Let a, b and c be any riel positive numbers such that abc = 1. Prove that
1
a3 (b+c)
+
1
b3 (a+c)
+
1
c3 (a+b)
≥ 32 .
Misal a, b dan c adalah bilangan riil positif sedemikian hingga abc = 1.
Buktikan bahwa
1
a3 (b+c)
+
1
b3 (a+c)
+
1
c3 (a+b)
≥ 32 .
Solution. Let
S=
1
1
1
+
+
a3 (b + c) b3 (a + c) c3 (a + b)
(3.20)
and a = x1 , b = y1 , c = z1 . Let T = x + y + z. Since abc = 1, we have xyz = 1.
Substituting the new a, b, c we have:
Solusi. Misal
S=
dan a =
1
,
x
b =
1
,
y
1
1
1
+ 3
+ 3
+ c) b (a + c) c (a + b)
a3 (b
c =
1
.
z
(3.21)
Misal T = x + y + z. Karena abc = 1 maka
xyz = 1. Dengan mensubstitusikan a, b, c yang baru didapat:
1
x2
T 2 − (T 2 − x2 )
T2
=
=
=
−T −x
a3 (b + c)
T −x
T −x
T −x
1
y2
T 2 − (T 2 − y 2 )
T2
=
=
=
−T −y
b3 (a + c)
T −y
T −y
T −y
1
z2
T 2 − (T 2 − z 2 )
T2
=
=
=
−T −z
c3 (a + b)
T −z
T −z
T −z
Substituting the above equations into (3.21) we have the following.
Dengan mensubstitusikan semua persamaan di atas ke dalam (3.21) maka
55
Chapter 3. Algebra Functions
diperoleh berikut ini.
µ
¶
1
1
1
2
S = T
+
+
− 3T − (x + y + z)
T −x T −y T −z
µ
¶
1
1
1
2
+
+
− 4T
= T
T −x T −y T −z
µ µ
¶¶
1
1
1
2 3
= T
+
+
− 4T
3 T −x T −y T −z
µ
= T ·3
2
1
T −x
+
1
T −y
+
1
T −z
3
{z
|
¶
− 4T
}
AM
µ
2
≥ T ·3
¶
3
1
1
T −x
|
µ
+
1
1
T −y
{z
+
1
1
T −z
3
− 4T
}
HM
¶
9
T
− 4T
(T − x) + (T − y) + (T − z)
µ
¶
9
2
T
− 4T
3T − (x + y + z)
µ ¶
9
2
− 4T
T
2T
9T
T
x+y+z
3 x+y+z
− 4T = =
= ·
2
2
2
2 | {z
3 }
2
=
=
=
=
AM
3√
3
3
xyz =
2
2
≥
Therefore
Dengan demikian
∴
1
1
1
3
+
+
≥
a3 (b + c) b3 (a + c) c3 (a + b)
2
3. Given that f (x) is a polynomial of degree 2. When f (x) is divided by x + 1,
the remainder is 3; when f (x) is divided by x − 3, the remainder is 23; and
56
Chapter 3. Algebra Functions
when f (x) is divided by x − 2, the remainder is 15. Find the polynomial
f (x).
Diketahui f (x) adalah sebuah polinomial berderajad 2. Saat f (x) dibagi
dengan x + 1, maka sisanya adalah 3; saat f (x) dibagi dengan x − 3, maka
sisanya adalah 23; dan saat f (x) dibagi dengan x − 2, maka sisanya adalah
15. Tentukan polinomial f (x) tersebut.
Solution. Let f (x) = qx2 + mx + n. When f (x) is divided by x + 1, the
remainder is s(−1) = q(−1)2 + m(−1) + n = 3; when f (x) is divided by
x − 3, the remainder is s(3) = q(3)2 + m(3) + n = 23; and when f (x) is
divided by x − 2, the remainder is s(2) = q(2)2 + m(2) + n = 15. Thus, we
have
Misal f (x) = qx2 + mx + n. Saat f (x) dibagi dengan x + 1, maka sisanya
adalah s(−1) = q(−1)2 + m(−1) + n = 3; saat f (x) dibagi dengan x − 3,
maka sisanya adalah s(3) = q(3)2 + m(3) + n = 23; dan saat f (x) dibagi
dengan x − 2, maka sisanya adalah s(2) = q(2)2 + m(2) + n = 15. Sehingga,
diperoleh
q−m+n=3
9q + 3m + n = 23
4q + 2m + n = 15
Solving the three equations, we get q = 1, m = 3 and n = 5.
Dengan menyelesaikan ketiga persamaan di atas, maka didapat q = 1, m =
3 dan n = 5.
¥¥¥¥¥¥
CHAPTER 4
Trigonometry
Trigonometri
Trigonometry (from Greek trigoÌ„non ”triangle” + metron ”measure”) is a branch
of mathematics that deals with triangles, particularly those plane triangles in
which one angle has 90 degrees (right triangles). Trigonometry deals with relationships between the sides and the angles of triangles and with the trigonometric functions, which describe those relationships.
Trigonometri (berasal dari kata yunani trigoÌ„non ”segitiga” + metron ”mengukur”)
adalah suatu cabang matematika yang berkenaan dengan segitiga, khususnya
segitiga dalam bidang dimana satu sudut mempunyai besar sudut 90 derajad
(segitiga siku-siku). Trigonometri berkaitan dengan hubungan anatara sisi-sisi
dan sudut-sudut sebuah segitiga dengan fungsi trigonometri yang menggambarkan sebuah relasi diantaranya.
Trigonometry has applications in both pure mathematics and in applied mathematics, where it is essential in many branches of science and technology. It is
usually taught in secondary schools either as a separate course or as part of a
precalculus course. Trigonometry is informally called ”trig”.
Aplikasi trigonometri dapat ditemukan baik dalam matematika murni maupun
matematika terapan yang keduanya ini sangat berguna pada beberapa cabang
dalam sain dan teknologi. Trigonometri ini biasanya diajarkan di sekolah menengah baik dalam matapelajaran tertentu atau merupakan bagian dari mata pelajaran kalkulus. Secara informal isitilah trigonometri ini disebut dengan ”trig”.
57
58
Chapter 4. Trigonometry
4.1
Trigonometric Function
Fungsi Trigonometri
Consider the following figure, we define the trigonometric functions as : sin α =
y
, cos α
r
= xr , tg α = yx , ctg α = yx , sec α =
1
,
cos α
cossec α =
1
.
sin α
Perhatikan gambar berikut, maka fungsi-fungsi trigonometri didefinisikan sebagai: sin α = yr , cos α = xr , tg α = yx , ctg α = yx , sec α =
1
,
cos α
cossec α =
1
.
sin α
y
r
0
α
P (x, y)
x
r
y
α
x
Figure 4.1: The right triangle trigonometric system
Theorem 4.1.1 Pythagoras equality is stated as follow.
Teorema 4.1.1 Kesamaan Pythagoras dinyatakan dalam berikut ini.
1.
sin2 α + cos2 α = 1, rumus identitas
2.
1 + ctg 2 α = cossec 2 α
3.
tg 2 α + 1 = sec2 α
The values of trigonometric function in every quadrant is described on a system
of quadrant, where 90o < α2 < 180o , 180o < α4 < 270o and 270 < α3 < 360o .
Considering the system of quadrant, the values of trigonometric function for any
angle α can be memorized easily. In Quadrant I, all of trigonometric function
59
Chapter 4. Trigonometry
sign are positive, Quadrant II, only sinus is positive, Quadrant III, only tangent
is positive, and Quadrant IV, only cosinus is positive. Therefore, the keyword to
memorize it is ASTC.
Nilai fungsi pada masing-masing kuadran tertera dalam sistem kuadran, dimana 90o < α2 < 180o , 180o < α4 < 270o dan 270 < α3 < 360o . Dengan
memetakan pada sistem kuadran ini maka nilai fungsi trigonometri untuk sebarang sudut α dapat dengan mudah diingat tandanya. Pada kuadaran I all
tanda fungsi trigonometri positif, kuadaran II hanya sinus yang positif, kuadaran
III hanya tangent yang positif dan kuadaran IV hanya cosinus yang positif.
Dengan demikian kata kunci untuk mengingatnya adalah ASTC.
Table 4.1: Trigonometric quadrant system
II
I
sin α2 > 0
all > 0
III
IV
tg α3 > 0 cos α4 > 0
Thus that, understanding the sign, to obtain the values of trigonometric function
we just need to remember its values at the first quadrant. Use the key words
ASTC to obtain the values on the following quadrant. For instance, to determine
sin 150o , we only need to know sin 30o as sin(180o − 30o ) = sin 30o . We refer to
second quadrant that sinus is positive which follows that sin 30o =
1
2
Dengan memahami nilai tanda ini maka untuk hal tertentu kita hanya dapat
mengingat nilai nilai fungsi trigonometri pada kuadran pertama, selanjutnya
gunakan kata kunci ASTC untuk menentukan nilai-nilai di kuadran lainnya. Sebagai contoh: untuk menetukan sin 150o , kita cukup mengetahui sin 30o karena
60
Chapter 4. Trigonometry
sin(180o − 30o ) = sin 30o . Kita ingat di kuadran kedua bahwa sinus adalah posisitif, sehingga sin 30o =
1
2
We summarize that the values of sinus, cosinus, tangent and cotangent for special angles are as follows.
Nilai sinus, cosinus, tangent dan cotangent untuk sudut-sudut istimewa pada
kuadran pertama adalah sebagai berikut.
Table 4.2: The value of trigonometric functions for special angles
0o
30o
sin
0
1
2
cos
1
tg
0
ctg
∞
√
1
3
2
√
1
3
3
√
3
45o
√
1
2
2
√
1
2
2
1
1
60o 90o
√
1
3 1
2
1
2
0
3
√
1
3
3
∞
√
0
The trigonometric functions in every quadrant does not remain the same. To
easily memorize the change of trigonometric functions of any angles (X o − α),
it can be showed as the following system. From Quadrant I −→ IV in clockwise direction, the situations are B(berubah), Tb(tidak berubah), B(berubah),
Tb(tidak berubah) or in short form we have an acronym BTBT. Applying the
acronym ASTC, we easily put sign + or − on the resulted values.
Fungsi trigonometri di beberapa kuadran tidak selalu tetap. Untuk mempermudah ingatan kita tentang perubahan nilai fungsi trigonometri dari beberapa
sudut (X o − α) dapat disajikan dalam sistem berikut. Dari kuadaran I −→
IV berlawanan jarum jam, kondisinya adalah B(berubah), Tb(tidak berubah),
B(berubah), Tb(tidak berubah) atau dapat disingkat BTBT. Dengan menerapkan singkatan ASTC kita dapat memberikan tanda + dan − pada nilai-nilai itu
61
Chapter 4. Trigonometry
dengan mudah.
Table 4.3: The value of trigonometric functions for any angle (X o − α)
Tb
o
B
o
sin(180 − α) = sin α
sin(90 − α) = cos α
cos(180o − α) = − cos α
cos(90o − α) = sin α
tg (180o − α) = − tg α
tg (90o − α) = ctg α
ctg (180o − α) = − ctg α
ctg (90o − α) = tg α
B
Tb
sin(270o − α) = − cos α
sin(360o − α) = − sin α
cos(270o − α) = − sin α
cos(360o − α) = cos α
tg (270o − α) = ctg α
tg (360o − α) = − tg α
ctg (270o − α) = tg α
ctg (360o − α) = − ctg α
If an angle −α is obtained by turning a phase in clockwise direction then the
angle −α take a place in Quadrant IV. Thus, we know that the values of trigonometric functions for negative angles are exactly the values of trigonometric functions on Quadrant IV. For detail, see the followings.
Bila sudut −α diperoleh dengan memutar jangka searah jarum jam maka sudut
−α berada pada kuadarn IV. Dengan demikian dapat dipahami bahwa nilai
fungsi trigonometri pada sudut negatif setara dengan nilai fungsi trigonometri
pada sistem di atas pada kuadran IV. Nilai selengkapnya disajikan dalam tabel
berikut.
Furthermore, we can develop a quadrant system for trigonometric function values for any angle (X o + α). In this case, we generate an acronym TBTB as a
keyword to remember.
62
Chapter 4. Trigonometry
Table 4.4: The value of trigonometric functions for negative angles
sin(−α) = sin(360o − α) = − sin α
cos(−α) = cos(360o − α) = cos α
tg (−α) = tg (360o − α) = − tg α
ctg (−α) = ctg (360o − α) = − ctg α
Disamping itu, dapat pula digambarkan sistem kuadran untuk menentukan
nilai fungsi trigonometri pada sudut (X o + α). Dalam sistem ini diperoleh
singkatan TBTB sebagai kata kunci untuk mengingat.
Table 4.5: The value of trigonometric functions for any angle (X o + α)
B
Tb
o
sin(360 + α) = sin α
o
cos(90 + α) = sin α
cos(360o + α) = cos α
tg (90o + α) = − ctg α
tg (360o + α) = tg α
ctg (90o + α) = − tg α
ctg (360o + α) = ctg α
sin(90 + α) = − cos α
Tb
o
B
sin(180o + α) = − sin α
sin(270o + α) = cos α
cos(180o + α) = − cos α
cos(270o + α) = − sin α
tg (180o + α) = tg α
tg (270o + α) = − ctg α
ctg (180o + α) = ctg α
ctg (270o + α) = − tg α
63
Chapter 4. Trigonometry
Example.
Contoh.
sin(270o −30o )
!
cos(180o −45o )
sin(270o −30o )
!
cos(180o −45o )
1. Determine the value of
Tentukan nilai dari
o
o
sin(270 −30 )
Solution. Remember the angle (X o −α), ASTC and BTBT, thus cos(180
o −45o ) =
p
o
− cos 30
= 3/2.
− cos 45o
o
o
sin(270 −30 )
Solusi. Ingat sudut (X o −α), ASTC dan BTBT, maka cos(180
o −45o ) =
p
3/2.
− cos 30o
− cos 45o
=
tg (360o +60o )
!
cos(180o +90o )
tg (360o +60o )
!
cos(180o +90o )
2. Determine the value of
Tentukan nilai dari
o
o
tg (360 +60 )
Solution. Remember the angle (X o +α), ASTC and TBTB, thus cos(180
o +90o ) =
√
tg 60o
== 3.
− sin 90o
o
o
tg (360 +60 )
Solusi. Ingat sudut (X o +α), ASTC dan TBTB, maka cos(180
o +90o ) =
√
3.
4.1.1
tg 60o
− sin 90o
Sine and Cosine Rule
Aturan Sinus dan Cosinus
The following figure shows 4ABC and a circle of radius R.
Gambar dibawah ini memperlihatkan 4ABC dan lingkaran dengan jari-jari R.
Theorem 4.1.2 (Sine Rule) The sine rule states:
Theorem 4.1.3 (Aturan sinus) Aturan sinus mengatakan bahwa:
b
c
a
=
=
= 2R
sin α
sin β
sin γ
=
64
Chapter 4. Trigonometry
B
B
β
β
a
c
A
α
0
B
γ
b
0
A
C
b
C
A
(ii)
(i)
a
c
β
α
Z
C
(iii)
Figure 4.2: Triangle and circle of radius R
Proof. As an exercise.
Bukti. Sebagai bahan latihan.
Theorem 4.1.4 (Cosine Rule) The cosine rule states:
Teorema 4.1.2 (Aturan cosinus) Aturan cosinus mengatakan bahwa:
a2 = b2 + c2 − 2bc cos α
b2 = a2 + c2 − 2ac cos β
c2 = a2 + b2 − 2ab cos γ
Proof. Consider Figure 5.1(iii)
Bukti. Perhatikan Gambar 5.1(iii)
CZ = AC − AZ = b − c cos α
BZ = c sin α
BC 2 = CZ 2 + BZ 2
a2 = (b − c cos α)2 + (c sin α)2
= b2 − 2bc cos α + c2 (cos2 α + sin2 α)
= b2 − 2bc cos α + c2
Chapter 4. Trigonometry
65
Thus a2 = b2 + c2 − 2bc cos α. In the same way, we can prove that b2 = a2 + c2 −
2ac cos β and c2 = a2 + b2 − 2ab cos γ.
Sehingga a2 = b2 + c2 − 2bc cos α. Dengan cara yang sama, dapat dibuktikan
b2 = a2 + c2 − 2ac cos β dan c2 = a2 + b2 − 2ab cos γ.
2
Example. Prove that
Contoh. Buktikan bahwa
sin2 α = sin2 β + sin2 γ − 2 sin β sin γ cos α
sin2 β = sin2 α + sin2 γ − 2 sin α sin γ cos β
sin2 γ = sin2 α + sin2 β − 2 sin α sin β cos γ
Solution. The sine rule states that a = 2R sin α, b = 2R sin β and c = 2R sin γ.
Substituting into cosine rule above, we will get those formulas.
Solusi. Aturan sinus menyatakan bahwa a = 2R sin α, b = 2R sin β dan c =
2R sin γ. Substitusikan ke dalam aturan cosinus di atas maka akan diperoleh
rumus-rumus itu.
4.1.2
Formulas of Sum and Difference of Angles
Rumus-rumus Jumlah dan Selisih Sudut
A circle of radius 1 with point A and B, where ∠AOX = α and ∠BOX = β, is
presented in Figure 4.3.
Sebuah lingkaran dengan jari-jari 1 dan titik A dan B, dimana ∠AOX = α dan
∠BOX = β, terlihat dalam Gambar 4.3.
Theorem 4.1.5 Let α and β be any angles. The formula of sum and difference of angles
can be stated as follows.
66
Chapter 4. Trigonometry
y
B
β
O
α
A
x
Figure 4.3: Sum and Difference of Angles
Teorema 4.1.3 Diberikan sudut α and β, maka rumus penjumlahan dan pengurangan
sudut dapat dinyatakan sebagai berikut.
1.
sin(α ± β) = sin α cos β ± cos α sin β
2.
cos(α ± β) = cos α cos β ∓ sin α sin β
tg α ± tg β
tg (α ± β) =
1 ∓ tg α tg β
3.
Bukti. Consider Figure 4.3, we have A(cos α, sin α) and B(cos β, sin β). the length
of AB can be obtained by the formula:
Bukti. Perhatikan Gambar 4.3, kita memiliki A(cos α, sin α) dan B(cos β, sin β).
Panjang AB dapat dicari dengan rumus:
AB 2 = (cos α − cos β)2 + (sin α − sin β)2
= 2(1 − (cos α cos β + sin α sin β)).
(4.1)
Cosine rule gives
Aturan cosinus memberikan Aturan cosinus memberikan
AB 2 = OA2 + OB 2 − 2 · OA · OB · cos ∠AOB
= 2(1 − cos(β − α)).
(4.2)
67
Chapter 4. Trigonometry
From Equations (4.1) and (4.2), it follows
Dari persamaan (4.1) dan (4.2) memberikan
cos(α − β) = cos α cos β + sin α sin β,
which completes the proof. We leave other formulas as an exercise
dengan demikian terbuktilah. Rumus lainnya ditinggal untuk bahan latihan. 2
Theorem 4.1.6 Given an angle α, the formula of multiple angles is presented as follows.
Teorema 4.1.4 Diberikan sudut α, maka rumus rumus sudut rangkap dapat ditunjukkan sebagai berikut.
1.
sin(2α) = 2 sin α cos α
(4.3)
2.
cos(2α) = cos2 α − sin2 α
2 tg α
tg (2α) =
1 − tg 2 α
(4.4)
3.
(4.5)
Bukti. For Equation 4.3
Bukti. Untuk Persamaan 4.3
sin(2α) = sin(α + α) = sin α cos α ± cos α sin α
= 2 sin α cos α.
We leave others for readers.
Pembuktian lainnya diserahkan kepada pembaca.
2
Theorem 4.1.7 Let α, β be any angles, the product formula of trigonometric function
is showed as follows:
Chapter 4. Trigonometry
68
Teorema 4.1.5 Diberikan sebarang sudut α and β, maka rumus perkalian fungsi trigonometri adalah sebagai berikut:
1.
2 sin α cos β = sin(α + β) + sin(α − β)
2.
2 cos α sin β = sin(α + β) − sin(α − β)
3.
2 cos α cos β = cos(α + β) + cos(α − β)
4.
−2 sin α sin β = cos(α + β) − cos(α − β)
Theorem 4.1.8 Let x, y be any angles, the formula of sum and difference of trigonometric functions are
Theorem 4.1.9 Diberikan sudut x and y, maka rumus penjumlahan dan pengurangan
fungsi trigonometri adalah:
1.
2.
3.
4.
1
1
sin x + sin y = 2 sin (x + y) cos (x − y)
2
2
1
1
cos x + cos y = 2 cos (x + y) cos (x − y)
2
2
1
1
sin x − sin y = 2 cos (x + y) sin (x − y)
2
2
1
1
cos x − cos y = −2 sin (x + y) sin (x − y)
2
2
(4.6)
(4.7)
(4.8)
(4.9)
Proof. Assuming that x = (α + β) and y = (α − β), and using Theorem4.1.5, we
can easily prove this theorem.
Bukti. Dengan memisalkan x = (α + β) dan y = (α − β), dan menggunakan
Teorema 4.1.5, maka teorema ini dapat dengan mudah dibuktikan.
Example. If 2 cos2 2x + sin( π2 − 2x) − 6 = 0 then cos x = . . .
Contoh. Jika 2 cos2 2x + sin( π2 − 2x) − 6 = 0 maka cos x = . . .
2
69
Chapter 4. Trigonometry
Solution.
Solution.
π
− 2x) − 6 = 0
2
2 cos2 2x + cos 2x − 6 = 0
2 cos2 2x + sin(
(2 cos2 2x − 3)(cos 2x + 2) = 0
3
cos 2x =
∨ cos 2x = −2
2
√
For cos 2x = 32 −→ 2 cos2 x − 1 = 32 −→ cos x = ± 12 5. For cos 2x = −2 −→
√
√
2 cos2 x − 1 = −2, the value of cos x is undefined. Thus cos x = 12 5 or − 12 5.
√
Untuk cos 2x = 32 −→ 2 cos2 x − 1 = 32 −→ cos x = ± 21 5. Untuk cos 2x = −2 −→
√
√
2 cos2 x − 1 = −2, nilai cos x tak terdefinisi. Sehingga cos x = 21 5 atau − 12 5.
4.1.3
Trigonometric Equation
Persamaan Trigonometri
The simple trigonometric equation can be solved by the following way:
Bentuk persamaan trigonometri sederhana dapat diselesaikan dengan cara berikut:
sin x = sin α =⇒ x1 = α + k · 360 ∨ x2 = (180o − α) + k · 360
cos x = sin α =⇒ x1 = α + k · 360 ∨ x2 = −α + k · 360
tg x = sin α =⇒ x1 = α + k · 180
ctg x = sin α =⇒ x1 = α + k · 180
To solve equation a cos x + b sin x = c, we should transform into equation cos(x −
α) = y first.
Sedangkan untuk menyelesaikan bentuk persamaan a cos x + b sin x = c, maka
70
Chapter 4. Trigonometry
harus ditransformasikan terlebih dahulu ke dalam bentuk cos(x − α) = y.
a cos x + b sin x = c
b
c
cos x + sin x =
a
a
sin α
c
b
sin α
sin x = , misal tg α = =
cos x +
cos α
a
a
cos α
c
cos x cos α + sin α sin x = cos α
a
c
a
cos(x − α) = cos α, dimana cos α = √
a
a2 + b2
(4.10)
So
Sehingga
a cos x + b sin x = c =⇒ cos(x − α) =
√ c
a2 +b2
Theorem 4.1.10 Equation of form a cos x + b sin x = c is solvable if c2 ≤ a2 + b2
Teorema 4.1.6 Bentuk persamaan a cos x + b sin x = c dapat diselesaikan apabila c2 ≤
a 2 + b2
From Equation (4.10), we have a cos x + b sin x = c =⇒ cos(x − α) =
value of cosine is −1 ≤ cos x ≤ 1, thus −1 ≤
2
2
√ c
a2 +b2
≤ 1 =⇒
c2
√ c
.
a2 +b2
a2 +b2
The
≤ 1 =⇒
2
c ≤ a + b . Bukti. Sesuai Persamaan (4.10), maka a cos x + b sin x = c =⇒
cos(x − α) =
−1 ≤
√ c
a2 +b2
√ c
.
a2 +b2
≤ 1 =⇒
Nilai cosinus terletak antara −1 ≤ cos x ≤ 1, sehingga
c2
a2 +b2
≤ 1 =⇒ c2 ≤ a2 + b2 .
2
Example. Determine the value of x satisfying tg x + ctg x = 4 cos 2x
Contoh. Tentukan harga x yang memenuhi persamaan tg x + ctg x = 4 cos 2x.
71
Chapter 4. Trigonometry
Solution.
Solusi.
tg x + ctg x = 4 cos 2x
sin2 x + cos2 x
= 4 cos 2x
cos x sin x
1 = 4 cos 2x cos x sin x
sin 90o = 2 cos 2x sin 2x = sin 4x
The last equation shows sin 4x = sin 90o −→ 4x1 = 90o + k · 360o ∨ 4x1 =
o
o
(180o − 90o ) + k · 360o . The value of x satisfying both of them {22 12 , 112 12 ,
o
o
202 12 , 292 12 }.
Dari persamaan terakhir sin 4x = sin 90o −→ 4x1 = 90o + k · 360o ∨ 4x1 =
o
o
(180o − 90o ) + k · 360o . Nilai x yang memenuhi keduanya adalah {22 21 , 112 12 ,
o
o
202 12 , 292 12 }.
4.2
Limit Fungsi
In mathematics, the limit of a function is a fundamental concept in calculus and
analysis concerning the behavior of that function near a particular input. Informally, a function assigns an output f (x) to every input x. The function has a
limit L at an input p if f (x) is ”close” to L whenever x is ”close” to p. In other
words, f (x) becomes closer and closer to L as x moves closer and closer to p.
More specifically, when f is applied to each input sufficiently close to p, the result is an output value that is arbitrarily close to L. If the inputs ”close” to p
are taken to values that are very different, the limit is said to not exist. Formal
definitions, first devised in the early 19th century, are given below.
Dalam matematika, limit suatu fungsi adalah merupakan konsep dasar kalkulus
dan analisis terkait dengan karakteristik fungsi di sekitar suatu input tertentu.
72
Chapter 4. Trigonometry
Secara informal, suatu fungsi akan memberikan output f (x) terhadap setiap input x. Fungsi itu akan memberikan sebuah nilai pendekatan L atas suatu input
p jika f (x) ”dekat” dengan L apabila x juga ”dekat” dengan p. Dengan kata
lain, f (x) menjadi dekat dan semakin dekat dengan L sebagaimana x bergerak
dekat dan semakin dekat dengan p. Secara sepesifik, saat f diterapkan terhadap
setiap input yang cukup dekat dengan p, hasilnya adalah sebuah output yang
juga cukup dekat dengan L. Jika input yang dekat dengan p dimasukkan dan
hasilnya sangat berbeda maka limit ini dikatakan tidak terdefinisi. Definisi formal, mucul pertama kali diawal abad 19-an, diberikan dalam berikut ini.
Definition 4.2.1 Intuitive Defintion of Limit. If f (x) is a real function and c is a
real number then:
lim f (x) = L,
x→c
that is f (x) can be sufficiently closer to L as x moves close to c.
Definisi 4.2.1 Pengertian limit secara intuitif. Jika f (x) adalah fungsi riil dan c
adalah bilangan riil, maka:
lim f (x) = L,
x→c
berarti f (x) dapat dibuat sedekat mungkin dengan L sebagaimana nilai x bergerak dekat
dengan bilangan c.
Theorem 4.2.1 Let f, g be functions whose limits exist for c. Let k, n be respectively
constant and positive real number. We have the followings.
73
Chapter 4. Trigonometry
Teorema 4.2.1 Misal f, g adalah fungsi-fungsi yang limitnya terdefinisi untuk c. Misal
k, n adalah masing-masing sebuah konstanta dan bilangan positif real. Maka akan
berlaku berikut ini.
1.
2.
3.
4.
5.
6.
7.
lim k = k and (dan) lim x = c
x→c
x→c
lim kf (x) = k lim f (x)
x→c
x→c
lim[f (x) ± g(x)] = lim f (x) ± lim g(x)
x→c
x→c
x→c
lim f (x) · g(x) = lim f (x) · lim g(x)
x→c
x→c
x→c
limx→c f (x)
f (x)
=
, provided (asalkan) lim g(x) 6= 0
x→c g(x)
x→c
limx→c g(x)
h
in
lim[f (x)]n = lim f (x)
x→c
x→c
q
p
n
lim f (x) = n lim f (x), provided (asalkan) lim f (x) ≥ 0
lim
x→c
x→c
x→c
for (untuk) n even (genap)
Definition 4.2.2 Newton’s difference quotient. Let f be a function. Function f 0
(read as ”f prime”) is the derivative of f whose value on any x is
f 0 (x) =
f (x + h) − f (x)
df (x)
= lim
h→0
dx
h
provided the limit exists.
Definition 4.2.3 Hasil beda Newton. Diketahui suatu fungsi f . Fungsi f 0 (dibaca
”f aksen”) adalah turunan dari f yang nilainya pada sebarang x adalah
f 0 (x) =
asalkan limit ini ada.
f (x + h) − f (x)
df (x)
= lim
h→0
dx
h
74
Chapter 4. Trigonometry
Example. If f (x) = x3 , then determine f 0 (x).
Contoh. Jika f (x) = x3 , maka tentukan f 0 (x).
Solution.
Solusi.
f (x + h) − f (x)
(x + h)3 − x3
= lim
h→0
h→0
h
h
2
2
3
3x h + 3xh + h
= lim
= lim (3x2 + 3xh + h2 )
h→0
h→0
h
2
= 3x .
f 0 (x) = lim
We expect the readers to remember the formula of derivative function for algebraic, exponential or trigonometry. Expressing the derivative definition is only
to show the connection between derivative concept and the solution of limit of
function. This module does not mean to discuss the concept of derivative in
detail. In the followings, we describe the solution of limit by using a derivative
concept.
Pembaca diharapkan dapat mengingat rumus-rumus turunan fungsi aljabar,
eksponensial dan trigonometri. Pencantuman definisi turunan dalam bagian
ini hanya untuk menggambarkan kaitan konsep turunan dengan penyelesaian
limit fungsi. Sehingga modul ini tidak dimaksudkan untuk membahas konsep
turunan secara detail. Berikut ini diberikan metode penyelesaian limit fungsi
dengan memanfaatkan konsep turunan ini.
In general, if f (x) = xn the the derivative function f 0 (x) is:
Secara umum, jika f (x) = xn maka fungsi turunan f 0 (x) adalah:
f 0 (x) =
d n
x
dx
= nxn−1
75
Chapter 4. Trigonometry
4.2.1
Solution Techniques
Metode Penyelesaian
Some techniques in solving limit have been developed. One of the most popular
is by recognizing an undefined form of function y = F (x) when an input x → p is
, ∞ ± ∞, ∞ × 0, 00 , ∞0 , ∞∞ .
substituted. Those undefined forms are 00 , ∞ × ∞, ∞
∞
Beberapa cara menyelesaikan limit fungsi telah dikembangkan, salah satu yang
paling populer yaitu dengan cara mengetahui bentuk takterdefinisi dari funggsi
y = F (x) apabila masukan x → p disubstitusikan. Bentuk-bentuk takterdefinisi
∞
itu adalah: 00 , ∞ × ∞, ∞
, ∞ ± ∞, ∞ × 0, 00 , ∞0 , ∞∞ .
Based on the undefined forms, we can solve the limit of function by the following principles.
Berdasarkan bentuk tak terdefinisi di atas maka dalam menyelesaikan limit fungsi
dapat dilakukan dengan cara berikut.
1. Forms (Bentuk)
0
0
∞
.
∞
and (dan)
f (x)
f 0 (x)
= lim 0
x→c g(x)
x→c g (x)
(4.11)
lim
2. Form (Bentuk) ∞ × 0.
1
f (x)
1
f (x)
lim f (x) · g(x) = lim(f (x) · g(x)) ×
x→c
x→c
such that we get forms
0
0
or
= lim
x→c
g(x)
1
f (x)
(4.12)
∞
.
∞
sedemikian hingga diperoleh bentuk
0
0
atau
∞
.
∞
3. Form Bentuk ∞ ± ∞.
lim[f (x) ± g(x)] = lim
x→c
x→c
1
g(x)
±
1
f (x)
1
f (x)·g(x)
(4.13)
76
Chapter 4. Trigonometry
such that we get forms
0
0
or
∞
∞
sedemikian hingga diperoleh bentuk
0
0
atau
∞
.
∞
4. When we do not end up with the three forms, we can solve it with simplifying the function by using special techniques and mathematics properties,
such as factorization, identity formulas etcetera. For the limit of trigonometric function can be the properties of trigonometry in Chapter 3
Apabila tidak diperoleh ketiga bentuk di atas maka dapat diselesaikan
dengan menyederhanakan fungsi itu dengan menerapkan teknik-teknik
khusus dan sifat-sifat matematis, seperti pemfaktoran, rumus identitas
dan lain sebagainya. Untuk fungsi trigonometri dapat menggunakan sifatsifat trigonometri pada Bab 3.
4.2.2
Limit of Algebraic Function
Limit Fungsi Aljabar
Theorem 4.2.2 Given that a function f (x) = a0 xm + a1 xm−1 + a2 xm−2 + · · · + am
(x)
is
and g(x) = b0 xn + b1 xn−1 + b2 xn−2 + · · · + bn . So limx→∞ fg(x)
Teorema 4.2.2 Diketahui suatu fungsi f (x) = a0 xm + a1 xm−1 + a2 xm−2 + · · · + am
(x)
adalah
dan g(x) = b0 xn + b1 xn−1 + b2 xn−2 + · · · + bn maka limx→∞ fg(x)



a0
,
b0
jika
=
0, jika


∞, jika
m = n,
m < n,
m > n.
(4.14)
Proof. We leave for a group discussion. (Hint. When m = n, divide f (x), g(x)
by xm = xn ; when m < n, divide f (x), g(x) by xn ; and when m > n, divide
Chapter 4. Trigonometry
77
f (x), g(x) by xm ).
Bukti. Sebagai bahan diskusi kelompok. (Petunjuk. Bila m = n maka bagi
f (x), g(x) dengan xm = xn ; bila m < n bagi f (x), g(x) dengan xn ; dan bila m > n
maka bagi f (x), g(x) dengan xm ).
2
Theorem 4.2.3 Given that a function f (x) = a0 xm + a1 xm−1 + a2 xm−2 + · · · + am
(x)
and g(x) = b0 xn + b1 xn−1 + b2 xn−2 + · · · + bn . So limx→0 fg(x)
is
Teorema 4.2.3 Diketahui suatu fungsi f (x) = a0 xm + a1 xm−1 + a2 xm−2 + · · · + am
(x)
dan g(x) = b0 xn + b1 xn−1 + b2 xn−2 + · · · + bn , maka limx→0 fg(x)
adalah

a0

 b0 , jika m = n,
=
(4.15)
∞, jika m < n,


0, jika m > n.
Theorem 4.2.4 Given that a function
p
√
f (x) = ax2 + bx + c and g(x) = px2 + qx + r.
So limx→∞ [f (x) − g(x)] is
Teorema 4.2.4 Diketahui suatu fungsi
p
√
f (x) = ax2 + bx + c dan g(x) = px2 + qx + r,
maka limx→∞ [f (x) − g(x)] adalah

b−q √

 2a a, jika m = n,
=
−∞,
jika a < p,


+∞,
jika a > p.
(4.16)
78
Chapter 4. Trigonometry
Proof.
Bukti.
i √ax2 + bx + c + ppx2 + qx + r
p
= lim
ax2 + bx + c − px2 + qx + r · √
x→∞
ax2 + bx + c + px2 + qx + r
(b − q)x + (c − r)
p
= lim √
, karena a = p
2
x→∞
ax + bx + c + px2 + qx + r
h√
=
p
(b − q) + (c−r)
x
p
b
c
a + x + x2 + p +
lim q
x→∞
q
x
+
r
x2
(b − q) √
a.
2a
h√
i
p
Hence limx→∞
ax2 + bx + c − px2 + qx + r =
=
(b−q) √
a.
2a
and a < p, we leave
h√for a group discussion.
i
p
Sehingga limx→∞
ax2 + bx + c− px2 + qx + r =
When the case a > p
(b−q) √
a.
2a
Untuk kasus a > p
dan a < p dapat digunakan sebagai bahan diskusi kelompok.
2
Apart from the three cases, in general we can solve the limit of algebraic function
by referring the methods in Subsection 4.2.1.
Diluar tiga kasus di atas, secara umum limit fungsi aljabar dapat diselesaikan
dengan mengacu pada metode-metode yang terdapat subbab 4.2.1.
x2 +(3−a)x−3a
x−a
x2 +(3−a)x−3a
limx→a
x−a
Example. Determine limx→a
Contoh. Tentukan
Solution. Case 4.11, we have
Solution. Kasus 4.11, maka
x2 + (3 − a)x − 3a
2x + (3 − a)
= lim
x→a
x→a
x−a
1
= a+3
lim
√
√
x3 +2x2 c+3xc2 −5− 6c3 −5)
x−c √
√
x3 +2x2 c+3xc2 −5− 6c3 −5)
limx→c
x−c
Example. Determine limx→c
Contoh. Tentukan
79
Chapter 4. Trigonometry
Solution. Case 4.11, we have
Solusi. Kasus 4.11, maka
√
√
x3 + 2x2 c + 3xc2 − 5 − 6c3 − 5)
3x2 + 4xc + 3c2
lim
= lim √
x→c
x→c 2 x3 + 2x2 c + 3xc2 − 5
x−c
√
c 11
= √
2 6c3 − 5
4.2.3
Limit of Trigonometric Function
Limit Fungsi Trigonometri
Theorem 4.2.5 The followings hold for limit of trigonometry
Teorema 4.2.5 Berikut ini berlaku untuk limit trigonometri
sin x
x
= lim
=1
x→0 x
x→0 sin x
sin ax
a
lim
=
x→0
bx
b
tan ax
a
lim
=
x→0
bx
b
sin ax
a
lim
=
x→0 tan bx
b
lim
Theorem 4.2.6 The followings hold for limit of trigonometry
(4.17)
(4.18)
(4.19)
(4.20)
80
Chapter 4. Trigonometry
Teorema 4.2.6 Berikut ini berlaku untuk limit trigonometri
sinn x
xn
lim
=
lim
=1
x→0 xn
x→0 sinn x
an
sinn ax
=
lim
x→0 bxn
b
tann ax
an
lim
=
x→0
bxn
b
an
sinn ax
lim
= n
x→0 tann bx
b
(4.21)
(4.22)
(4.23)
(4.24)
Proof. Case (4.22) can be proved by the following way:
Bukti. Kasus (4.22) dapat dibuktikan dengan cara berikut:
sinn ax
(sin ax)n
=
lim
x→0 bxn
x→0
bxn
³
1
sin ax ´n
=
lim
b x→0 x
an
=
, respecting to (berdasarkan) Theorem (Teorema) 4.2.6
b
lim
For the next proof, we leave for an exercise.
Untuk pembuktian selanjutnya diberikan sebagai bahan latihan.
2
Likely the limit of algebraic function, the method for solving trigonometric function is relatively the same. But, Limit of trigonometric function in general has
undefined output when the input is taken.
Sebagaimana limit fungsi aljabar metode penyelesaian limit fungsi trigonometri
dapat dilakukan dengan cara yang sama. Namun demikian pada umumnya
limit fungsi trigonometri mempunyai bentuk takterdefinisi apabila inputnya digantikan.
sin 8x−sin 4x
7x
sin 8x−sin 4x
limx→0
7x
Example. Determine limx→0
Contoh. Tentukan
Chapter 4. Trigonometry
81
Solution. Equation (4.8), so the solution can be obtained in two ways. First, we
should understand the properties of trigonometric function.
Solusi. Persamaan (4.8), maka penyelesaian dapat dilakukan dengan dua cara,
pertama dengan memahami sifat-sifat fungsi trigonometri berikut.
lim
x→0
sin 8x − sin 4x
2 cos 6x sin 2x
= lim
x→0
7x
7x
sin 2x
= lim 2 cos 6x ·
x→0
7x
sin 2x
= lim 2 cos 6x · lim
x→0
x→0 7x
4
=
7
Secondly, we should consider that this problem meets Case (4.11), so
Cara kedua adalah dengan mempertimbangkan bahwa soal ini termasuk kasus
(4.11), sehingga
sin 8x − sin 4x
8 cos 8x − 4 cos 4x
= lim
x→0
x→0
7x
7x
4
=
7
lim
(It must be more efficient!)
(Tentu ini lebih efisien!)
P ROBLEMS
AND
S OLUTIONS
S OAL - SOAL DAN P EMBAHASAN
1. Let α, β and γ be angles of a triangle. Determine sine and cosine the angles
α + β, α + γ and β + γ
Misal α, β dan γ adalah sudut-sudut dalam segitiga. Tentukan sinus dan
cosinus dari sudut-sudut α + β, α + γ dan β + γ
82
Chapter 4. Trigonometry
2. Solution. The sum of all triangle angles is α + β + γ = 180o . Consider
the angle (X o − α), the acronyms ASTC and BTBT such that sin(α + β) =
sin(180o − γ) = sin γ. For other problems we leave for exercise.
Solusi. Jumlah seluruh sudut segitiga α + β + γ = 180o . Ingat sudut
(X o − α), ASTC dan BTBT, maka untuk sin(α + β) = sin(180o − γ) = sin γ.
Selanjutnya untuk pertanyaan lainnya diberikan sebagai latihan.
sin 5x+sin 3x
= tg 4x
cos 5x+cos 3x
sin 5x+sin 3x
=
bahwa cos
5x+cos 3x
3. Prove that
Buktikan
tg 4x
Solution. Ordinary way. From the formula above:
Solusi. Cara biasa. Sesuai dengan rumus di atas:
1
1
sin A + sin B = 2 sin (A + B) cos (A − B)
2
2
1
1
cos A + cos B = 2 cos (A + B) cos (A − B)
2
2
Hence, we get
Sehingga diperoleh:
1
1
sin 5x + sin 3x = 2 sin (5x + 3x) cos (5x − 3x)
2
2
= 2 sin 4x cos x
1
1
cos 5x + cos 3x = 2 cos (5x + 3x) cos (5x − 3x)
2
2
= 2 cos 4x cos x
Therefore
Dengan demikian
2 sin 4x cos x
sin 5x + sin 3x
=
cos 5x + cos 3x
2 cos 4x cos x
= tg 4x
83
Chapter 4. Trigonometry
4. Prove that sin 54o × sin 18o =
1
4
Buktikan bahwa sin 54o × sin 18o =
Solution. If we multiple by
2 cos 18o
2 cos 18o
1
4
then we will obtain
Solusi. Jika kita mengalikan dengan
2 cos 18o
2 cos 18o
maka kita akan mendapat
(2 sin 18o cos 18o ) sin 45o
sin 36o cos(90o − 54o )
=
2 cos 18o
2 sin(90o − 18o )
sin 36o cos 36o
sin 36o cos 36o
=
=
2 sin 72o
2 sin(36o + 36o )
o
o
sin 36 cos 36
1
=
=
o
o
2(2 sin 36 cos 36 )
4
sin 45o sin 18o =
1
5. Determine limx→1 ( x4 −2x+1
−
1
Tentukan limx→1 ( x4 −2x+1
−
1
)
x7 −1
1
)
x7 −1
Solution. Case category 4.13, thus
Solusi. Kategori kasus 4.13, maka
lim (
x→1
1
1
(x4 − 2x + 1) − (x7 − 1)
−
)
=
lim
x→1 (x4 − 2x + 1)(x7 − 1)
x4 − 2x + 1 x7 − 1
refer to Case 4.11, we have
Menjadi kategori kasus 4.11, maka
(x4 − 2x + 1) − (x7 − 1)
(4x3 − 2 − 7x6 )
=
lim
x→1 (x4 − 2x + 1)(x7 − 1)
x→1 (4x3 − 2)(x7 − 1) + 7(x4 − 2x + 1)x6
= ∞
lim
5x5 −x3 +2x2 −x+14
13x5 −5
5x5 −x3 +2x2 −x+14
limx→∞
13x5 −5
6. Determine limx→∞
Tentukan
5x5 −x3 +2x2 −x+14
13x5 −5
5x5 −x3 +2x2 −x+14
limx→∞
13x5 −5
Solution. Case 4.14, thus limx→∞
=
Solusi. Kasus 4.14, maka
q
7 −x9 +243x12
7. Determine limx→0 3 −18−5x
7−x+9x5 +9x12
q
7 −x9 +243x12
Tentukan limx→0 3 −18−5x
7−x+9x5 +9x12
=
5
13
5
13
84
Chapter 4. Trigonometry
Solution. Case 4.15, applying Theorem 4.2.1, thus we have
Solusi. Kasus 4.15, dengan menerapkan Teorema 4.2.1 maka diperoleh
r
r
7
9
12
−18 − 5x7 − x9 + 243x12
3
3 −18 − 5x − x + 243x
lim
=
lim
x→0
x→0
7 − x + 9x5 + 9x12
7 − x + 9x5 + 9x12
r
3 243
=
=3
9
√
£√
¤2
8. Determine limx→∞ 16x2 + 6x + 7 − 16x2 + 10x + 6
√
¤2
£√
Tentukan limx→∞ 16x2 + 6x + 7 − 16x2 + 10x + 6
Solution. Case 4.15, applying Theorem 4.2.1, thus we have
Solusi. Kasus 4.15, dengan menerapkan Teorema 4.2.1 maka diperoleh
√
£√
¤2
lim
16x2 − 5x + 7 − 16x2 + 10x + 6
x→∞
√
√
£
¤2
=
lim 16x2 − 5x + 7 − 16x2 + 10x + 6
x→∞
£ 6 − 10 √ ¤2 1
=
16 =
2 × 16
4
9. Obtain the value of the following limits
Tentukan harga limit berikut ini:
2x
x→0 sin 3x
sin 8x
(iii) lim
x→0 tg 5x
(i) lim
tg 5x
x→0 6x
3 tg 2x
(iv) lim
x→0 sin 12x
(ii) lim
Solution. From Theorem 4.2.6, thus (i) = 23 ; (ii) = 65 ; (iii) = 85 ; (iv) = 12 .
Solution. Sesuai Teorema 4.2.6, maka (i) = 32 ; (ii) = 65 ; (iii) = 85 ; (iv) =
1
.
2
1
5x+x2
1
5x+x2
10. Determine limx→0 cossec 4x −
Tentukan limx→0 cossec 4x −
Solution. Case 4.14, thus
85
Chapter 4. Trigonometry
Solusi. Kategori kasus 4.13, maka
lim cossec 4x −
x→0
1
(5x + x2 ) − sin 4x
=
lim
x→0 (5x + x2 ) sin 4x
5x + x2
(5 + 2x) − cos 4x
= lim
x→0 (5 + 2x) sin 4x + 4(5x + x2 ) cos 4x
4
=
=∞
0
sin3 3x−sin3 3x cos 2x
4x2 tg 3 9x
sin3 3x−sin3 3x cos 2x
limx→0
4x2 tg 3 9x
11. Determine limx→0
Tentukan
Solution. Referring Theorem 4.14, thus
Solusi. Mengacu pada Teorema 4.2.6, maka
sin3 3x − sin3 3x cos 2x
sin3 3x(1 − cos 2x)
=
lim
x→0
x→0
4x2 tg 3 9x
4x2 tg 3 9x
sin3 3x 2 sin2 x
= lim
·
x→0 4 tg 3 9x
x2
1
=
54
lim
¥¥¥¥¥¥
CHAPTER 5
Geometri
5.1
Segitiga
Definition 5.1.1 Poligon adalah gabungan himpunan titik-titik P1 , P2 , . . . , Pn dengan ruas garis P1 P2 , P2 P3 , . . . , Pn−1 Pn , Pn P1 sedemikian hingga jika dua sebarang ruas
garis berpotongan maka titik potongnya adalah salah satu dari titik-titik P1 , P2 , . . . , Pn .
Selanjutnya P1 , P2 , . . . , Pn disebut titik-titik sudut poligon dan P1 P2 , P2 P3 , . . . , Pn−1 Pn , Pn P1
disebut sisi-sisi poligon.
Definition 5.1.2 Transversal adalah suatu garis yang memotong dua garis lain di dua
titik yang berlainan.
Theorem 5.1.1 Jika dua garis sejajar dipotong oleh transversal maka sudut-sudut sehadapnya, bersebrangan luarnya dan bersebrangan dalamnya kongruen.
f
4
3
8
7
1
2
5
A
g
γ
β
h
6
b
c
B
(i)
C
(ii)
Figure 5.1:
86
87
Chapter 4. Geometri
Definition 5.1.3 Segitiga adalah poligon yang bersisi tiga.
Lema 5.1.1 Beberapa hal yang dipenuhi segitiga (lihat Gambar 5.1) adalah:
1. Jumlah semua sudut segitiga adalah 180o
2. Jika AB = AC maka ∠B = ∠C, dan jika AB = AC = BC maka ∠A = ∠B =
∠C.
Bukti. Bukti nomor 2. Perhatikan Gambar 5.1 (ii), sin α =
karena b = c maka α = γ.
t
c
dan sin γ =
t
b
2
GEOMETRI RIRIS
Pada bab ini akan dibahas mengenai teknik untuk menyelesaikan masalah-masalah
geometri. Pada umumnya, banyak masalah segi banyak yang dapat diselesaikan dengan mengunakan konsep segitiga dan lingkaran. Oleh sebab itu, kali
ini akan dibahas hal-hal penting yang perlu diketahui menegenai segitiga dan
lingkaran.
Sebelumnya perhatikan definisi berikut:
Definisi
Diberikan dua titik A dan B. Garis yang menghubungkan kedua titik ini kemudian disebut garis AB dan dari dua titik ini akan terbentuk sebuah segmen
garis dengan panjang tertentu. Selanjutnya segmen berarah AB dan BA mempunyai panjang yang sama tetapi arahnya berlawanan. Jarak dari titik A dan B
kemudian dinotasikan AB.
5.2
Segitiga
Beberapa hal penting dalam sebuah segitiga ABC yaitu
88
Chapter 4. Geometri
A
α
B
β
γ
C
1. Jumlah ketiga sudutnya adalah 1800
2. Aturan sinusnya
sin α
sin β
sin γ
=
=
BC
AC
AB
3. Aturan kosinus BC 2 = AB 2 + AC 2 − 2.AB.AC cos α
Definisi kesebangunan:
R
γ
β Q
C
γ
β
A
α
α
B P
Dua buah segitiga ABC dan P QR dikatakan sebangun jika dan hanya jika mempunyai sudut-sudut bersesuaian yang sama besar atau sisi-sisi yang bersesuaian
mempunyai perbandingan yang sama pula.
AB
PQ
=
BC
QR
=
CA
RP
Teorema:
Luas segitiga-segitiga yang tingginya sama sebanding dengan alas-alasnya.
89
Chapter 4. Geometri
1. Pada segitiga ABC, M terletak pada sisi AB sehingga AM : M B = 1 : 3
dan N pada sisi AC sehingga AN : AC = 3 : 5. berapakah
L4M N C
L4ABC
(Soal kompetensi matematika XVIII Jakarta)
Jawab:
A
1
3
3
5
C
B
1
bc sin A
2
= L4AM C − L4AM N
1 1 3
1 1
. c.b sin A − . c. b sin A
=
2 4
2 4 8
5
=
bc sin A
64
L4ABC =
L4M N C
Jadi, didapatkan
L4M N C
L4ABC
1
bc sin A
2
5
bc sin A
64
2. Tentukan perbandingan luas antara daerah yang diarsir dan yang tidak
diarsir pada gambar berikut ini!
Jawab:
Perhatikan gambar!
Dari gambar tersebut jelas terlihat bahwa perbandingan antara daerah yang
diarsir dengan daerah yang tidak diarsir 1 : 1.
Garis-garis yang kongruen dan titik kolinear
90
Chapter 4. Geometri
Teorema: CEV A
Jika titik D, E dan F terletak pada sisi-sisi BC, CA dan AB dari 4ABC sedemikian
hingga garis AD, BE dan CF kongruen melalui titik 0, maka
BD CE AF
.
.
=1
DC EA F B
Bukti:
C
E
A
O
F
D
B
91
Chapter 4. Geometri
Pada 4ABD dan 4ADC mempunyai tinggi yang sama, demikian halnya dengan 4OAB dan 4ODC. Menurut teorema sebelumnya diperoleh
BD
4ABD
4OBD
4ABD − 4OBD
4ABO
=
=
=
=
DC
4ADC
4ODC
4ADC − 4ODC
4CAO
Kemudian pada 4CBE dan 4ABE mempunyai tinggi yang sama, demikian
halnya dengan 4COE dan 4AOE. Menurut teorema sebelumnya diperoleh
CE
4CBE
4COE
4CBE − 4COE
4BCO
=
=
=
=
EA
4ABE
4AOE
4ABE − 4AOE
4ABO
Dengan cara yang sama, diperoleh pula
AF
FB
=
4CAO
4BCO
Dari sisi didapatkan
BD CE AF
4ABO 4BCO 4CAO
.
.
=
.
.
=1
DC EA F B
4CAO 4ABO 4BCO
Terbukti
Contoh:
D, E dan F adalah titik-titik pada sisi BC, CA dan AB dari 4ABC dan AD, BE
dan CF kongruen terhadap titik O.
Tunjukkan bahwa
OD
AD
+
OE
BE
+
OF
CF
= 1!
Jawab:
Perhatikan gambar :
C
E
A
O
F
D
B
Menurut teorema CEV A didapatkan
L4ODB
L4ODC
L4ODB + L4ADC
L4
DO
=
=
=
=
AD
L4ADB
L4ADC
L4ADB + L4ADC
L4
92
Chapter 4. Geometri
Kemudian dengan cara yang sama didapatkan pula
OE
L4OCA
OF
L4OAB
=
dan
=
BE
L4ABC
CF
L4ABC
Dari ketiga persamaan diatas, maka
OD OE OF
L4OBC L4OCA L4OAB
+
+
=
+
+
=1
AD BE CF
L4ABC
L4ABC L4ABC
Teorema berikut merupakan kebalikan dari teorema CEV A
Teorema:
Jika tiga titik D, E dan F terletak pada sisi-sisi 4ABC yaitu di BC, CA dan AB
sedemikian hingga berlaku
BD CE AF
. .
DC EA F B
= 1, maka AD, BE dan CF adalah kon-
gruen.
Bukti:
Misalkan AD dan BE berpotongan di O, perpanjangan CO akan memotong AB
di F 0 . Karena AD, BE dan CF 0 kongruen maka
karena diketahui pula
dapatkan
AF
FB
=
AF 0
F 0B
AF 0
⇔
AB
FB
AB
FB
−1=
AF
FB
AF
FB
=
=
F 0B
AF 0
F 0B
⇔
=
⇔
bahwa BD
. CE . AF
DC EA F B
0B
AB−F B
= AB−F
FB
F 0B
AB
−1
F 0B
BD CE AF 0
. .
DC EA F 0 B
=1
= 1, maka dari dua persamaan ini di-
AB
F 0B
Dari sini dapat dilihat bahwa F B = F 0 B, sehingga F dan F 0 berhimpit. Jadi
titik O juga dilalui CF , sehingga terbukti AD, BE dan CF kongruen.
Contoh
Buktikan bahwa semua garis tinggi dalam 4ABC adalah kongruen.
Jawab:
Misalkan AD, BE dan CF adalah garis 4ABC. Sehingga BE⊥CA dan CF ⊥AB.
Diperoleh pula CF ⊥AB. Diperoleh pula 4AEB dengan 4AF C dengan ∠A
AE
= AB
. Dengan cara yang
AF
AC
CA
AB BC CA
sehingga BD
. CE . AF = BC
. CA . AB =
CB
DC EA F B
merupakan sudut persekutuan. Sebagai akibatnya
BE
sama jika AD⊥BC, BD
=
1 Terbukti
BC
BA
dan
CD
CE
=
93
Chapter 4. Geometri
Berikut adalah definisi dari garis transfersal sisi dan transfersal sudut dari 4ABC.
Definisi:
C
l
A
B
k
Diberikan 4ABC. Garis lurus k disebut sebagai transversal sisi jika k memotong sisi-sisi atau perpanjangan sisi 4ABC. Garis lurus ` disebut sebagai
transversal sudut jika ` melalui titik sudut 4ABC.
Teorema:Manelaos
Jika sebuah garis tranversal sisi memotong BC, CA dan AB dari 4ABC pada
titik-titik D, E dan F , maka
BD CE AF
. .
DC EA F B
= −1
Perhatikan gambar di bawah ini.
A
P
F
B
E
C
D
Karena 4AP F dan 4BDF sebangun maka FAFB =
sebangun dengan 4AP F
Karena P A = −AP maka
maka DC
= CE
.
AP
EA
berlaku BD
. CE . AF
DC EA F B
=
PA
.
BD
Berikutnya karena 4CDE
BD DC P A
. .
DC AP BD
= −1 Terbukti
94
Chapter 4. Geometri
Teorema berikut merupakan kebalikan dari teorema Menelaus.
Teorema:
Jika titik-titik D, E dan F terletak pada sisi-sisi 4ABC sedemikian hingga berlaku
BD CE AF
.
DC EA F B
= −1, maka D, E dan F kolinear(segaris).
Misal perpotongan DE terhadap AB adalah F 0 . Menurut Teorema Menelaus
diperoleh
BD CE AF
.
DC EA F B
= −1 dari sini didapatkan
AF 0
F 0B
=
AF
,
FB
akibatnya F dan F 0
berimpit dan F terletak di DE. Jadi, terbukti D, E dan kolinear.
A
F
E
C
B
D
Contoh:
Sebuah garis transversal memotong sisi-sisi AB, BC, CD dan DA dari segiempat ABCD di P, Q, R, dan S berturut-turut.
Buktikan bahwa
AP BQ CR DS
.
.
P B QC RD SA
=1
Jawab:
A
B
Q
D
x
S
R
C
P
Menurut teorema Manelaus dari gambar di samping, pada 4ABC didapatkan
AP BQ CX
. .
P B QC XA
= 1....1
95
Chapter 4. Geometri
CX
Kemudian pada 4ACD didapatkan CX
. AS . DR = −1, sehingga
= − DS
. CR ....2
XA SD RC
XA ¶
SA RD
µ
Jika (2) disubstitusikan ke (1), akan didapatkan
Ekivalen dengan
AP BQ DS CR
. . .
P B QC SA RD
AP BQ
. .
P B QC
−
DS CR
.
SA RD
= −1
= 1. Terbukti.
Teorema Phytagoras merupakan bentuk khusus dari aturan cosinus dengan sudut
α = 900 .
Teorema:Phytagoras
B
c
a
C
b
A
Pada sebuah segitiga siku-siku, maka kuadrat hypotenuse sama dengan jumlah
kuadrat dua sisi yang lain.
Bukti:
Teorema di atas sama dengan mengatakan bahwa kuadarat panjang sisi terpanjang sama dengan jumlah kuadrat sisi yang lainnya.
Untuk segitiga ABC siku-siku dengan siku-siku di C. Misalnya panjang AB
adalah c, panjang BC adalah a dan panjang AC adalah b. Akan ditunjukkan
bahwa c2 n = b2 + a2 .
Dibentuk persegi dengan panjang sisi a + b seperti pada gambar berikut
Dari gambar di atas, jelas bahwa luas ABCD = 4x luas 4AP S+ luas P QRS.
Sehingga didapatkan
µ
¶
(a + b)2 =
4x 12 (axb) + c2
a2 + 2ab + b2 = 2ab + c2
a2 + b2 = c2 . Terbukti.
Contoh:
96
Chapter 4. Geometri
D b
a
c
c
c
c
S
b
a
R
A a
P
C
b
Q
a
b
B
Buktikan teorema Apollonius berikut.Jika D adalah titik tengah BC pada 4ABC,
maka berlaku AB 2 + AC 2 = 2AD2 + 2DC 2 !
Jawab:
Misalkan ∠BDA = α0 maka ∠CDA = (180 − α)0 . dari segitiga ABD, didapatkan aturan cosinus sebagai berikut:
AB 2 = BD2 + AD2 − 2BD.AD. cos α
.....1
Kemudian dari segitiga ACD, didapatkan aturan cosinus sebagai berikut:
AC 2 = DC 2 + AD2 − 2DC.AD. cos(180 − α)0
AC 2 = DC 2 + AD2 − 2DC.AD. cos α0
.....2
Jika persamaan 1 dan 2 dijumlahkan maka didapatkan
AB 2 + AC 2 = DC 2 + BD2 + 2AD2 + DC 2
Karena D merupakan titik tengah BD maka panjang BD sama dengan panjang
DC sehingga diperoleh
AB 2 + AC 2 = 2AD2 + 2DC 2 Terbukti.
5.3
Lingkaran
Definisi:
Lingkaran adalah himpunan semua titik yang mempunyai jarak r terhadap titik O. r
kemudian disebut degan jari-jari dan O disebut dengan titik pusat.
97
Chapter 4. Geometri
Beberapa sifat penting dalam lingkaran adalah:
1. Sudut keliling yang menghadap busur yang sama maka besarnya sama
pula.
α
α
2α
α
2. Sudut pusat yang menghadap busur yang sama dengan sudut keliling,
besarnya dua kali sudut keliling.
3. Sudut-sudut yang berhadapan dari segiempat tali busur saling berpelurus.
E
180 − a
D
O
a
F
A
C
B
4. Jika garis AB menyinggung lingkaran di titik C, maka untuk setiap tali
busur CD melalui titik C diperoleh ∠ACD = ∠CED dan ∠BCD = ∠CF D
98
Chapter 4. Geometri
B
400
O
S
C
A
Contoh:
Diketahui segitiga ABC dengan sisi-sisi AB, CAdanCB masing-masing menyinggung lingkaran yang pusatnya O. Jika ∠ACB = 400 , tentukan ∠AOB!
Jawab:
Misalkan R, S dan T adalah titik singgung lingkaran. Karena ∠ACB = 400 ,
maka ∠ACB + ∠CAB = 1400 . Akibatnya ∠T AS = 1800 − ∠CAB dan ∠RBS =
1800 − ∠CBA. Dari sini didapatkan ∠T AS − ∠RBS = 3600 − 1400 = 2200
Teorema: Talibusur
B
D
P
A
C
Jika AC dan BD adalah taibusur-talibusur sebuah lingkaran yang berpotongan
di dalam lingkaran dititik P , maka P A.P C = P B.P D.
Bukti:
Perhatikan dua buah segitiga 4AP Bdan4CP D. Karena ketiga sudut yang bersesuaian sama besarnya yaitu maka dua segitiga tersebut adalah sebangun. Akibatnya diperoleh
PA
PD
=
PB
.
PC
Terbukti bahwa P A.P C = P B.P D
99
Chapter 4. Geometri
P
A
B
A0
B0
Contoh:
Diketahui AA0 dan BB 0 adalah talibusur-talibusur sebuah lingkaran yang berpotongan di titik P diluar lingkaran. Tunjukkan bahwa P A.P A0 = P B.P B 0 !
Jawab:
Perhatikan bahwa ∠A0 P B dan ∠B 0 P A adalah sudut yang menghadap busur
yang sama, akibatnya ∠A0 P B = ∠B 0 P A. Demikian pula halnya dengan ∠A0 P B =
∠B 0 P A. Demikian pula halnya dengan ∠A0 P B = ∠B 0 P A. Dari sini diperoleh
dua buah segitiga yang kongruen yaitu 4P A0 B dan 4P B 0 A, sehingga diperoleh
P A0
P B0
=
PB
,
P A0
ekivalen dengan P A.P A0 = P B.P B 0 Terbukti
Persamaan tersebut sering pula disebut Teorema Secant.
Teorema: Secant-Tangent
Jika P adalah sebuah titik diluar lingkaran dan garis singgung dari P menyinggung lingkaran di titik T dan 2 garis melalui memotong A di A0 dan memotong
B di B 0 maka
P A.P A0 = P B.P B 0 = P T 2
Bukti:
Misal ∠P A0 T = α. Karena AOT menghadap busur yang sama maka ∠AOT =
2α. Perhatikan 4AOT adalah segitiga sama sisi, sehingga sudut kakinya mempunyai besar yang sama. Dari sini didapatkan ∠OT A = 900 − α. Karena
100
Chapter 4. Geometri
P
α
A
T
O
A0
P T adalah garis singgung, maka didaptkan ∠P T A = α. Bandingkan kedua
segitiga 4P A0 T dan 4P T A. Karena mempunyai sudut-sudut yan besarnya
sama, 4P T A. Karena mempunyai sudut-sudut yang besarnya sama, maka kedua segitiga ini adalah sebangun. Jadi diperoleh
0
PT
P A0
=
PA
.
PT0
Ekivalen dengan
2
P A.P A = P T . Terbukti.
Contoh:
Jika panjang P T = 6cm, SQ = 2, 5cm. Berapkah panjang T Q
R
Q
O
T
P
Jawab:
Karena ∠OSR = ∠OSQ maka didapatkan dua buah segitiga yang kongruen sehingga QR = 2SQ = 2x2, 5cm = 5cm. Menurut teorema Secant-Tangent diper-
101
Chapter 4. Geometri
oleh
T Q.T R = T P 2
T Q.(T Q + 5) = 62
T Q2 + 5T Q − 36 = 0
(T Q + 9)(T Q − 4) = 0
Karena panjang tidak mungkin bernilai negatif maka nilai T Q yang tidak memenuhi
adalah 4.
SOAL-SOAL DAN PEMBAHASAN
Bagian 1
1. Tentukan perbandingan luas antara daerah yang diarsir dengan daerah
yang tidak diarsir pada gambar berikut!
(Soal kompetensi matematika XVII Jakarta)
Pembahasan:
√
Perhatikan gambar segitiga kecil. Sisi-sisi segitiga tersebut adalah 1, 1, 2.
Dari sini diperoleh luas setiap segitiga kecil adalah 12 .1.1 =
1
2
satuan luas
dan luas sehingga daerah yang diarsir luasnya 8. 21 = 4 satuan luas. Kemudian luas daerah yang tidak diarsir adalah:
102
Chapter 4. Geometri
(2 +
√
2)2 - 4. 21
√
= 4+4 2+2−2
√
= 4(1 + 2)
2. Perhatikan gambar,segitiga ABC siku-siku di A dan D adalah pertengahan
BC. Titik F membagi dua sama panjang sisi AB, sedangkan titik E dan
G berturut-turut membagi AF dan F B menjadi dua bagian sama panjang.
Garis AD memotong garis hubung CE, CF dan CG berturut-turut dititik
P, Q dan R. Tentukan nilai perbandingan P Q : P R?
(soal kompetensi matematika XVII Jakarta)
C
D
QR
P
A E F G B
Pembahasan:
Untuk mempermudah, AD diperpanjang sampai titik H sedemikian hingga
ABHC merupakan persegi. Misalkan panjang AH = x. Perhatikan bahwa
4AEP dan 4HCP adalah dua segitiga yang sebangun. Akibatnya diperoleh
AP
HP
=
AE
HC
= 14 . Dari sini didapatkan AP = 51 AH. Jadi AP = 15 x. ke-
mudian selanjutny perhatikan pula bahwa 4AP Q dan 4HCQ sebangun,
demikian pula 4AGR dan 4HCR.Dengan cara yang sama, akan didap-
103
Chapter 4. Geometri
atkan AQ = 13 x dan AR = 37 x. Karena
P Q = AQ − P Q
1
1
=
x− x
3
5
2
=
x
15
dan
P R = AR − AP
3
1
=
x− x
7
5
8
=
x
35
Didapatkan P Q : P R =
2
x
15
:
8
x
35
= 7 : 12
3. Keliling sebuah segitiga sama sisi adalah p. Misalkan Q sebuah titik di
dalam segitiga tersebut. Jika jumlah jarak dari Q ke ketiga sisi segitiga
adalah s. Nyatakan panjang p dalam s !
(soal olimpiade matematika SMA tk.Propinsi)
Pembahasan:
Perhatikan gambar berikut:
A
Z
B
Y
Q
X
C
104
Chapter 4. Geometri
Misalkan QX, QY dan QZ berturut-turut adalah jarak Q ke sisi BC, AC
dan AB. Perhatikan bahwa Luas 4ABC = Luas 4BQC + Luas 4AQC +
Luas 4AQB sehingga
1
1
1
1
.AB.AC. sin 600 =
.BC.QX + .AC.QY + .AB.QZ
2
2
2
2
1 1 1√
1 1
1 1
1 1
. p.
3 =
. p.QX + . p.QY + . p.QZ
2 3 2
2 3
2 3
2 3
√
Diperoleh, 13 p. 21 3 = QX + QY + QZ
√
Sehingga p = √63 s = 2s 3
4. Segitiga AOB adalah segitiga samakaki yang siku-siku diO dengan panjang AB sedemikian hingga pajang AP dan panjang AO sama. Tentukan
jari-jari lingkaran yang memenuhi semua syarat berikut:
(a) berpusat di ruas garis AB
(b) melalui titik P dan
(c) menyinggung ruas garis AO
(olimpiade matematika jawa timur)
Pembahasan:
Perhatikan gambar
A
K
K
L
P
O
B
105
Chapter 4. Geometri
√
Misalkan jari-jari r. Maka AL = LP = r 2. Karena AB = m, maka
√
√
√
AO = AP = m2 2 . Padahal AP = AL + LP . Oleh karena itu m2 2 = r 2 + r.
Dari sini diperoleh r =
√
m(2− 2)
2
5. Tunjukkkan bahwa sudut antara garis tinggi D dan diameter lingkaran
luar 4ABC yang dibuat dari A yaitu AL, besarnya sama dengan seliih
sudut-sudut pada titik-titik sudut B dan C dan terbagi dua sama besar
oleh garis bagi ∠A !
Pembahasan:
Perhatikan gambar
A
O
B
M
C
D X
L
Karena ∠AOC = 2B, kita dapatkan ∠CAO = 900 − B dan juga ∠DAB =
900 − B,sehingga ∠CO = ∠DAB. Karena garis bagi AX maka ∠XAB =
∠XAC. Sehingga kita dapatkan ∠XAD = ∠XAO atau dengan kata lain
AX merupakan garis bagi ∠ADL.∠DAL = 2(∠XAB − ∠DAX) = B − C
6. Dinotasikan O, I, R dan r berturut-turut sebagai pusat lingkaran luar dan
dalam, jari-jari lingkaran luar dan dalam. Tunjukkan bahwa berlaku OI 2 =
R2 − 2Rr!
Pembahasan:
Perhatikan gambar
Karena ∠AM C = ∠M AC = 12 A, diperoleh ∠M BI = 12 (A + B) = ∠IBA +
∠BAI yang berakibat BM = BI. Misalkan garis bagi dalam dan luar
106
Chapter 4. Geometri
N A
X
L
O
I
Y
B
C
M
∠A memotong lingkaran luar ABC di M da N , maka ∠M AN = 900 , sebab M N adalah diameter. Dibentuk garis XY yang merupakan diameter
yang memuat O dan I, sehingga XI.IY = AI.IM sedangkan AI.IM =
(R + OI)(R − OI) = R2 − IO2 . Perhatikan bahwa IL⊥AB, karena ∠LAI =
∠BN M dan ∠ALI = ∠M BN , maka 4ALI sebangun dengan 4N BM .
Akibatnya berlaku
AL
IL
=
MN
MB
dan selanjutnya karena BM = IM , didap-
atkan:
AI.IM = M N.IL = Rr
AI.IM = R2 − OI 2
2Rr = R2 − OI 2
OI 2 = R2 − Rr
Persamaan ini disebut persamaan Euler
7. Diberikan sebarang segitiga sama kaki ABC dengan AB = AC. Jika r
adalah jari-jari lingkaran luarnya dan p adalah jari-jari lingkaran dalamnya, tunjukkan bahwa jika d merupakan jarak kedua titik pusat kedua
p
lingkaran itu maka d = r(r − 2p)!
Pembahasan:
Perhatikan gambar
107
Chapter 4. Geometri
A
L
O
I
C
B
M
Menurut persamaan euler maka BM = IM = r − d, sedangkan BD = 12 a.
Luas 4ABI adalah:
1
1
AB.IL =
AI.BD
2
2
1
c.p = (r + d) a
2
2cp = a(r + d)
Sedangkan luas 4ABM adalah
1
1
AB.BM =
AM.BD
2
2µ ¶
1
c(r − d) = 2r a
2
c(r − d) = 2r
2cpr
c(r − d) =
r+d
2pr = (r − d)(r + d) = r2 − d2
d2 = r62 − 2prr(r − 2p)
p
d2 =
r(r − 2p)
T erbukti
108
Chapter 4. Geometri
8. Buktikan torema Ptolemeus berikut: ’pada setiapsegiempat talibusur selalu brlaku perkalian diagonal-diagonalnya sama denga jumlah perkalian
sisi-sisi yang berhadapan.’
Pembahasan:
Perhatikan gambar
A
D
B
E
C
Pada gambar dibentuk garis AE sedemikian hingga berlaku ∠BAE =
∠CAD.4BAE sebangun dengan 4CAD karena ∠BAE = ∠CAD dan
∠ABE = ∠ACD. Sehingga
AB
AC
=
BE
CD
atau AC.BE = AB.CD. Analog
dengan cara diatas 4ADE sebangun dengan 4ACB sehingga
AD
AC
=
DE
BC
atau AC.DE = AD.BC. Jika kedua persamaan ini dijumlahkan akan didapat
AC.BE + AC.DE = AB.CD + AD.BC
AC(BE + DE) = AB.CD + AD.BC
AC.BD = AB.CD + AD.BC
T erbukti
9. Jika ABCD adalah segiempat yang bukan segiempat tali busur, tunjukkan
bahwa AB.CD + AD.BC > AC.BD!
109
Chapter 4. Geometri
Pembahasan:
Perhatikan gambar
A
D
B
E
C
Karena ABCD bukan segiempat tali busur maka ∠ABD 6= ∠ACD sehingga dapat kita tentukan titik E di luar BD sedemikian hingga ∠ABE =
∠ACD dan ∠BAE = ∠CAD. Akibatnya 4BAE sebangun denga 4CAD.
AB
AC
=
BE
CD
atau AB.CD = AC.BE. Analog dengan cara diatas didapatkan
4ABC sebangun dengan 4AED.
BC
AC
=
ED
AD
atau BC.AD = AC.ED. Jika
kedua persamaan tersebut dijumlahkan maka didapat:
AB.CD + BC.AD = AC.BE + AC.ED
AB.CD + BC.AD = AC(BE + DE)
AB.CD + BC.AD > AC.BD
T erbukti
10. Jika panjang sisi-sisi BC, CA dan AB pada segitiga ABC adalah a, b dan
c berturut-turut dan 2s = a + b + c, buktikan bahwa luas daerah segitiga
p
ABC adalah L = s(s − a)(s − b)(s − c) !
(soal kompetensi matematika SMU Bandung)
110
Chapter 4. Geometri
Pembahasan:
Perhatikan pada 4ADC berlaku
t2 = b2 − x2 ......1
t2 = a2 − (c − x)2
t2 = a2 − c2 + 2cx − x2 ......2
Dari persamaan 1 dan 3 didapatkan
b2 − x2 = a2 − c2 + 2cx − x2
b2 + c2 − a2 = 2cx
b2 + c2 − a2
x =
......3
2c
µ
2
2
Jika 3 disubsitusikan ke 1, maka t = b −
2
t
=
=
=
=
2
c
¶2
µ
µ 2
¶2 ¶µ
µ 2
¶2 ¶
b + c2 − a2
b + c2 − a2
b−
b+
2c
2c
2
2
2
2
2
2
2bc − b − c + a 2bc + b + c − a
.
2c
2c
a2 − (b − c)2 (b + c)2 − a2
.
2c
2c
(a − b + c)(a + b − c)(b + c − a)(b + c − a)
4c2
(2s − 2b)(2s − 2c)(2s − 2a)(2s)
4c2
=
Sehingga t =
b2 +c2 −a2
2c
p
s(s − a)(s − b)(s − c)
Luas segitiga ABC adalah
1
1 2p
L = .c.t = .c.
s(s − a)(s − b)(s − c)
2
2 c
111
Chapter 4. Geometri
Jadi, L =
p
s(s − a)(s − b)(s − c) Terbukti
Bagian 2
1. Jika dalam 4ABC, AD adalah garis tinggi dan AE adalah diameter lingkaran
luar yang melalui A maka AB.AC = AD.AE
Pembahasan:
A
B
C
O D
E
Karena ∠ABC = ∠ABD dan ∠ABD = ∠ACE = 900 , sehingga 4ABD
sebangun dengan 4AEC maka
AB
AE
=
AD
AC
atau AB.AC = AD.AE. Terbukti
Pernyataan ini disebut Brahmagupta.
2. Perhatikan gambar
C
A
B
Tiga lingkaran berpusat di A, B dan C. Lingkaran berpusat di A mempunyai jari-jari 3. Lingkaran berpusat di B mempunyai jari-jari 5. Besar sudut
BAC adalah
π
3
Pembahasan:
radian. Besar sudut ABC adalah
112
Chapter 4. Geometri
Misalkan jari-jari lingkaran berpusat di C = r. Dengan mengunakan aturan cosinus didapatkan
(r + 5)2 = 82 + (r + 3)2 − 16(r + 3) cos
π
3
Maka diperoleh r = 2. Misalkan β adalah sudut ABC maka dengan menggunkan aturan cosinus didapatkan pula 52 = 82 + 72 − 2.8.7 cos β Dari sini
diperoleh cos β =
11
14
11
sehingga β = arccos 14
3. Perhatikan gambar. Jika sudut q = 14 p. Tunjukkkan bahwa kedua daerah
yang diarsir mempunyai luas yang sama! (soal kompetensi Jakarta IV Jakarta)
Pembahasan:
Misalkan jari-jari lingkaran besar adalah R
L1 = LjuringBAE − L4BAE
µ ¶2
µ ¶2 µ ¶2
1
1
1 1
900
xπ R −
R
R
=
0
360
2
2 2
2
1
1
=
πR2 − R2 ......(1)
16
8
L1 = LjuringACD − LjuringBCE − L4BAE
µ ¶2
µ ¶2 µ ¶2
1
π
1
1
900
1 1
2
4
=
xπR −
xπ
R
−
R
R
2π
3600
2
2 2
2
1 2
1
1
=
R − πR2 − R2
8
16
8
1
1
=
πR2 − R2 ......(2)
16
8
Dari (1) dan (2) didapatkan Terbukti.
√
4. Suatu persegi panjang berukuran 8x2 2 mempunyai titik pusat yang sama
dengan suatu lingkaran berjari-jari 2. Berapakah luas daerah irisan antara
persegi panjang da lingkaran tersebut?
113
Chapter 4. Geometri
Soal olimpiade matematika kabupaten
Pembahasan:
Dari gambar diperoleh bahwa cos =
√
2
.
2
Jadi θ = 450 .
Luas tembereng = luasjuring − luas4P AB
90
1
=
xπx2x2 − x2x2
360
2
= π−2
Luas daerah irisan = luas lingkaran − luas tembereng
= πx2x2 − 2(π − 2)
= 4π − 2π + 4
= 2π + 4
5. Buktikan bahwa jumlah luas bujur sangkar yang diluar sama dengan tiga
kali jumlah luas bujur sangkar yang di dalam!
(Soal matematika RIA Bogor)
Pembahasan:
Misalkan panjang sisi busur sangkar yang ada didalam adalah a, b, dan c,
maka jumlah luas bujur sangkar tersebut adalah a2 + b2 + c2 . Keemudian
dimisalkan panjang sisi bujur sangkar yang ada di luar dalah x, y dan z,
maka jumlah luas bujur sangkar tersebut adalah x2 + y 2 + z 2 . Akan ditunjukkan bahwa x2 + y 2 + z 2 = 3(a2 + b2 + c2 ). Perhatikan 4ABC. Menurut
114
Chapter 4. Geometri
aturan cosinus didapatkan
b2 + c2 − a2
2bc
a2 + c2 − b2
cos β =
2ac
2
a + b2 − c2
cos γ =
2ab
cos α =
Kemudian untuk setiap segitiga sama kaki diatas didapatkan
x2 = a2 + c2 − 2ac cos(1800 − β)
= a2 + c2 − 2ac cos β
µ 2
¶
a + c2 − b2
2
2
= a + c − 2ac
2ac
2
2
= 2a + 2c − b
Dengan cara yang sama didapatkan pula
y 2 = 2b2 + 2c2 − a2
z 2 = 2a2 + 2b2 − c2
Dari ketiga persamaan ini diperoleh
x2 + y 2 + z 2 = 2a2 + 2c2 − b2 + 2b2 + 2c2 − a2 + 2a2 + 2b2 − c
x2 + y 2 + z 2 = 3a2 + 3b2 + 3c2
Terbukti
115
Chapter 4. Geometri
6. Diberikan segitiga sama sisi ABC dan sebuah titik P sehingga jarak P ke
A dan ke C tidak lebih jauh dari jarak P ke B. Buktikan bahwa jika P terletak pada lingkaran luar segitiga ABC maka P B = P A.P C!Soal seleksi
olimpiade matematika
Pembahasan:
C
1200
O
P
A
B
1
1
x∠BOC = x1200 = 600
2
2
1
1
x∠AOB = x1200 = 600
∠AP B =
2
2
AB 2 = P A2 + P B 2 − 2P A.P B cos 600
∠BP C =
AB 2 = P A2 + P B 2 − P A.P B......(1)
BC 2 = P C 2 + P B 2 − 2P C.P B cos 600
BC 2 = P C 2 + P B 2 − 2P C.P B......(2)
Karena ABC segitiga samasisi maka AB = BC, sehingga didapatkan
P A2 + P B 2 − P A.P B = P C 2 + P B 2 − P C.P B
P A2 − P C 2 = P A.P B − P C.P B
(P A − P C)(P A + P C) = (P A − P C)P B
T erbukti bahwa P B = P A.P C
116
Chapter 4. Geometri
7. Segitiga ABC siku-siku di C. Garis berat CM tegak lurus garis berat BN .
Panjang sisi BC = s. Tentukan panjang BN !
Pembahasan:
Misalkan AC = b dan AB = c. Karena BN dan CP garis berat, maka
BN : P N = 1 : 2 dan CM : P M = 2 : 1.
Perhatikan 4BN C
C
N
B
M
A
BN 2 = s2 + CN 2
1
BN 2 = s2 + b2 .......(1)
4
Perhatikan 4BP M
µ
BP 2
¶2
2
BN
3
4
BN 2
9
4
BN 2
9
4
BN 2
9
5
BN 2
9
= BM 2 + P M 2
µ ¶2 µ
¶2
1
1
=
c +
CP
2
2
1 2 1
=
c + CP 2
4
4
1 2 1 2
=
c + (s − CBP 2 )
4
4
1 2 1 2 1
=
c + s − BN 2
4
4
9
1 2 1 2
=
c + s .......(2)
4
4
117
Chapter 4. Geometri
Jika (2) - (1) didapatkan
4
1 2
3
− BN 2 =
(c − b2 ) − s2
9
4
4
4
1 2 3 2
2
− BN =
s − s
9
4
4
4
2 2
2
− BN =
s
9
4
18 2
BN 2 =
s
16
√
Jadi panjang BN adalah BN = 34 s 2
8. Misalkan sisi BC dari segitiga siku-siku ABC adalah garis tengah sebuah
lingkaran yang memoton sisi miring AB di D. Garis singgung lingkaran
di D memotong sisi CA di F . Buktikan bahwa besar sudut CDF adalah
dua kali besar sudut A. Soal seleksi olimpiade Matematika
Pembahasan:
Perhatikan 4OBD pada gambar dibawah.
B
D
O
C
F
A
118
Chapter 4. Geometri
∠OBD = ∠OBD
∠OBD = ∠B
∠DOB = 1800 − 2∠B
∠COD = 2∠B
∠CF D = 1800 − 2∠B
= 2(900 − ∠B)
= 2∠A
Terbukti
9. Segitiga ABC siku-siku di C. Titik P dan Q terletak pada AB sedemikian
hingga AB terbagi menjadi tiga bagian yang sama. Buktikan bahwa CP 2 +
P Q2 + QC 2 = 23 AB 2
Soal kompetensi matematika V Jakarta
Pembahasan: Perhatikan 4CAP .
C
D
P
B
A
119
Chapter 4. Geometri
Menurut aturan cosinus didapatkan
CP 2 = CA2 + P A2 − 2CA.AP cos A
µ
¶2
1
1
CA
2
2
CP = CA +
AB − 2CA. AB
2
3
AB
1
1
CP 2 =
CA2 + AB 2 .......(1)
3
9
Perhatikan 4CAQ
QC 2 = CA2 + AQ2 − 2CA.AQ cos A
µ
¶2
2
CA
2
2
2
QC = CA +
AB − 2CA. AB
3
3
AB
1
4
QC 2 = − CA2 + AB 2 .......(1)
3
9
Jadi,
1
1
1
1
4
CA2 + AB 2 + AB 2 − CA2 + AB 2
3
9
9
3
9
2
=
AB 2
3
CP 2 + P Q2 + QC 2 =
Terbukti
10. Sebuah titik A terletak disebelah luar lingkaran yang berpusat di titik M .
Dari titik A ditarik garis yang memotong lingkaran dititik B da C ( titik
B terletak diantara C). Dibuat garis CM sehingga memotong lingkaran
di titik D, ternyata AD menyinggung lingkaran dan titik E terletak pada
garis KD. Jika panjang AE = 1cm, XB = 2 cm dan BC = 6 cm, Buktikan bahwa AM, DB dan CE berpotongan di sebuah titik!Soal kompe-
120
Chapter 4. Geometri
tensi matematika XII jakarta
Pembahasan:
B
2
C
6
R
M
A
1
E
R
D
Berdasarkan teorema Secant-Tangent didapatkan
AD2 = AB.AC = 2.(2 + 6)
AD2 = 16makaAD = 4
ED = 4 − 1 = 3
AB CM DE
.
.
BC M D EA
= 26 . R
. 3 = 1, Berarti AM, DB, dan CE berpotongan pada satu
R 1
titik
11. Panjang sisi AB, BC, CD, DA dari persegi panjang ABCD yang panjang
sisinya a dan b. Dipilih titik E, F, G dan H sedemikian hingga. Tentukan
luas daerah segiempat yang dibatasi oleh garis AG, BH, CE dan DF .
Pembahasan:
Karena tan α. tan β =
a
2
a
3
=
2
a
3
a
= 1, maka BH ⊥ EC. Berarti segiempat
yang terarsir berupa persegi. Misalkan panjang sisinya x maka berlaku
BC
x
a
x
=
⇔ 1 = 1 √
AE
CE
a
a 13
3
3
sehingga diperoleh x =
√a .
13
Jadi luas daerah terarsir x2 =
2
√a
13
121
Chapter 4. Geometri
A P
B
12. Dari gambar diketahui luas yang diarsir adalah A. Tunjukkan bahwa luas
persegi panjang juga A.
Pembahasan:
2p + 2q = A
1
x+p =
luas lingkaran
2
µ ¶2
1
1
x+p =
π b
2
2
1 2
2x + 2p =
πb
4
(5.1)
(5.2)
(5.3)
(5.4)
(5.5)
1
luas lingkaran
2
µ ¶2
1
1
y+q =
π a
2
2
1 2
2y + 2q =
πa
4
y+q =
(5.6)
(5.7)
(5.8)
(5.9)
122
Chapter 4. Geometri
Jika (1) dan (2)dijumlahkan didaptkan
1
1
2x + 2p + 2y + 2q = πb2 + πa2
4
4
1 2
2x + 2y + A = π(b + a2 )
4
1 2
2x + 2y = π(b + a2 ) − A
4
Luas lingkaran = luas persegi + 2x + 2y
1
π(b2
4
+ a2 ) = luas persegi + 14 π(b2 + a2 ) − A, terbukti bahwa luas persegi
adalah A.
13. Sebuah lingkaran memotong sisi-sisi pada bagian dalam yaitu DC di D
dan D0 , CA di E da E 0 , AB di F dan F 0 . Jika AD, BE, dan CF kongruen,
tunjukkan bahwa AD0 , BE 0 dan CF 0 juga kongruen.
Pembahasan: Menurut teorema Secant diperoleh:
AF.AF
0
BD.BD0
CE.CE 0
AF
AE 0
= AE.AE ⇒
=
AE
AF 0
BD
BF 0
= BF.BF 0 ⇒
=
BF
BD0
CE
CD0
= CD.CD0 ⇒
=
CD
BD0
0
123
Chapter 4. Geometri
karena AD, BE dan DF kongruen maka
AF BD CE
.
.
=1
F B DC EA
AF BD CE
.
.
=1
EA F B DC
AE 0 BF 0 CD0
.
.
=1
AF 0 BD0 CE 0
AF 0 BD0 CE 0
.
.
=1
AE 0 BF 0 CD0
AF 0 AF 0 CE 0
.
.
=1
F 0B F 0B E 0A
T erbukti
14. Misalkan ABC segitiga dan P suatu titik di dalam segitiga tersebut. Garis
AP, BP dan CP memotong garis BC, CA dan AB berturut-turut dititik
D, E, dan F . Garis DE, EF , dan F D berturut-turut memotong garis AB, BC
dan CA di titik titik K, M dan N . Tunjukkan bahwa K, M, dan N terletak
dalamsatu garis.
Pembahasan: Karena garis-garis AD, BE, dan CF berpotongan disatu
titik P , maka menurut teorema Ceva diperoleh
AF BD CE
.
.
=1
F B DC EA
Karena titik-titik D, E, dan K terletak pada satu garis, dengan Teorema
Manelaos diperoleh
AK BD CE
. .
KB DC EA
= 1. Ekuivalen dengan
gan cara yang sama akan diperoleh juga
BM
MC
=
EA F B
.
CE AF
AK
KB
dan
DC EA
. . DenBD CE
CN
DC
= FAFB . BD
.
NA
=
Dari persamaan (1),(2) dan (3) didapatkan
µ
¶2
AK BM CN
F B DC EA
.
.
=
.
=1
KB M C N A
AF BD CE
Sehingga menurut Teorema Menelaos terbukti bahwa K, M , dan N terletak pada satu garis.
Chapter 4. Geometri
124
15. Lingkaran dalam segitiga ABC menyingguan sisi sisi BC, CA, dan AB
berturut-turut di titik titik D, E, dan F . Misalkan M titik tengah EF dan
garis AD memotong lingkaran dalam segitiga ABC di titik K, buktikan
bahwa ∠KM E = ∠KM D = ∠DM E
Pembahasan:Misalkan I adalah titik pusat lingkaran dalam segitiga ABC.
Tampa mengurangi sifat keumuman, misalkan K dan F terletak pada sisi
yang sama terhadap garis AM . Maka berlaku ∠KM E = ∠KM A + 900
dan ∠DM E = ∠DM I + 900 . Akan dibuktikan bahwa ∠KM A = ∠DM I.
karena segitiga AM F dan AF I sebangun maka berlaku AM.AI = AF 2 .
Dengan teorema titik kuasa pada A didapatkan AK.AD = AF 2 . Dari sini
didapatkanAK.AD = AM.AI yang artinya KM ID adalah suatu segiempat tali busur. Karena ID = IK, maka sudut keliling yang meghadap
busur-busur tersebut adalah sama besar yaitu ∠DM I = ∠IDK. Dengan
sifat segiempat talibusur didapatkan ∠KM A = ∠IDK. Dengan demikian
didapatkan ∠KM E = ∠KM A + 900 = ∠DM I + 900 = ∠DM E.
¥¥¥¥¥¥
CHAPTER 6
Kombinatorika
Konsep-konsep penting yang perlu dipahami dalam menyelesaikan masalah
dalam kombinatorika diantaranya adalah konsep permutasi dan kombinasi, prinsip inklusi-ekslusi, koefisien binomial, prinsip sarang merpati (pigeon hole principle), paritas dan relasi rekurensi.
6.1
Permutasi dan Kombinasi
Definition 6.1.1 Permutasi. Banyaknya cara menyusun r anggota dari sebuah himpunan yang mempunyai n anggota dengan memperhatikan urutan adalah P (n, r) =
n!
Prn = (n−r)!
, dimana n! = n · (n − 1) . . . 3 · 2 · 1. Apabila sebuah objek dapat dipilih lebih dari satu kali maka disebut permutasi dengan pengulangan dengan rumus
P (n, r) = Prn = nr .
Definition 6.1.2 Kombinasi. Banyaknya cara menyusun r anggota dari sebuah himpunan yang mempunyai n anggota tanpa memperhatikan urutan adalah C(n, r) =
n!
Crn = r!(n−r)!
. Apabila sebuah objek dapat dipilih lebih dari satu kali maka disebut
kombinasi dengan pengulangan dimana C(n + r − 1, r) = Crn+r−1 =
(n+r−1)!
.
r!(n−1)!
Example. Tentukan banyaknya susunan dua angka dari lima angka 1,2,3,4,5.
Solution. Susunan selain diagonal utama adalah suatu permutasi, dan bila termasuk diagonal utama disebut permutasi dengan pengulangan. Sedangkan
susunan kombinasi adalah hanya di atas diagonal utama atau hanya di bawah
diagonal utama.
125
126
Chapter 9. Kombinatorika
1
z}|{
1 |{z}
11
6.2
2
3
4
5
13
14
15
24
25
35
2
21
12
z}|{
22
|{z}
3
31
32
23
z}|{
33
|{z}
4
41
42
43
34
z}|{
44
|{z}
5
51
52
53
54
45
z}|{
55
|{z}
Prinsip Inklusi-Ekslusi dan Peluang
Prinsip ini digunakan untuk menentukan kardinalitas dari gabungan himpunanhimpunan yang tidak harus saling lepas.
Theorem 6.2.1 Misal Ai adalah sebarang himpunan, 1 ≤ i ≤ n. Kardinalitas dari
gabungan himpunan-himpunan yang tidak harus saling lepas adalah:
n
n
X
X
¯[
¯\
¯ X
¯
¯ Ai ¯ =
|Ai ∩ Aj ∩ Ak | − . . . (−1)n+1 ¯ Ai ¯
|Ai | −
|Ai ∩ Aj | +
i=1
i=1
i<j
i<j<k
i=1
Definition 6.2.1 Jika S = {k1 , k2 , . . . , kn } adalah himpunan dari semua kejadian dalam
suatu ruang sampel, sedangkan K adalah kejadian yang terjadi pada himpunan K ⊆ S
maka peluang terjadinya K adalah p(K) = n(K)
. Dalam hal ini pi ≥ 0 dengan
n(S)
i = 1, 2, . . . , n dan p1 + p2 + · · · + pn = 1.
Definition 6.2.2 Misalkan S adalah ruang sampel berhingga dengan kejadian A dan
B, maka probabilitas bersyarat dari kejadian A dengan syarat B, ditulis p(A/B) adalah:
p(A/B) = p(A/B)
.
p(B)
Example. Pada sebuah klub olahraga diketahui bahwa 10 orang menyukai tennis, 15 orang menyukai squash, 12 orang menyukai badminton, 5 orang menyukai
127
Chapter 9. Kombinatorika
tennis dan squash, 4 orang menyukai tennis dan badminton, 3 orang menyukai
squash dan badminton dan 2 orang menyukai ketiga olahraga. Berapa banyak
anggota klub yang menyukai sedikitnya satu dari ketiga cabang olahraga ini?
Solution. Misalkan T, S dan B masing-masing adalah himpunan anggota klub
yang menyukai tennis, squash dan badminton. Maka menurut teorema InklusiEkslusi
|T ∪ S ∪ B| = |T | + |S| + |B||T ∩ S| − |T ∩ B| − |S ∩ B| + |T ∩ S ∩ B|
= 10 + 15 + 12 + 5 − 43 + 2
= 12
Jadi, banyaknya anggota klub yang menyukai sedikitnya satu dari ketiga cabang olahraga ini adalah 27 orang.
Example. Sepasang dadu dilempar satu kali. Misal A adalah kejadian muncul
mata 2 pada paling sedikit satu dadu, dan B kejadian jumlah mata yang muncul
pada kedua dadu sama dengan 6. Tentukan peluang kejadian A dengan syarat
B.
Solution. S = {11, 12, . . . , 66} sehingga n(S) = 36. A = {21, 22, 23, . . . , 62}, B =
{15, 24, 33, . . . , 51} dan n(A) = 11, n(B) = 5. Dengan demikian A ∩ B = {24, 42}
dan n(A ∩ B) = 2 sehingga p(A/B) =
6.3
p(A/B)
p(B)
=
2/36
5/36
= 52 .
Koefisien Binomial
Prinsip ini digunakan untuk menentukan kardinalitas dari gabungan himpunanhimpunan yang tidak harus saling lepas.
128
Chapter 9. Kombinatorika
Theorem 6.3.1 Jika x dan y adalah variabel dan n adalah bilangan asli maka berlaku:
n µ ¶
X
n n−r r
n
(x + y) =
x y
r
r=0
Bila (x + y)n diekspansikan maka berlaku beberapa sifat berikut:
• Terdapat (n + 1) suku, jumlah pangkat dari x dan y pada tiap suku adalah
n.
• Pangkat dari x turun satu demi satu dari n ke 0, sedangkan pangkat dari y
naik satu demi satu dari 0 ke n.
• Koefisien dari suku-suku yang berjarak sama dari xn dan y n adalah sama.
Beberapa contoh dapat dilihat dalam berikut ini.
(x + y)0 = 1
(x + y)1 = x + y
(x + y)2 = x2 + 2xy + y 2
(x + y)3 = x3 + 3x2 y + 3xy 2 + y 3
(x + y)4 = x4 + 4x3 y + 6x2 y 2 + 4xy 3 + y 4
(x + y)5 = x5 + 5x4 y + 10x3 y 2 + 10x2 y 3 + 5xy 4 + y 5
(x + y)6 = x6 + 6x5 y + 15x4 y 2 + 20x3 y 3 + 15x2 y 4 + 6xy 5 + y 6
Kalau koefisien-koefisiennya disusun dalam bentuk segitiga, maka susunan segitiga tersebut dinamakan ’Segitiga Pascal’. Segitiga ini mempunyai sifat: (i) Bilangan pertama dan bilangan terakhir dalam tiap baris adalah 1, (ii) Tiap bilangan selain 1, dapat dicari dengan menjumlahkan dua bilangan tepat di atasnya.
1
129
Chapter 9. Kombinatorika
1+1
1+2+1
1+3+3+1
1+4+6+4+1
1 + 5 + 10 + 10 + 5 + 1
1 + 6 + 15 + 20 + 15 + 6 + 1
Example. Hitunglah berapa nilai dari
P1003 ¡ 2007 ¢
k=0 2k+1 .
Solution. Perhatikan bahwa binomial untuk x = 1 dan y = 1 adalah
(1 + 1)
2007
¶
2007 µ
X
2007
k=0
k
2007
(1 + (−1))
¶
2007 µ
X
2007
k=0
k
¶
2007 µ
X
2007
=
(1)2007−k (1)k = 22007
k
k=0
= 22007
(6.1)
¶
2007 µ
X
2007
(1)2007−k (−1)k = 02007 = 0
=
k
k=0
(−1)k = 0
X µ2007¶
X µ2007¶
k
(−1) +
(−1)k
k
k
k genap
k ganjil
µ
¶
X 2007
X µ2007¶
−
k
k
k genap
k ganjil
X µ2007¶
k
k genap
¶
2007 µ
X
2007
2k
k=0
= 0
= 0
X µ2007¶
=
k
k ganjil
¶
2007 µ
X
2007
=
2k + 1
k=0
(6.2)
130
Chapter 9. Kombinatorika
Karena menurut (6.2)
¶
2007 µ
X
2007
k=0
k
=2
¶
2007 µ
X
2007
k=0
2k
¶
2007 µ
X
2007
=2
2k + 1
k=0
Maka sesuai (6.1) diperoleh
¶
¶
2007 µ
2007 µ
X
2007
1 X 2007
1
=
= · 22007 = 22006
2k + 1
k
2 k=0
2
k=0
Pn
¡n¢
r = n2n−1 .
P ¡ ¢
Solution. Menurut koefisien binomial didapat (x+1)n = nr=0 nr xr . Turunkan
Pn ¡n¢ r−1
sekali terhadap x didapat n(x + 1)n−1 =
. Substitusikan untuk
r=0 r rx
Pn ¡n¢
Pn ¡n¢
n−1
r−1
x = 1 diperoleh n(1 + 1)
= r=0 r r(1)
atau r=0 r r = n2n−1 .
¡ ¢
P
Example. Buktikan bahwa nr=1 r2 nr = n(n + 1)2n−2 .
P ¡ ¢
Solution. Menurut koefisien binomial didapat (x+1)n = nr=0 nr xr . Turunkan
P ¡ ¢
dua kali terhadap x didapat n(n−1)(x+1)n−2 = nr=2 nr r(r−1)xr−2 . Kemudian
Example. Jika n bilangan asli, buktikan bahwa
r=1
r
substitusikan x = 1 didapat:
n µ ¶
X
n
n(n − 1)(1 + 1)
=
r(r − 1)
r
r=2
n µ ¶
n µ ¶
X
n 2 X n
n−2
n(n − 1)2
=
r −
r
r
r
r=2
r=2
n µ ¶
n µ ¶
X
X
n
n 2
n−2
n(n − 1)2
+
r=
r
r
r
r=2
r=2
n−2
131
Chapter 9. Kombinatorika
Tambahkan n di kedua ruas
n−2
n + n(n − 1)2
+
n µ ¶
X
n
r2
r
r=2
µ
¶
µ
¶
n
n
X
X
n
n 2
n−2
n(n − 1)2
+
r=
r
r
r
r=1
r=1
n µ ¶
X
n 2
n−2
n−1
n(n − 1)2
+ n2
=
r
r
r=1
r=2
Sehingga
6.4
Pn
r=1
¡ n¢
r
r
r =n+
n µ ¶
X
n
r2 = n(n + 1)2n−2 .
Prinsip Sarang Merpati
Prinsip sarang merpati (pigeon hole principle) juga dikenal dengan nama shoe
box argument atau dirichlet drawer principle.
Theorem 6.4.1 Jika terdapat lebih dari n barang yang didistribusikan ke dalam n buah
kotak, maka sedikitnya satu kotak akan menerima lebih dari satu barang.
Contoh:
Selama bulan juni (30 april) Antony melakukan pertandingan catur sedikitnya
satu kali sehari. Banyak pertandingan selama bulan tersebut tidak lebih dari 45
kali. Tunjukkan bahwa terdapat periode dimana Antony melakukan tepat 14
pertandingan!
Jawab:
Misalkan ai menyatakan banyaknya pertandingan yang dilakukan Antony selama i hari. Karena dalam sebulan banyaknya pertandingan tidak lebih dari 45
kali maka
0 < a1 < a2 < ... < a30 ≤ 45
Chapter 9. Kombinatorika
132
Jumlahkan dengan 14 didapatkan
14 < a1 + 14 < a2 + 14 < ... < a30 + 14 ≤ 59
Dari ketaksamaan (1) dan (2) didapatkan 60 bilangan. Karena maksimal hanya
ada 59 bilangan maka setidaknya terdapat i dan j sedemikian hingga ai = aj +
14. Ekivalen dengan ai − aj = 14. Terbukti
6.5
Paritas
Prinsip ini digunakan untuk mengeliminasi kemungkinan-kemungkinan tertentu dengan cara memperhatikan dua masalah saja, misalnya ganjil genap atau
hitam putih. Misal kedua bilangan adalah genap atau ganjil maka bilanganbilangan itu mempunyai paritas sama. Jika satu ganjil dan satunya genap maka
bilangan-bilangan itu mempunyai paritas yang berbeda.
Contoh:
Misalkan n > 1 adalah bilangan ganjil dan misalkan pula A adalah matriks
berukuran nxn yang simetris. Jika setiap baris dan kolom S merupakan suatu
permutasi dari 1, 2, ...n Buktikan bahwa setiap i, i = 1, 2, ...n akan muncul pada
diagonal utama!
Jawab:
Karena setiap baris dan kolom adalah permutasi dari 1, 2, 3, ...n akan muncul
sebanyak n kali dalam matriks A. Misalkan submatriks dari A yang terletak
dibawah dan diatas diagonal utama sebagai A1 dan A2 . Andaikan terdapat bilangan i0 diantara 1, 2, 3, ...n yang tidak muncul di A1 dan A2 haruslah sama.
Akibatnya banyaknya i0 yang muncul di A adalah genap, suatu kontradiksi.
Chapter 9. Kombinatorika
6.6
133
Relasi Rekurensi
Definition 6.6.1 Diberikan suatu fungsi numerik a0 , a1 , . . . , ar , . . . untuk sebarang r.
Suatu persamaan yang mengkaitkan ar dengan satu atau lebih ai , untuk i < r, disebut
relasi rekurensi atau persamaan beda (difference equations).
Contoh: Misalkan kita melempar sebuah koin n kali. Berapa banyaknya hasil
percobaan yang didalamnya sisi gambar tidak pernah muncul dua kali berturutturut? Jawab: Misalkan an menyatakan banyaknya hasil percobaan ke-n. Perhatikan bahwa untuk mendapatkan hasil percobaan sehingga sisi gambar tidak
boleh muncul dua kali berturut-turut maka
1. n − 1 barisan yang didalamnya sisi gambar tidak pernah muncuk dua kali
bertrut-turut pada barisan ke-n dipasangkan dengan sisi angka.
2. n − 2 barisan yang didalamnya sisi gambar tidak pernah muncul dua kali
berturut-turut pada barisab ke-(n − 1) dipasankan dengan sisi angka kemudian barisan ke-n dipasangkan dengan sisi gambar.
6.7
Soal-soal dan Pembahasan
Bagian I
1. Berapa cara untuk bergerak diruang xyz dari titk (0, 0, 0) ke (4, 3, 5) sehingga langkah yang diambil adalah arah x positif, y positif dan z positif?
Pembahasan: Karena langkah yang diambil adalah arah x positif,y positif dan z positif, maka langkah dari (0, 0, 0) ke (4, 3, 5) adalah langkah
yang terpendek. Sehingga banyaknya cara untuk bergerak di ruang tersebut yaitu harus terdiri dari 12 langkah dimana 4 langkah ke kanan(x), 3
134
Chapter 9. Kombinatorika
langkah ke belakang y dan 5 langkah ke atas (z). Jadi banyaknya cara
¡ ¢¡8¢¡5¢
adalah 12
= 27720
4
3 5
2. Hitunglah berapa koefesien dari x102 y 98 dari ekspansi (2x − 5y)200 !
Pembahasan:
Menurut teorema binomial untuk menghitung koefesien x102 y 98 dari ekspansi
(2x − 5y)200 yaitu dengan mengambil r = 102, sehingga didapatkan suku
¡ ¢
ke-102, sehingga didapatkan suku ke-102 sebagai berikut 200
(2x)102 (−5y)98
102
¢
¡
(2x)102 (−5y)98
Jadi koefesien x102 y 98 adalah 200
102
3. Berapa banyak solusi bilangan asli untuk persamaan x1 +x2 +x3 +...+xk =
n?
Pembahasan:
Dibentuk bilangan yi = xi − 1. Karena xi merupakan bilangna asli, maka
jelas bahwa yi merupakan bilangan bulat. Sehingga banyak solusi untuk
x1 +x2 +x3 +...+xk = n ekivalen dengan banyak solusi persamaan berikut:
x1 + x2 + x3 + ... + xk = n
(x1 − 1) + (x2 − 1) + (x3 − 1) + ... + (xk − 1) = n − k
y1 + y2 + y3 + ... + yk = n − k
Sehingga banyaknya solusi bilangan bulat adalah
¡n−k+k−1¢
k−1
=
¡n−1¢
k−1
4. Buktikan bahwa diantara n + 1 bilangan bulat yang dipilih terdapat dua
bilangan yan selisihnya hais dibagi n!
Pembahasan:
Misalkan bilangan yang dipilih tersebut adalah a1 , a2 , ..., an+1 . Diambil sebarang bilangan dari a1 , a2 , ..., an+1 misalkan a1 . Dibentuk
a1 − a2 ,
a1 − a3 ,
...,
a1 − an+1
Chapter 9. Kombinatorika
135
Jika salah satu dariselisih diatas habis dibagi n, maka pernyataan terbukti.
jika n selisih diatas tidak habis dibagi n, maka kemudian dibentuk
a1 − a2 = t1 n + r1 ,
a1 − a3 = t2 n + r2 ,
...,
a1 − an+1 = tn n + rn ,
dengan 0 < ri < n. Karena terdapat n hasil bagi sedangkan banyaknya
bilangan 0 < ri < n hanya ada n − 1 bilangan, maka pasti terdapat dua bilangan yang mempunyai sisa hasil bagi yang sama. Akibatnya didapatkan
dua bilangan yang selisihnya habis dibagi n. Terbukti.
5. Diberikan himpunan n + 1 bilangan bulat yang kurang dari 2n. Buktikan
bahwa setidaknya terdapat satu anggota himpunan yang membagi habis
anggota himpunan yang lain!
Pembahasan:
Misalkan anggota himpunan tersbut adalah x1 , x2 , x3 , ..., xn+1 . Untuk setiap i, xi = 2nii yi , dengan ni adalah bilangan bulat non-negatif dan yi adalah
bilangan ganjil. T = {yi : i = 1, 2, ..., n + 1}, maka T adalah koleksi n + 1
bilangan ganjil yang kurang dari 2n. Karena hanya terdapat n bilangan
ganjilyang kurang dari 2n, maka setidaknya terdapat dua bilangan yang
sama misalkan yi = yj , i < j. Akibatnya xi = 2ni yi dan xj = 2nj yj . Jika
ni ≤ nj maka xj membagi xi dan jika ni ≥ nj maka xi membagi xj . Terbukti.
6. Misalkan x1 , x2 , x3 , ..., xn adalah sebarang permutasi dari 1, 2, 3, ...n. Jika
n adalah bilangan ganjil buktikan bahwa perkalian (x1 − 1)(x2 − 2)(x3 −
3)...(xn − n) adalah genap!
Pembahasan:
Andaikan hasil perkalian tersebut ganjil, maka tiap-tiap faktor tidak bernilai genap. Jadi (xk − k) bernilai ganjil untuk setiap k. Diketahui n bilangan
Chapter 9. Kombinatorika
136
ganjil maka jumlah semua bilangan ganjil tersebut juga ganjil. Sedangkan
(x1 − 1)(x2 − 2)(x3 − 3)...(xn − n) = 0 bilangan genap. Kontradiksi, Jadi,
terbukti bahwa hasil perkalian tersebut bernilai genap.
7. Buktikan bahwa ada 2n − 2 bilangan yang terdiri dari atas n angka 1 dan 2
dengan masing-masing angka muncul sedikitnya sekali!
Pembahasan:
Karena banyaknya bilangan dengan angka 1 tidak pernah muncul adalah
1. Jadi banyaknya bilangan yang terdiri dari angka 1 dan 2 dengan masingmasin angka muncul sedikitnya sekali adalah 2n − 1 − 1 = 2n − 2 Terbukti.
8. Misakan T adalah sebuah segitiga sama sisi dengan panjang 1. Tunjukkan
bahwa jika terdapat 5 titik pada T maka dua diantaranya akan berjarak
tidak lebih dari 12 !
Pembahasan:
Bentuk tiga garis pada segitiga sama sisi tersebut sehingga tiga garis tersebut akan menghasilkan empat buah segitiga kecil sama sisi. Jelas bahwa
maksimal panjang segitiga kecil sama sisi tersebut adalah 12 . karena terdapat lima titik dan empat segitiga kecil, maka pasti terdapat dua titik yang
terletak pada segitiga kecil yang sama. Terbukti.
9. Tentukan banyaknya daerah yang dibentuk oleh n garis pada bidang datar.Pembahasan:
Diasumsikan tidak ada dua garis yang sejajar dan tidak ada tiga garis yang
berpotongan pada titik yang sama. Misalkan an menyatakan banyaknya
daerah yang dicari. Karena garis ke-n dipotong oleh n − 1 garis sebelumnya menjadi n ruas garis dan setiap ruas garis memotong suatu daerah
menjadi dua daerah baru maka didapatkan an = an−1 + n dengan a0 =
1, a2 = 2, a3 = 4, a4 = 7. Persamaan yang memenuhi syarat ini adalah
an = 1 + 12 n(n + 1)
10. Misalkan dimiliki perangko dengan nilai 5 sen dan 3 sen. Tunjukkan bahwa
Chapter 9. Kombinatorika
137
dapat dibentuk perangko senilai tepat 8 sen atau lebih!Pembahasan:
(a) Perangko senilai 8 sen jelas dapat dibentuk
(b) Perangko senilai 9 sen jelas dibentuk dengan 3 perangko nil ai 3 sen
dan perangko senilai 10 senjelas dibentuk dengan 2 perangko senilai
n > 10 dilakukan dengan cara berikut:
i. jika perangk n sen terdiri dari minimal 3 perangko bernilai 3 sen,
maka jika perangko ini diganti 2 perangko masing-masing bernilai 5 sen maka akan diperoleh perangko bernilai n + 1 sen.
ii. jika perangko k sen terdiri dari minimal 1 perangko bernilai 5
sen, maka jika perangko 5 sen diganti dengan 2 perangko masingmasing bernilai 3 sen akan diperoleh perangko bernilai n + 1 sen.
Bagian II
1. Berapa banyaknya bilangan bulat positif yang merupakan faktor dari 30030?
Pembahasan:
Karena 30030 = 2x3x5x7x11x13, dan hasil kali r bilangan 2, 3, 5, 7, 11, 13, 1 ≤
r ≤ 6 juga merupakan faktor dari 30030, maka diperoleh banyaknya faktor
¡ ¢
P
adalah 6n=0 n6 = 26
2. Misalkan S sebuah himpunan dengan n unsur. Buktikan bahwa banyaknya
himpunan bagian bagi S ada 2n !
Pembahasan:
Banyaknya unsur di dalam suatu himpunan bagian adalah 0 atau 1 atau
2,... atau n. Selain itu banyaknya himpunan bagian yang memiliki r unsur
¡ ¢
adalah nr untuk r = 0, 1, 2, ...n. Dengan demikian, banyaknya himpunan
¡ ¢
P
bagian ialah nn=0 nr = 2n .Terbukti.
3. Sebuah kata biner yang panjangnya n adalah suatu barisan angka yang
terdiri atas angka 0 atau 1. Berapa banyak kata biner dengan panjang 10
138
Chapter 9. Kombinatorika
yang diawali dengan tiga angka 0 atau diakhiri dengan dua angka 1?
Pembahasan:
Banyak kata biner dengan panjang 10 yang diawali dengan tiga angka 0
atau diakhiri dengan dua angka 1 = (banyak kata biner yang diawali dengan tiga angka 0) + (banyak kata biner yang diakhiri dengan dua angka
1) - (banyak kata biner yang diawalai dengan tiga angka 0 dan diakhiri
dengan dua angka 1). Kemudian, untuk mendapatkan kata biner yang
diawali dengan tiga angka 0 adalah 1.1.1.2.2.2.2.2.2.2 = 27 , diakhiri dengan dua angka 1 adalah 2.2.2.2.2.2.2.2.1.1 = 28 dan diawali tiga angka 0
dan diakhiri dua angka 1 adalah 1.1.1.2.2.2.2.2.1.1 = 25 . Dengan demikian
didapatkan 27 + 28 − 25 .
4. Hitunglah berapa nilai dari
Pk=0 ¡ 2007 ¢
1003 2k−1 !
Pembahasan:
Perhatikan bahwa binomial untuk x = 1 dan y = 1 adalah
¶
k=0 µ
X
2007
2007
k
= (1 + 1)2007 = 22007
.....(1)
Sedangkan binomial untuk x = 1 dan y = −1 adalah
¶
k=0 µ
X
2007
2007
Sehingga
Akibatnya
k
(1)2007−k (−1)k = (1 − 1)2007 = 02007 = 0
¶
k=0 µ
X
2007
(−1)k = 0
k
2007
X µ2007¶
X µ2007¶
k
(−1) +
(−1)k = 0
k
k
genap
ganjil
139
Chapter 9. Kombinatorika
Ekivale dengan
X µ2007¶ X µ2007¶
+
=0
k
k
genap
ganjil
X µ2007¶
X µ2007¶
=
k
k
genap
ganjil
.....(2)
Sehingga dari (1) dan (2) didapatkan
¶ X
¶
1003 µ
1003 µ
X
2007
2007
=
= 22007
2k + 1
2k + 1
k=0
k=0
¶
1003 µ
X
2007
2
= 22007
2k
+
1
k=0
Diperoleh
¶
1003 µ
X
2007
= 22006
2k
+
1
k=0
5. Buktikan bahwa
Pn
2
r=1 r
¡ n¢
r
= n(n + 1)2n−2 !
Pembahasan:
Menurut koefisien binomial didapatkan (x + 1)n =
Pn
r=0
¡n¢
r
xr Jika ruas
kanan dan kiri diturunkan dua kali terhadap x maka didapatkan
n(x + 1)
n(n − 1)(x + 1)
n−1
n−2
=
=
n µ ¶
X
n
r=1
n µ
X
r=2
r
rxr−1
¶
n
r(r − 1)xr−2
r
140
Chapter 9. Kombinatorika
Kemudian subsitusikan x = 1 sehingga didapakan
n µ ¶
X
n
n−2
n(n − 1)2
=
r(r − 1)
r
r=2
n µ ¶
n µ ¶
X
n 2 X n
n−2
n(n − 1)2
=
r −
r
r
r
r=2
r=2
n µ ¶
n µ ¶
X
X
n
n 2
n−2
n(n − 1)2
−
r =
r
r
r
r=2
r=2
Tambahkan n diruas kanan dan kiri sehingga diperoleh
n µ ¶
n µ ¶
X
X
n
n 2
n−2
n + n(n − 1)2
+
r = n+
r
r
r
r=2
r=2
Dari sini diperoleh
n µ ¶
X
n
n µ ¶
X
n 2
n(n − 1)2
+
r = n+
r
r
r
r=1
r=2
n µ ¶
X
n 2
n−2
n−1
n(n − 1)2
+ n2
=
r
r
r=1
n µ ¶
X
n 2
n−2
n(n − 1)2
=
r
r
r=1
n−2
6. Misalkan terdapat lima titik P1 , P2 , P3 , P4 , danP5 pada bidang dimana masingmasing mempunyai koordinat bilangan bulat. Buktikan bahwa terdapat
sedikitnya sepasang titik (Pi , Pj ) dengan i 6= j sedemikian hingga segmen
garis (Pi , Pj ) memuat sebuah titik Q yang mempunyai koordinat bilangan
bulat dimana Q 6= P atau Q 6= Pj !
Pembahasan:
Berdasarkan ganjil genapnya, terdapat 4 jenis koordinat untuk kelima titik
Chapter 9. Kombinatorika
141
ini yaitu (genap, genap), (ganjil,ganjil), (genap, ganjil) dan (ganjil, genap).
Karena terdapat lima buah titik maka menurut prinsip sangkar merpati,
setidaknya terdapat dua buah titik yang mempunyai jenis yang sama. misalkan kedua titik ini adalah Pi = (xi , yj )danPj = (xj , yj ) dengan i 6= j.
Karena xi da xj paritas yang sama maka
xi + xj¶adalah genap, demikian
µ
xi +xj yi +yj
juga dengan yi + yj . Akibatnya Q =
, 2
yang merupakan koor2
dinat titik tengah segmen garis mempunyai koordinat bilangan bulat.
7. Misalkan A adalah sebuah himpunan yang beranggotaan 10 bilangan bulat positif yang kurang dari 107. Buktikan bahwa terdapat dua suphimpunan saling lepas dari A yang hasil tamabah dari anggota-anggotanya
adalah sama!
Pembahasan:
Himpunan A yang hasil tambah dari anggotanya terbesar adalah A = dengan hasil tambah sama dengan 1050. Buatlah 1060 kotak dengan label
0 sampai dengan 1050. Sekarang perhatikan sebarang himpunan A dan
semua subhimpunan, tulis hasil tambah anggota-anggota pada selembar
kertas dan masukkan kertas tersebut pada kotak yang sesuai. Terdapat
1024 kertas dan 1060 kotak. Menurut prinsip sangkar merpati maka terdapat sebuah kotak yang berisi sedikitnya dua kertas. Hal ini berarti jumlah
anggota dari dua subhimpunan ini adalah sama. Perhatikan bahwa kedua sub himpunan ini bisa tidak saling lepas. Tetapi jika demikian kita
dapat menghapus anggota sekutu dari dua sub himpunan ini untuk mendapatkan dua sub himpunan lain yang saling lepas dan hasil tambahan
anggota anggotanya sama.
8. Misalkan n > 1 adalah bilangan bulat ganjil, dan misalkan A adalah matriks berukuran nxn yang simetris. Jika setiap baris dan kolom A merupakan suatu permutasi dari 1, 2, ...n, buktikan bahwa setiap i, i = 1, 2, ...n
142
Chapter 9. Kombinatorika
akan muncul pada diagonal utama A!
Pembahasan:
karena setiap baris dan kolom adalah permutasi dari 1, 2, ...n maka setiap
bilangan i, i = 1, 2, ...n akan muncul sebanyak n kali dalam matriks A. Sebut submatriks dari A yang terletak dibawah dan diatas diagonal utama
sebagai A1 dan A2 . Andaikan terdapat bilangan i0 diantara 1, 2, ...n yang
tidak muncul pada diagonal utama A. Karena A simetris maka banyaknya
i0 yang munculnya di A adalah genap. Suatu kontradiksi.
9. Misalkan P 1, P 2, P 3, P 4 dan P 5 terletak didalam sebuah persegi yang panjang sisinya adalah 1. Notasikan dij adalah jarak antara titik Pi dan Pj .
Tunjukkan bahwa terdapat dua titik yang jaraknya kurang dari
√
2
!
2
Pembahasan:
Bagi persegi tersebut menjadi empat buah persegi kecil yang sama sisi.
Akibatnya menurut pigeon hole setidaknya terdapat dua titik yang terletak pada persegi kecil yang sama. Karena panjang sisi persegi adalah 1,
maka panjang sisi persegi kecil adalah 12 . Menurut teorema phytagoras,
jarak maksimal dari titik sudut ke sudut yang lain yaitu
√
2
.
2
Terbukti.
10. Berapakah banyaknya bilangan bulat positif diantara 1 dan 1000 yang tidak
habis dibagi 2 dan tidak habis dibagi 5?
Pembahasan:
Misalkan S = {1, 2, 3, ..., 1000}, A = {x²S, 2|x} dan {B = {x²S, 5|x}. Akan
Chapter 9. Kombinatorika
143
dicari |A0 ∩ B 0 |. Menurut hukum De-Morgan, maka didapatkan
|A0 ∩ B 0 | = |(A ∪ B)0 |
= |S| − |(A ∪ B)|
= |S| − (|A| + |B| − |A ∩ B|)
µ·
¸¶ µ·
¸¶ µ·
¸¶
1000
1000
1000
= 1000 −
+
−
2
5
10
= 400
Jadi banyaknya bilangan bulat positif ada 400 bilangan.
11. Berapakah banyaknya penyelesaian bilangan bulat untuk persamaan x1 +
x2 + x3 = 24 dengan 1 ≤ x1 ≤ 5, 12 ≤ x2 ≤ 18, −1 ≤ x3 ≤ 5?
Pembahasan:
Notasikan y1 = x1 − 1, y2 = x2 − 12, y3 = x3 + 1, sehingga persamaan di atas
menjadi y1 + y2 + y3 = 12 dengan penyelesaikan bilangan bulat nonnegatif
dan y1 ≤ 4, y2 ≤ 6, y3 ≤ 13. Misalkan A, B dan C berturut-turut adalah
himpunan semua penyeesaian sedemikian hingga y1 > 4, y2 > 6, y3 >
13. Dengan demikian, banyaknya solusi nonnegatif untuk y1 + y2 + y3 =
¡
¢
¡3−1+7¢
¡3−1+5¢
12 adalah 12−1+3
=
19,
|A|
=
,
|B|
=
, |C| = 0 dan juga
7
5
T12
T
T
T T
diperoleh |A B| = 1, |A C| = |B C| = |A B C| = 0. Jadi, dari sini
menurut P IE didapatkan 91 − (36 + 21 + 0) − 0 = 35.
12. Buktikan bahwa banyaknya pemetaan dari M = {1, 2, ...m} ke N = {1, 2, ..., n}
adalah
µ ¶
µ ¶
n
n
m
(n − 2)m − ... + (−1)n−1 1m
n −
(n − 1) +
2
1
m
Pembahasan:
Banyaknya pemetaan dari M ke N adalah nm . Jika Ai adalah himpunan
144
Chapter 9. Kombinatorika
pemetaan dari M ke N sehingga i²N tidak mempunyai prapeta, maka
banyaknya pemetaan dari M ke N yang tidak pada adalah |A1 ∪ A2 ∪ ... ∪
An |.Perhatikan bahwa :
¡ ¢
|Ai | = n1 (n − 1)m
P
(b) Untuk setiap i1 , i2 ²N, |Ai1 ∪ Ai2 | = (n − 2)m . Akibatnya in1 <i2 |Ai1 ∪
(a) Untuk setiap i²N, |Ai | = (n − 1)m . Akibatnya
Pi=1
n
Ai2 | = (n − 2)m
Demikian seterusnya sehingga untuk setiap untuk i1 , i2 ...in−1 ²N , akibatnya
Ai1 ∩ Ai2 ∩ .... ∩ Ain | = (n(n − 1))m = 1m
Jadi
µ ¶
µ ¶
µ
¶
n
n
n
m
m
n−1
|A1 ∪ A2 ∪ ... ∪ An | =
(n − 1) −
(n − 1) − ... − (−1)
1m
1
2
n−1
Akibanya banyaknya pemetaan dari M ke N adalah
¶
µ
µ ¶
µ ¶
n
n
n
m
n−1
m
m
1m
(n−1) −...−(−1)
(n−1) −
n −|A1 ∪A2 ∪...∪An | =
n−1
2
1
13. Sebuah pesta dihadiri oleh n orang dan setiap hadirin meninggalkan topinya dibagian penyimpanan. Dalam berapa banyak cara topi-topi itu dikembalikan sehingga tidak seorang pun menerima topinya kembali?
Pembahasan:
Misalkan Ai melambangkan hmpunan kemungkinan hadirin ke-i mendapatkan kembali topinya, untuk setiap i. Banyaknya cara pengembalian
sehingga hadirin ke-i mendapatkan kembali topinya ada (n − 1)!. Jadi ,
|Ai = (n − 1)!. Selanjutnya untuk setiap pilihan k bilangan bulat berbeda,
misalkan Ai1 i2 i3 ...ik = Ai1 ∪ Ai2 ∪ Ai3 ... ∪ Aik adalah semua himpunan kemungkinan pengembalian sehingga hadirin ke-ik mendapatka kembali topinya. Dari sini dieroleh |Ai1 i2 i3 ...ik | = (n − k)!. pencacahan ini tidak bergan-
Chapter 9. Kombinatorika
145
tung pada pemilihan i1 , i2 , i3 ...ik . Karena banyaknya cara mengabilk bi¡ ¢
¡ ¢
P
. Selanjutnya
langan adalah nk . sehingga Sk =
|Ai1 i2 i3 ...ik | = nk ! = n!
k!
menurut prinsip inklusi eksklusi diperoleh
µ ¶
µ ¶
n
k n
| Ai | = n! −
(n − 1) + ... + (−1)
.(n − k)! + ... + (−1)n
1
k
i=1
·
¸
1
1
1
= n! 1 − + + ... +
1! 2!
n!
n
\
14. Letakkan sebuah kuda catur didalam papan catur berukuran 7x7. apakah
mungkin setiap kuda catur secara serempak dapat bergerak menurut aturan yang berlaku?
Pembahasan:
Asumsikan papan catur tersebut diwarnai dengan pola seperti umumnya.
Papan tersebut mempunyai 49 kotak, misalkan 24 diantaranya putih dan
25 warna hitam. Dengan mempertimbangkan kedua catur yang terletak
dalam kotak hitam, jika setiap kuda catur membuat gerakan sesuai aturan,
maka kuda catur tersebut harus bergerak menuju 25 kotak putih. Karena
hanya ada 24 kotak putih yang tersedia, maka ada sebuah gerakan yang
tidak dapat dilakukan. Jadi kuda catur tidak dapat bergerak secara serempak.
15. Letakkan sebuah kuda catur ke dala papan catur berukuran 4xn. Apakah
mungkin kuda catur dapat menempati setiap kotak dan kembali ke kotak
awal dengan gerakan berurutan dalam 4n kali gerakan?
Pembahasan:
Sebelumnya perhatikan untuk kasus papan catur berukuran 7x7. Misalkan
pada awalnya kuda catur terletak dikotak warna hitam. Pada gerakan
pertama, kuda catur akan bergerak menempati kotak berwarna hitam.
Chapter 9. Kombinatorika
146
Demikian seerusnya. Sehingga agar kuda catur dapat kembali menempati posisinya semula dibutuhkan sejumlah gerakan genap. Karena diiginkan kda catur dapat menemati setiap kotak maka dibutuhkan 49 gerakan harus menempati bilangan genap. Jadi dapat disimpulkan bahwa
kuda catur tidak dapat menempati setipa kotak dan kembali ke kotak awal
sesuai dengan aturan.
Selanjutnya untuk kasus diatas, perhatikan bahwa 4xn adalah bilangan
genap. Untuk menanganinya, warnai papan catur dengan pola yang berbeda,
contohnya seperti gambar dibawah. Perhatikan bahwa gerakan kuda catur
dari kotak putih diatas dan dibawah akan mengantarkan kekotak putih
baris kedua dan ketiga. Demikian sebaliknya gerakan dari kotak putih
baris kedua dan ketiga akan mengantarkan ke kotak putih baris pertama
dan keempat. Terdapat n kotak putih di baris pertama dan keempat yang
hanya dapat dicapai dari n kotak putih baris kedua dan ketiga. Hal ini
berakibat jalur kuda catur tidak akan bergerak dari kotak putih ke kotak
hitam. Jadi, kuda catur tidak mungkin dapat menempati setiap kotak dan
kembali ke posisi semula.
16. Misalkan a1 , a2 , ..., a2n+1 adalah himpunan bilangan bulat yang mempunyai sifat jika salah satunya ada yang dipindahkan maka bilangan yang tersisa dapat dibagi dalam dua himpunan bilangan bulat yang jumlahnya
sama. Buktikan bahwa a1 = a2 = ... = a2n+1
Pembahasan:
147
Chapter 9. Kombinatorika
Misalkan A = a1 + a2 + ... + a2n+1 . Dari claim diatas berakibat bahwa
A−ai merupakan bilangan genap untuk setiap i. Jadi semua bilangan bulat
a1 , a2 , ..., a2n+1 mempunyai paritas yang sama. Notasikan a adalah bilangan bulat terkecil dari a1 , a2 , ..., a2n+1 . Kemudian dibentuk bi = ai − a untuk setiap i. Jelas bahwa b1 , b2 , ..., b2n+1 juga memenuhi sifat diatas. Karena
b1 , b2 , ..., b2n+1 mempunyai paritas yang sama dan salah satu dan salah satunya merupakan bilangan nol, maka paritas dari b1 , b2 , ..., b2n+1 adalah bilangan genap. Andaikan terdapat bilangan diantara b1 , b2 , ..., b2n+1 yang
bukan bilangan nol. Notasikan k adalah bilangan bulat terbesar sedemikian
hingga 2k membagi habis setiap bi . Dibentuk ci =
bi
2k
untuk setiap i. Jelas
bahwa ci , c2 , ..., c2n+1 juga memenuhi sifat diatas. Karena k adalah bilangan
bulat terbesar yang membagi habis setiap bi , maka terdapat ci yang merupakan bilangan ganjil sedangkan salah satu dari ci , c2 , ..., c2n+1 merupakan
bilangan genap. Jadi ci , c2 , ..., c2n+1 tidak mempunyai paritas yang sama.
Kontradiksi dengan yang diketahui bahwa ci , c2 , ..., c2n+1 memenuhi sifat
diatas. Jadi tidak ada bilangan di antara b1 , b2 , ..., b2n+1 yang bukan bilangan nol, sehingga terbukti bahwa a1 = a2 = ... = a2n+1
17. Misalkan an adalah barisan yang terdiri dari n suku dimana suku-suku
1, 2atau3. Jika barisan ini tidak memiliki dua angka 1 yang berurutan, hitunglah berapa banyak barisan a10 !
Pembahasan:
Misalkan Sn menyatakan banyaknya barisan an yang tidak memiliki dua
angka satu yang berurutan. Perhatikan bahwa Sn dapat dibagi menjadi
dua bagian yaitu Sn,1 yaitu banyaknya barisan an yang diawali dengan
angka 1 dan tidak memiliki dua angka 1 dan Sn,2,3 adalah banyaknya barisan
an yang diawali dengan angka 2 dan 3 dan tidak memiliki dua angka 1.
Jelas bahwa Sn,1 diperoleh dengan meletakkan angka 2 atau 3 di awal
unsur-unsur Sn−1 .Disamping itu pada setiap unsur di dalam Sn,2,3 angka
148
Chapter 9. Kombinatorika
kedua pasti 1 dan barisan n − 2 bersal dari Sn−2 . Dari sini diperoleh Sn =
Sn−1 + 2Sn−2
18. Notasi Qn menyatakan banyaknya cara meletakkan benteng catur pada
papan catur berukuran nxn sedemikian hingga pengaturannya simetri terhadap diagonaldari sudut kiri bawah dengan sudut kanan atas. Buktikan
bahwa Qn = Qn−1 + (n − 1)Qn−2 !
Pembahasan:
Sebuah benteng pada kolom pertama mungkin atau tidak mungkin menduduki kotak pada sudut kiri bawah. Jika iya, maka terdapat Qn−1 cara
meletakkan sisa n − 1 benteng. Jika tidak, maka benteng dapat diletakkan
di n − 1 kotak pada kolom pertama. Peletakan benteng yang pertama
menentukan lokasi kesimettrisannya pada baris pertama. n − 2 benteg
sisanya kemudian diletakkan dengan Qn−2 benteng sisanya kemudian dapat diletakkan dengan Qn−2 cara. Dari sini maka didapatkan Qn = Qn−1 +
(n − 1)Qn−2 .
19. Sebuah koin diundi n kali. Berapakah peluang didapatkan dua kepala
secara berurutan?
Pembahasan:
Notasikan Pn sebagai peluang bahwa dua kepla secara berurutan tidak
muncul dalam n lemparan. Jelas bahwa P1 = 1, P2 =
3
.
4
Untuk n > 2,
maka ada kasus:
(a) Jika lemparan pertama adalah ekor, maka dua ke[ala secara berurutan
tidak muncul di sisa n − 1 undian berikutnya dengan peluang Pn−1 .
(b) Jika lemparan pertama muncul kepala, maka pada undian kedua harus
kepala untuk menghindari dua kepala muncul berurutan. Kemudian
dua kepala tidak akan muncul pada n−1 sisa undian berikutnya dengan peluang Pn−2 .
149
Chapter 9. Kombinatorika
Dari sini didapatkan Pn =
1
P
2 n−1
+ 41 Pn−2 , n > 2 transformasi ini dapat
dirubah kedalam bentuk rekurensi yang lebih umum dengan mengalikan
2n sehingga didapatkan 2n Pn = 2n−1 Pn−1 + 2n−2 Pn−2 Misalkan Sn = 2n Pn ,
akan didapatkan Sn = Sn−1 + Sn−2
20. Misalkan n sebuah bilangan bulat yang lebih besar atau sama dengan 3.
Berikan suatu tafsiran kombinatorial langsung bagi identitas berikut
µ
¶
n+1
=3
4
Pembahasan:
Ambil n titik pada sebuah lingkaran yang berpusat di O. Banyaknya tali
¡ ¢
busur yang terbentuk dari titik ini adalah n2 dan banyaknya pasangan
tali busur adalah ... Selanjutnya perhatikan n + 1 titik termasuk n titik
yang diberika dan titik pusat O. Dapat diambil 4 dari n + 1 titik ini untuk
¡ ¢
meperoleh n+1
kombinasi. Jika kombinasi demikian ini mengandung O,
4
maka akan diperoleh 6 ruas garis yang 3 diantaranya adalah jari-jari dan 3
¡¢
lainnya tali busur. Ketiga tali busur ini kemudian menhasilkan 32 pasangan tali busur dengan tepat satu titik sekutu terhadap suatu pasanga yang
dihasilkan. Misalkan {O, A, B, C} menghasilkan tali busur AB, AC, BC
dan tiga pasangan {AB, AC}, {AB, BC} dan {AC, BC}. Selanjutnya su¡¢
atu himpunan 4 titik tidak termasuk O menghasilkan 42 tali busur dan
¡6¢
pasangan tali busur. Dari 15 pasangan tali busur ini, terdapat 3 pasan2
gan tali busur yang tidak memiliki sekutu. Misalkan {O, A, B, C} menghasilkan tiga pasangan tali busur {AB, AC}, {AB, BC} dan {AC, BC}.
Download